You are on page 1of 147

MODULE IN

MATHEMATICS IN THE MODERN WORLD

Department of Mathematics
SCHOOL OF ACCOUNTANCY, MANAGEMENT, COMPUTING and INFORMATION STUDIES

Property
Property of and
of and forfor
thethe exclusive
exclusive useuse of SLU.
of SLU. Reproduction,
Reproduction, storing
storing in ainretrieval
a retrieval system,
system, distributing,
distributing, uploading
uploading or or posting
posting online,
online, or or transmitting
transmitting in any
in any form
form or or
by by
anyany
means,
means, electronic,
electronic, mechanical,
mechanical, photocopying,
photocopying, recording,
recording, or otherwise
or otherwise of any
of any partpart of this
of this document,
document, without
without the prior
the prior written
written permission
permission of SLU,
of SLU, is strictly
is strictly prohibited. 1
prohibited.
GMATH
INSERT RELATED PICTURE HERE

COURSE LEARNING OUTCOMES


At the end of the module, you should
be able to:
1. Discuss and argue about the
nature of mathematics, what is,
how is expressed, represented,
and used.
2. Use different types of reasoning
to justify statements and
arguments made about
mathematics and mathematical
concepts.
3. Discuss the language and
symbols of mathematics.
4. Use a variety of statistical tools to
process and manage numerical
data.
5. Use mathematics in other areas

MATHEMATICS IN such as finance, voting, health


and medicine, business,

THE MODERN
environment, arts and design,
and recreation.
6. Appreciate the nature and uses
WORLD of mathematics in everyday life.
7. Affirm honesty and integrity in the
application of mathematics to
various human endeavors.

Property of and for the exclusive use of SLU. Reproduction, storing in a retrieval system, distributing, uploading or posting online, or transmitting in any form or by any
means, electronic, mechanical, photocopying, recording, or otherwise of any part of this document, without the prior written permission of SLU, is strictly prohibited. 2
COURSE INTRODUCTION

This course deals with the nature of mathematics, appreciation of its practical,
intellectual and aesthetic dimensions, and application of Mathematical tools in daily life. The
course begins with an introduction to the nature of mathematics as an exploration of
patterns (in nature and the environment) and as an application of inductive and deductive
reasoning. By exploring these topics, students are encouraged to go beyond the typical
understanding of mathematics as merely a set of formulas but as a source of aesthetics in
patterns of nature, for example, and a rich language in itself (and of science) governed by
logic and reasoning. The course then proceeds to survey ways in which mathematics
provides a tool for understanding and dealing with various aspects of present-day living,
such as managing personal finances, making social choices, appreciating geometric
designs, understanding codes used in data transmission and security, and dividing limited
resources fairly. These aspects will provide opportunities for actually doing mathematics in a
broad range of exercises that bring out the various dimensions of mathematics as a way of
knowing, and test the students’ understanding and capacity. Further, it instils appreciation
towards God’s creation and encourages stewardship and preservation of natural resources.

Property of and for the exclusive use of SLU. Reproduction, storing in a retrieval system, distributing, uploading or posting online, or transmitting in any form or by any
means, electronic, mechanical, photocopying, recording, or otherwise of any part of this document, without the prior written permission of SLU, is strictly prohibited. 3
TABLE OF CONTENTS
Page
Module 1: The Nature of Mathematics……………………………………………………... 6

Learning Reinforcement 1……………………………………………………………………. 12

Module 2 : Mathematics as a study of Pattern ………………………………………….. 13

Learning Reinforcement 2……………………………………………………………………. 20

Module 3 : Mathematics as a Language………………………………………………….. 22

Learning Reinforcement 3……………………………………………………………………. 44

Module 4 : Reasoning and Problem Solving………………………………………………. 46

Learning Reinforcement 4……………………………………………………………………. 73

Module 5 : Mathematics of Finance………………………………………………………... 76

Learning Reinforcement 5……………………………………………………………………. 87

Module 6 : Data Management ……………………………………………………………… 89

Learning Reinforcement 6……………………………………………………………………. 120

Module 7 : Mathematics of Graphs…………………………………………………………. 122

Learning Reinforcement 7……………………………………………………………………. 127

Module 8 : Linear Programming ……………………………………………………………. 131

Learning Reinforcement 8……………………………………………………………………. 143

References……………………………………………………………………………………….. 144

MATHEMATICS IN THE MODERN WORLD

Property of and for the exclusive use of SLU. Reproduction, storing in a retrieval system, distributing, uploading or posting online, or transmitting in any form or by any
means, electronic, mechanical, photocopying, recording, or otherwise of any part of this document, without the prior written permission of SLU, is strictly prohibited. 4
When asked how they feel about studying mathematics, most would comment
regarding their anxieties and fear about the subject. We were trained differently. Those
notions of ours should change.
Let us now look at the pandemic we are facing. We are tasked to stay home, keep
social or in a strict sense physical distancing. These are recommendations because of the
nature of transmission of the virus from one who is infected, there is a great possibility that
they will infect ten others. These ten would also infect 10 each. Thus it is exponential in nature.
There is a need to flatten the curve.
The terminologies that are being used are that of mathematics. The calculations that
are made become the basis of decision-making. That is why there is a great need to
understand mathematics. It is not numbers. When we have the essence regarding the
number, then it becomes more meaningful to us. Mathematics is a foreign language to us,
that is why at times we do not simply understand it.
Kdramas are better, though it is in a foreign language, we have translations in the
forms of subtitles being seen. But as a comparison, the mathematics language is simpler for
us to understand.
We have been using this language unknowingly. It is a matter of appreciation and
realization that mathematics is important. Its utility is far more than the utilities of the other
courses you have been learning.
Mathematics is the language in which God has created the Universe. It is the
language of God.
I hope that with this in mind, respect to our neglected and feared course would again
be given proper credence. That is why we are encouraging you to learn with us.
Mathematics has its Greek root in the word mathemata which means fond of learning. When
we strive to learn and have fun in it, then we become mathematicians.
The major objective of our course is to understand the nature and language of
Mathematics and the wonderful utilities in it.
Mathematics: just do it. It is not a watch and learn. It is where we learn. Once again,
MATHEMATICS: JUST DO IT!

Property of and for the exclusive use of SLU. Reproduction, storing in a retrieval system, distributing, uploading or posting online, or transmitting in any form or by any
means, electronic, mechanical, photocopying, recording, or otherwise of any part of this document, without the prior written permission of SLU, is strictly prohibited. 5
MODULE 1: THE NATURE OF MATHEMATICS

This module is an introductory part. I would be presenting the definition and nature
of Mathematics. A video presentation “Math from a Biblical Worldview” @
https://www.youtube.com/watch?v=YcKi2t54djk helps us out in looking at its aesthetic value.
The target Learning outcomes of our Module 1, particularly for topics 1 and 2, are: a)
Articulate the importance of mathematics in one’s life; b) Express appreciation for
mathematics as a human endeavor.

Pre-assessment Question:
In 150 words explain the role of Mathematics in your field?

UNIT 1: THE MEANING OF MATHEMATICS AND ITS CHARACTERISTICS


PowerPoint 01: Definitions of Mathematics and Its characteristics.

Mathematics may be viewed from different perspectives. In its wildest significance, it


is the development of all types of formal deductive reasoning. Generally, it is said to be the
science of calculation. Others view it as a science of numbers and space, and others say it
is a science of measurement, quantity, and magnitude. Locke said, “Mathematics is a way
to settle in the mind of children a habit of reasoning.” It is a discipline investigating “formal
structures” (Bernays), it is the “science of orders” (Russell), it is the “science of order in
progression” (Hamilton). Mathematics has also been seen as a logical construct based on
many axioms of either set theory or number theory.
Traditionally, mathematics is presented deductively at school. And is often perceived
as well structured, and problems are algorithmically approached. Merriam dictionary
defines mathematics as the science of numbers and their operations, interrelations,
combinations, generalizations, abstractions, and space configurations of their structure,
measurement, transformations, and generalizations.
Mathematics is derived from the ancient word manthanein meaning "to learn." The
Greek root mathesis means "knowledge" or its other form máthema meaning science,
knowledge, or learning, and mathematikós or mathemata means "fond of learning." These
might have been the notion of the early mathematicians and philosophers that is why they
continue to seek knowledge and the truth. Mathematics could then be defined as a desire
for a particular kind of knowing. Knowing that is self-contained on the individual or may be
seen as autonomous thinking (Schaaf, 1963).
Mathematics is described in so many ways that it fits within the area of human
knowledge. Basically, it is seen as a study of patterns and relations. It is also a way of thinking.

Property of and for the exclusive use of SLU. Reproduction, storing in a retrieval system, distributing, uploading or posting online, or transmitting in any form or by any
means, electronic, mechanical, photocopying, recording, or otherwise of any part of this document, without the prior written permission of SLU, is strictly prohibited. 6
Mathematics is seen as an art that is characterized by order and internal consistency. It is a
language that uses carefully defined terms and symbols. Thus, mathematics is a tool (Reys,
Lindquist, Lambdin, Smith, and Suydam, 2004).
Mathematics has five essential characteristics’ namely: precision, definition,
reasoning, coherence, and purposefulness. They are not independent of each other. It is
precise in the sense that mathematical statements are clear and unambiguous. It is clear
what is known and what is not known. Definitions abound in mathematics. It is the bedrock
of mathematical structure and the platform that supports reasoning. Reasoning is the
lifeblood of mathematics. It is the engine that drives proving and problem-solving. Its
absence is the root cause of the learning by rote approach. Concepts and skills are
interwoven in mathematics. And lastly, mathematics is goal-oriented, and for every concept
or skill, there is a purpose for it.
Hardy (1941) states that the beauty of mathematics resides in the fact that
mathematics is all about not just patterns but patterns of ideas. Devlin defines mathematics
as the “science of patterns” and then more fully as: “the science of order, patterns, structure,
and logical relationships” (Devlin, 2001, p. 73). Mathematics has also often been described
as the language of science. Since the mixture of symbols and words is so powerfully
descriptive and communicative, perhaps the definition: “mathematics is the language of
the science of order, patterns, structure, and logical relationships” may be considered.
Whatever form or way one defines mathematics so long as it becomes meaningful to the
user, it would be a definition for that user.

UNIT 2: MATHEMATICS IN NATURE


PowerPoint 02: Mathematics in Nature, Shapes

This module would give you a perspective that mathematics is not numbers and
operations. It is more than that. We have been emphasizing that God created the universe
mathematically. Everything you see around you is Mathematics.
The target learning outcomes of module 2 are: a) Identify patterns in nature and
regularities in the world; b) Articulate the importance of mathematics in one’s life; c) Express
appreciation for mathematics as a human endeavor

To start with, lets watch this video entitled Nature’s Mathematics 1 and 2
https://www.youtube.com/watch?v=VE_RU0fNjt0
https://www.youtube.com/watch?v=n2WHNMfRmHE&t=643s

Euclid said that "The laws of nature are but the mathematical thoughts of God."
Galileo affirmed by stating that “Mathematics is the language in which God has written the
Universe.”
Mathematics is everywhere. It is seen anywhere in the universe. With the development
of a formal system of thought for recognizing, classifying, and exploiting patterns, one could

Property of and for the exclusive use of SLU. Reproduction, storing in a retrieval system, distributing, uploading or posting online, or transmitting in any form or by any
means, electronic, mechanical, photocopying, recording, or otherwise of any part of this document, without the prior written permission of SLU, is strictly prohibited. 7
systematize and organize these ideas of patterns. It would be here that we could discover
great secrets of nature’s patterns. They are not just there to be admired; they are vital clues
to the rules that govern the natural process.
Analyzing thoroughly, having the essentials of mathematics as our basis, we could
further discover mathematics in our world and unravel the mystery of the universe. The
majority of our knowledge of mathematics and modern science is strictly based and
supported by our environmental observations. What was once seen as the randomness of
nature is now distinguished as the intricate applications of mathematics and illustrates the
complexities of our natural world. Here are a very few properties of mathematics that are
depicted in nature.

A. SHAPES

Geometry is the branch of mathematics that basically describes shapes and


establishes the relationships between them. Figures with regular shapes are categorized as
polygons. Polygons are fascinating, especially when they are approximated in nature. When
looking carefully, one can see them all around us.
Spatial patterns can be represented by a fairly small collection of fundamental
geometrical shapes and relationships that have corresponding symbolic representations.
The human mind relies heavily on its perception of shapes and patterns to make sense of the
world. The artifacts around us (such as buildings, vehicles, toys, utensils, and basic things we
use in life) and the familiar forms we see in nature (such as animals, leaves, stones, flowers,
and the moon and sun) can often be characterized in terms of geometric form. Some of the
ideas and terms of geometry have become part of everyday language. Although real
objects never perfectly match a geometric figure, they more or less approximate them. The
properties and characteristics of geometric figures and
relationships can be associated with objects. For many purposes, it is sufficient to be familiar
with points, lines, planes; triangles, rectangles, squares, circles, and ellipses; rectangular solids
and spheres; relationships of similarity and congruence; relationships of convex, concave,
intersecting, and tangent; angles between lines or planes; parallel and perpendicular
relationships between lines and planes; forms of symmetry such as displacement, reflection,
and rotation; and the Pythagorean theorem.
Both shape and measurement (magnitude) or scale can have important
consequences for the performance of systems. For example, triangular connections
maximize rigidity, smooth surfaces minimize turbulence, and a spherical container minimizes
surface area for any given mass or volume. Changing the size of objects while keeping the
same shape can have profound effects due to scaling geometry: Area varies as the square
of linear dimensions, and volume varies as the cube. On the other hand, some fascinating
kinds of patterns known as fractals look very similar to one another when observed at any
scale whatever—and some natural phenomena (such as the shapes of clouds, mountains,
and coastlines) seem to be like that.

Property of and for the exclusive use of SLU. Reproduction, storing in a retrieval system, distributing, uploading or posting online, or transmitting in any form or by any
means, electronic, mechanical, photocopying, recording, or otherwise of any part of this document, without the prior written permission of SLU, is strictly prohibited. 8
Some Common Shapes Visible in Nature

Sphere. A sphere is a perfectly round geometrical object in three-


dimensional space, such as the shape of a round ball. The
shape of the Earth is very close to that of an oblate spheroid,
a sphere flattened along the axis from pole to pole such
that there is a bulge around the equator.

Hexagons. A hexagon is a Two-dimensional six-sided


closed polygon. For a beehive, close packing is
important to maximize the use of space. Hexagons fit
most closely together without any gaps; so hexagonal
wax cells are what bees create to store their eggs and
larvae.

Cones. A cone is a three-dimensional


geometric shape that tapers smoothly from a flat,
usually circular base to a point called the apex or
vertex. Volcanoes 2.5 form cones, the steepness
and height of which depends on the runniness
(viscosity) of the lava. Fast, runny lava forms flatter cones; thick, viscous
lava forms steep-sided cones.

Parallel lines. In mathematics, parallel lines stretch to infinity,


neither converging or diverging. The parallel dunes in
the Australian desert aren't perfect - the physical world
rarely is.

Interesting Figures in Nature

Fractals. Like other figures, fractals are also geometric figures. Fractals involve dilation.
They are objects with fractional dimensions and most have self-similarity. Self-similarity is when
small parts of objects, when magnified resemble the same figure. The boundaries are of
infinite length and are not differentiable anywhere (never smooth enough to have a tangent
at a point). Thus, fractals have basic components that are similar to the whole. This means
that you can find similar shapes even if you zoom the figure. However, fractals involve a
complex process because it goes through an infinite number of iterations.
Fractals can also be observed in nature such as trees, flowers, clouds, ocean waves,
etc. Even the human body also has a fractal structure.

Property of and for the exclusive use of SLU. Reproduction, storing in a retrieval system, distributing, uploading or posting online, or transmitting in any form or by any
means, electronic, mechanical, photocopying, recording, or otherwise of any part of this document, without the prior written permission of SLU, is strictly prohibited. 9
Fractals possess the following characteristics: self-similarity, fractional dimension, and
formation by iteration.
Natural objects exhibit scaling symmetry, but only over a limited range of scales. They
also tend to be approximately self-similar, appearing more or less the same at different scales
of measurement. Sometimes this means that they are statistically self-similar; that is to say,
they have a distribution of elements that are similar under magnification.

Below are some manifestations of fractals in nature.

Ferns Clouds Bacterial Colony

Lightning Waterfalls Mountains

The world around us seems to make up several distinct patterns, evolving various
complex steps of formation. However, looking more deeply we see many similarities and
resemblances. The numerous models explained above have no experimental proof and
may not be correct, but they definitely show linkages between patterns formed under highly
contrasting natural conditions e.g. (a zebra coat and sand dunes) and also show that the
mechanisms between the formations of these patterns need not necessarily be complex.

B. SYMMETRY
PowerPoint 03: Symmetry

Symmetry is a type of invariance: a property that something does not change under
a set of transformations. It is a mapping of the object onto itself which preserves the structure.
Symmetry in everyday language refers to a sense of harmonious and beautiful proportion
and balance. Although these two meanings of "symmetry" can sometimes be told apart,
they are related. Plainly, symmetry is when a figure has two sides that are mirror images of

Property of and for the exclusive use of SLU. Reproduction, storing in a retrieval system, distributing, uploading or posting online, or transmitting in any form or by any
means, electronic, mechanical, photocopying, recording, or otherwise of any part of this document, without the prior written permission of SLU, is strictly prohibited. 10
one another. It would then be possible to draw a line through a picture of the object and
along either side the image would look exactly the same. This line would be called a line of
symmetry.

There are Two Kinds of Symmetry

One is bilateral symmetry in which an object has two sides that


are mirror images of each other. The human body would be an
excellent example of a living being that has bilateral symmetry.

The other kind of symmetry is radial symmetry. This is where


there is a center point and numerous lines of symmetry could be
drawn. The most obvious geometric example would be a circle.

C. PATTERNS
PowerPoint 04: Patterns in Nature

Though every living and non-living thing of the world may seem to follow a pattern of
its own, looking deeply into the geometry and mechanism of the pattern formation can lead
you to classify them into merely two categories broadly:
1. Self-organized patterns/ Inherent organization
2. Invoked organization

Self-Organized patterns/ Inherent organization. A self-organizing pattern follows a


simple set of rules, and they use only local information to determine how a particular subunit
evolves. Successive patterns represent them. This pattern can be described as successive
horizontal rows; the 'successor' pattern is just under its predecessor. A complex pattern
develops when the basic rule just defined is applied to that row (the active row) and then
to subsequent rows. Thus, self-organization is a process in which patterns at the global level
of a system emerge solely from numerous interactions among the lower-level components
of the system. Moreover, the rules specifying interactions among the system’s components
are executed using only local information, without reference to the global pattern. In other
words, the pattern is an emergent property of the system rather than a property imposed on
the system by an external influence.
Therefore, if rules are useful for understanding life patterns, such as the stripes on a
zebra's coat, there must be a specific rule. The zebra's coat alternates in contrasting areas
of light and dark pigmentation. Hence, the zebra's coat patterns reflect the early interaction
of chemicals as they diffused through the embryonic skin.

Property of and for the exclusive use of SLU. Reproduction, storing in a retrieval system, distributing, uploading or posting online, or transmitting in any form or by any
means, electronic, mechanical, photocopying, recording, or otherwise of any part of this document, without the prior written permission of SLU, is strictly prohibited. 11
Invoked Organization. Not all patterns that occur in nature arise through self-
organization. A weaver bird uses its own body as a template to build the hemispherical egg
chamber of the nest. A spider when creating a web follows a genetically determined recipe
in relation to its sticky orb and the various radii and spirals it creates. A similar invoked
organization is that of the honeycomb made by bees. In these cases, the structures are built
by an architect that oversees and imposes order and pattern. There are no subunits that
interact with one another to generate a pattern. Each animal acts like a stonemason or
laborer, measuring, fitting, and moving pieces into place.

This is an example of social insect architecture: the wasp nests, an example of an


invoked organization in nature.
More than this biological system existing on the patterns created in nature, great
mathematics is embedded in it aside from the plain geometric figure. The geometry of the
patterns could also be linked to mathematical numbers directly or indirectly. The series of
numbers seem to have been forced on them. The Fibonacci numbers or sequence and the
Golden ratio are but the primary example of them.

Learning Reinforcement 1

Directions: Submit your output as a single pdf file in the submission


bin for this activity in the Classroom.

Search or take pictures of revelations of Mathematics in


nature. 1 picture that exhibit Golden Ratio and another
picture that exhibit Fibonacci Sequence. Explain each
picture in 150 words how does each picture exhibit said
pattern. Cite your references. Output must be placed in
a short bond paper.

Property of and for the exclusive use of SLU. Reproduction, storing in a retrieval system, distributing, uploading or posting online, or transmitting in any form or by any
means, electronic, mechanical, photocopying, recording, or otherwise of any part of this document, without the prior written permission of SLU, is strictly prohibited. 12
MODULE 2: MATHEMATICS AS A STUDY OF PATTERNS

Patterns are everywhere. We predict and forecast using the previous patterns that we
have been observed. It becomes the basis of our decisions. Pattern recognition is also always
an integral part of board examination or psychological tests. This module will help you
develop critical thinking and abstract reasoning.
Our target learning outcomes for this module are the following: a) solve problems
involving patterns; b) express appreciation for mathematics as a human endeavor; and c)
articulate the importance of mathematics in one’s life.

View the following videos to supplement this module:


God’s Fingerprint – The Fibonacci Sequence – Golden Ratio
https://www.youtube.com/watch?v=4VrcO6JaMrM
The Fractal Nature of Reality.
https://www.youtube.com/watch?v=K_1KK5O85n0
Why is 1.618034 So Important
https://www.youtube.com/watch?v=keLN89CWZ-A

Mathematics is the science of patterns and relationships. Patterns provide a sense of


order. It allows one to make an educated guess. A lot of disciplines are based on making
hypotheses, and hypotheses are often based on patterns. Assumptions are also based on
recurring patterns. Thus, the understanding of patterns aids in developing mental skills
needed in the transformation of ideas to information then to knowledge.
As a theoretical discipline, mathematics explores the possible relationships among
abstractions without concern for whether those abstractions have counterparts in the real
world. Investigating the patterns that one finds in numbers, shapes, and expressions would
make mathematical discoveries. Patterns, relationships, and functions constitute a unifying
theme of mathematics. Patterns are an effective way to demonstrate the relationship
between variables. It provides opportunities to model and analyzes situations. The
opportunities to analyze, extend, and create a variety of patterns and to use pattern-based
thinking to understand and represent mathematical and other real-world phenomena are
of great essence in developing mathematical skills. It gives opportunities for problem-solving,
making and verifying generalizations, and building mathematical understanding and
confidence.
The abstractions can be anything from strings of numbers or letters to geometric
figures to measurements to sets of equations. Inquiries such as "Does the interval between
prime numbers forms a pattern?" “Is there an expression for the change in the surface area
of any regular solid as its volume approaches zero?”; needs investigatory approaches of
patterns.

Property of and for the exclusive use of SLU. Reproduction, storing in a retrieval system, distributing, uploading or posting online, or transmitting in any form or by any
means, electronic, mechanical, photocopying, recording, or otherwise of any part of this document, without the prior written permission of SLU, is strictly prohibited. 13
To recognize patterns, one needs to understand critical thinking and logical
reasoning, essential skills everyone needs. These are major components in problem-solving
and are involved in data analysis.
A central line of investigation of patterns is identifying a small set of basic ideas and
rules evident in each sequence. One usually looks at the differences between succeeding
items. The difference may be arithmetical in nature, but not always. At times one investigates
the movement of shapes or the relation of the sequences of letters. Logical and abstract
reasoning are great tools in investigating these mathematical patterns.
Patterns essentially provide a means of recognizing broader aspects that can be
shoved down to arrive at a specific answer to a particular problem. The benefit of
understanding patterns opens many doors where there is the knowledge that can be
applied. The study of patterns allows one to develop the sense of observation, hypothesizing,
experimenting, discovering, and creating. Predicting and estimating and thought-provoking
activities of recognizing patterns emphasize the role and purpose of mathematics itself.

A. LOGIC PATTERNS

Logic patterns are related to geometric patterns and number patterns. It helps us
classify objects or figures. One kind of logic pattern deals with the characteristics of various
objects. Another kind deals with orders: there is a sequence of objects and a pattern in the
attributes the objects possess. They are commonly part of aptitude tests. The development
of numeracy and geometric literacy follows after one develops a sense of patterns involving
logic or reasoning through association or relation.

Watch the following videos to learn more tips in dealing with logic patterns:
 IQ and Aptitude Test Questions, Answers, and Explanations
https://www.youtube.com/watch?v=Yo_6xhG-Bcc
 Non-Verbal Reasoning Test Tips and Tricks for Job Tests & Interviews
https://www.youtube.com/watch?v=CzMbznbN3xk

Property of and for the exclusive use of SLU. Reproduction, storing in a retrieval system, distributing, uploading or posting online, or transmitting in any form or by any
means, electronic, mechanical, photocopying, recording, or otherwise of any part of this document, without the prior written permission of SLU, is strictly prohibited. 14
Example 1. A diagonal connects two vertices that are not already connected by an
edge in a polygon.

How many diagonals can be drawn in a hexagon?


Explanation:
Connect non-adjacent vertices of each polygon.

We can see the triangle has no diagonals because each vertex has only adjacent
vertices. Therefore, the number of diagonals in a polygon triangle is 0. The rectangle has two
diagonals, while the pentagon has five diagonals. Hence, the hexagon has nine diagonals.
The figure is shown below:

Example 2. Look at the following pattern.

How many circles will be in the next figure in the pattern?


Explanation:
From the pattern, the next figure would be

Hence, there are 40 circles.

Property of and for the exclusive use of SLU. Reproduction, storing in a retrieval system, distributing, uploading or posting online, or transmitting in any form or by any
means, electronic, mechanical, photocopying, recording, or otherwise of any part of this document, without the prior written permission of SLU, is strictly prohibited. 15
B. NUMBER PATTERNS

A number pattern is an ordered list of numbers, called terms, which follow a certain
property or sequence. The sequence of numbers can be formed by performing repeated
arithmetic operations following a certain rule on the terms of the pattern.

An arithmetic sequence is made by adding a fixed number to the previous term to


obtain the next term. The fixed number added is called the common difference. The
sequence 1, 5, 9, 13, 17, and so on, is an example of an arithmetic sequence. Its first term is
1 and the common difference is 4, which is added repeatedly to a term to get the next
number in the pattern.

Another way to form a number pattern is to multiply a fixed number to the terms of
the sequence. This number pattern is called a geometric sequence and the fixed number
multiplied to the previous term to get the next term is called the common ratio. For example,
the sequence 4, 16, 64, ..., is a geometric sequence in which 4 is the first term and 4 is also
the common ratio. The sequence 32, 16, 8, 4, 2, ..., and so on is another example in which
the common ratio is ½.

Other number patterns can be formed using arbitrary rules. There are patterns where
the resulting sequence has numbers in increasing or decreasing order. In this pattern, the
amount that is added to the terms changes every time in a predictable manner. For
example, consider the sequence 4, 5, 7, 10, 14, 19, ... . Starting with the first term, the numbers
1, 2, 3, 4 and so on are added to the previous term to get the next term.

Another number pattern that also occurs in nature is the well-known Fibonacci
sequence 0, 1, 1, 2, 3, 5, 8, 13, 21, and so on. It is a sequence of numbers that starts with 0
and 1, and every term after the first two is obtained by adding up the two terms before it.

Several interesting number patterns can also be found in Pascal's Triangle (named
after Blaise Pascal, a famous French Mathematician, and Philosopher). To build the triangle,
start with 1 at the top and continue placing numbers below it in a triangular pattern. Every
new row of numbers begins and ends with 1, and other numbers in the row are the sum of
the numbers directly above it. The Fibonacci sequence (except the first term 0) can be
obtained from this configuration, as shown in the figure, by getting the sum of numbers in the
same colored squares.

Property of and for the exclusive use of SLU. Reproduction, storing in a retrieval system, distributing, uploading or posting online, or transmitting in any form or by any
means, electronic, mechanical, photocopying, recording, or otherwise of any part of this document, without the prior written permission of SLU, is strictly prohibited. 16
To determine missing terms in a sequence, we first have to know how the terms of a
sequence are related. It may be arithmetic or geometric. Sometimes it helps to get
differences (or quotients) between consecutive terms (e.g. 2 nd term – 1st term, 3rd term – 2nd
term, etc) to see the rule. At times, one needs to recognize a property or characteristic the
terms of the sequence have. In the case of the sequence 1, 8, 27, 64, …, the terms are cubes:
13 , 23 , 33 , and 43 . So, the next terms are 53 = 125 and 63 = 216.

Example 3. Find the next number in the pattern 45, 48, 51, 54, 57, ___
Solution:
The sequence is arithmetic, with a common difference of 3. That is, there is a repeated
addition of 3 to get the next term. Thus, the sixth term is 60.

Example 4. Find out the missing number in the pattern 67, 74, 81, 88, __, 102
Solution:
Here, the pattern is obtained through repeated addition of 7 (arithmetic with
common difference 7). Therefore, the missing term in the above pattern is 95.

Example 5. What is the next number in the pattern 108, 102, 96, ___
Solution:
Here, the pattern is the repeated subtraction of 6 (arithmetic with common difference
-6). Therefore, the next number in the above pattern is 90 for the subtraction of 6.

Example 6. What is the next number in the pattern 5, 14, 27, 44, 65, ___
Solution:
Taking differences between consecutive terms (called first differences), the pattern is
not an arithmetic sequence since the first differences (shown in row 2 of the figure that
follows) are not all the same. In such a situation, it is often helpful to compute the successive
differences of the first differences. The differences of the first differences are called the
second differences. The differences of the second difference are called the third differences.

Property of and for the exclusive use of SLU. Reproduction, storing in a retrieval system, distributing, uploading or posting online, or transmitting in any form or by any
means, electronic, mechanical, photocopying, recording, or otherwise of any part of this document, without the prior written permission of SLU, is strictly prohibited. 17
Sequence 5 14 27 44 65 _____

First difference 9 13 17 21

Second difference 4 4 4

To predict the next term of a sequence, we often look for a pattern in a row of
differences. For instance, in the following table, the second differences are all equal 4. If the
pattern continues, then a 4 would also be the next second difference, and we can extend
the table to the right as shown.

5 14 27 44 65 _____

9 13 17 21

4 4 4 4
Now we work upward. That is, we add 4 to the first difference 21 to produce the next
first difference, 25. We then add this difference to the fifth term, 65, to predict that 90 is the
next term in the sequence. This process can be repeated to predict additional terms of the
sequence.
5 14 27 44 65 90
_____

9 13 17 21 25

4 4 4 4

C. GEOMETRIC PATTERNS

A geometric pattern is a kind of pattern formed from sequences of lines and curves
to form geometric shapes and figures. It is a motif, pattern, or design depicting abstract,
nonrepresentational shapes such as lines, circles, ellipses, triangles, rectangles, and
polygons. We generally associate geometric patterns with wallpaper designs and tiling. They
could also be associated with number patterns and used to predict the next geometric
shape that would follow the sequence.

Property of and for the exclusive use of SLU. Reproduction, storing in a retrieval system, distributing, uploading or posting online, or transmitting in any form or by any
means, electronic, mechanical, photocopying, recording, or otherwise of any part of this document, without the prior written permission of SLU, is strictly prohibited. 18
Example 7. Determine what shape, figure, or series of figure would follow the sequence

Answer:
From the sequence, the next pattern is

Example 8. Determine what shape, figure, or series of figure would follow the sequence

_______ _________ _______

Answer:
From the sequence, the next pattern is

Property of and for the exclusive use of SLU. Reproduction, storing in a retrieval system, distributing, uploading or posting online, or transmitting in any form or by any
means, electronic, mechanical, photocopying, recording, or otherwise of any part of this document, without the prior written permission of SLU, is strictly prohibited. 19
Learning Reinforcement 2

Directions: Write your solutions and answers on a clean sheet of


paper, or you may print this page and answer there. Submit the
image of your HANDWRITTEN SOLUTIONS as a single pdf file in the
submission bin for this activity in the Classroom. You may use
image scanning apps on your phone (CamScanner or Tap
Scanner) to save several images into one pdf file, or place your
images in a document and save them as a pdf file.

A. Write the missing term/s in the given series of numbers.


1. Find the next three terms in:  14,  10,  6,  2, ? , ? , ? .
2. Find the 35th term in the arithmetic sequence 3, 9, 15, 21, ? .
3. Find the next two terms in the sequence: 5, 2, 8, 3, 11, 4, 14, 5, 17, 6, ? , ? .
4. Find the next number: 2, 3, 4, 6, 6, 9, 8, ? .
5. What should become in place of x in the following series: 2, 2, 4, 12, 48, 240, x ?
6. Identify the next number in the following series 2, 8, 26, 62, 122, 212, ? .
7. What is the next number in the pattern 4, 12, 36, 108, ? .
B. Multiple choice: Write the letter of the correct answer on the space provided before
each number.

8. Which of the following completes the sequence?

A B C D

9. Which figure comes next in the series?

A B C D

Property of and for the exclusive use of SLU. Reproduction, storing in a retrieval system, distributing, uploading or posting online, or transmitting in any form or by any
means, electronic, mechanical, photocopying, recording, or otherwise of any part of this document, without the prior written permission of SLU, is strictly prohibited. 20
10. Which figure comes next in the sequence?

A B C D

11. Which shape completes the sequence?

A B C D

12. Which of the following shapes is the missing section?

A B C D

Property of and for the exclusive use of SLU. Reproduction, storing in a retrieval system, distributing, uploading or posting online, or transmitting in any form or by any
means, electronic, mechanical, photocopying, recording, or otherwise of any part of this document, without the prior written permission of SLU, is strictly prohibited. 21
MODULE 3: THE LANGUAGE OF MATHEMATICS

Our target learning outcomes in this module are a) discuss the language, symbols and
conventions of mathematics; b) explain the nature of mathematics as a language; c)
perform operations on mathematical expressions correctly; and d) cite examples that
highlight the importance of mathematics as a language.
Mathematics is known as the language of Science. Different notations and
terminologies are commonly used in learning and studying Mathematics. Having a
systematic notation using different symbols and graphical presentations makes learning
Mathematics compact and focused. People have trouble understanding mathematics
because the ideas are difficult, but because they cannot understand a foreign language.
The Language of Mathematics, particularly the symbols being used in Mathematics,
has three characteristics:
1. Precise - must be able to distinguish different thoughts or make very fine distinctions.
2. Concise - must be able to say things briefly.
3. Powerful - must be able to express complex thoughts with ease.
Numbers are the most commonly known symbol that represents Mathematics. It
specifies a certain value or quantity. We currently use the Hindu-Arabic number system
represented by ten symbols 0, 1, 2, 3, 4, 5, 6, 7, 8, and 9.

UNIT 1: SETS

In all sorts of situations, we classify objects into sets of similar objects and count them.
This procedure is the most basic motivation for learning the whole numbers and learning how
to add and subtract them.
In our ordinary language, we try to make sense of the world we live in by classifying
collections of things. The English language has many words for such collections. For example,
we speak of ‘a flock of birds’, ‘a herd of cattle,’ ‘a swarm of bees’ and ‘a colony of ants.’
We do a similar thing in mathematics and classify numbers, geometrical figures, and
other things into collections that we call sets.
A set is a well-defined collection of distinct objects. Objects that belong in a set are
called its elements or members of the set. Consider the following examples:
1. The set of denominations of Philippine currency.
2. The set of even numbers.
3. The set of prime numbers.
4. The set of consonants in the English alphabet.
5. 𝐸 = {𝑥: 𝑥 2 − 3𝑥 + 2 = 0}

Property of and for the exclusive use of SLU. Reproduction, storing in a retrieval system, distributing, uploading or posting online, or transmitting in any form or by any
means, electronic, mechanical, photocopying, recording, or otherwise of any part of this document, without the prior written permission of SLU, is strictly prohibited. 22
A set must be well defined. This means that our description of the elements of a set is
unambiguous. For example, a group of beautiful people is not a set because people
disagree about what ‘beautiful’ means.

A. Notation
A set is usually denoted by a capital letter, such as 𝐴, 𝐵, 𝐶, 𝑋, 𝑌, 𝑍, …, whereas lower-
case letters, 𝑎, 𝑏, 𝑐, 𝑥, 𝑦, 𝑧, … are used to denote the elements of sets. Elements of a set are
enclosed in braces/curly brackets { }.
Consider 𝑆 = {𝑤, 𝑥, 𝑦, 𝑧}. We say that 𝑤 belongs to 𝑆, or equivalently 𝑤 is an element
of 𝑆. In symbols, this is written as: 𝑤 ∈ 𝑆. But 𝑝 is not an element of 𝑆, so we write 𝑝 ∉ 𝑆.
Take note that in listing the elements of a set, repetition of an element is ignored. A
set remains the same even if its elements are repeated or arranged. Say for instance, the set
𝑊 = {𝑎, 𝑎, 𝑏} has only the two elements 𝑎 and 𝑏. The second mention of 𝑎 is an unnecessary
repetition and can be ignored.

B. Describing a Set
We describe a set in two ways, namely:
1. A set can be described by listing all of its elements. This is called the tabular
form of the set or the roster method.
2. A set can also be described by writing a description of its elements between
braces. This is called the set-builder notation or the rule method

Example 9. Write 𝐴 = {𝑎, 𝑒, 𝑖, 𝑜, 𝑢} using the set-builder notation.


Answer:
𝐴 = {𝑥: 𝑥 𝑖𝑠 𝑎 𝑙𝑒𝑡𝑡𝑒𝑟 𝑖𝑛 𝑡ℎ𝑒 𝐸𝑛𝑔𝑙𝑖𝑠ℎ 𝑎𝑙𝑝ℎ𝑎𝑏𝑒𝑡, 𝑥 𝑖𝑠 𝑎 𝑣𝑜𝑤𝑒𝑙} or
𝐴 = {𝑥: 𝑥 𝑖𝑠 𝑎 𝑣𝑜𝑤𝑒𝑙 𝑖𝑛 𝑡ℎ𝑒 𝐸𝑛𝑔𝑙𝑖𝑠ℎ 𝑎𝑙𝑝ℎ𝑎𝑏𝑒𝑡}. Here, 𝑒 ∈ 𝐴 but 𝑞 ∉ 𝐴.

Example 10. Write 𝐵 = {𝑥: 𝑥 𝑖𝑠 𝑎𝑛 𝑒𝑣𝑒𝑛 𝑖𝑛𝑡𝑒𝑔𝑒𝑟, 𝑥 > 0} using the roster method.
Answer:
𝐵 = {2, 4, 6, 8, 10, … }

Example 11. Write 𝐸 = {𝑥: 𝑥 2 − 3𝑥 + 2 = 0} using the roster method.


Answer:
Here, 𝐸 = {𝑥: 𝑥 2 − 3𝑥 + 2 = 0} is read as: 𝐸 is the set of all 𝑥 such that 𝑥 2 − 3𝑥 + 2 = 0,
meaning 𝐸 consists of those numbers which are solutions of the equation 𝑥 2 − 3𝑥 + 2 = 0. To
get the solution set, we solve the quadratic equation 𝑥 2 − 3𝑥 + 2 = 0. Factoring the left side
of the equation, we have: (𝑥 − 2)(𝑥 − 1) = 0. Then, equating each factor to zero, we
obtain
𝑥 − 2 = 0 implies 𝑥 = 2, and
𝑥 − 1 = 0 implies 𝑥 = 1.
Hence, 𝐸 = {1, 2}.

Property of and for the exclusive use of SLU. Reproduction, storing in a retrieval system, distributing, uploading or posting online, or transmitting in any form or by any
means, electronic, mechanical, photocopying, recording, or otherwise of any part of this document, without the prior written permission of SLU, is strictly prohibited. 23
C. Hierarchy of Numbers

(Z’)

1. The set of numbers whose elements are the natural numbers and zero is called the
set of whole numbers, denoted by 𝑾. We write 𝑾 = {0, 1, 2, 3, 4, … }

2. The set of integers, denoted by 𝒁, is composed of the set of whole numbers and
the negative integers. We write 𝒁 = {… , −3, −2, −1, 0, 1, 2, 3, … }

3. The set of rational numbers, denoted by 𝑸, are those numbers that can be
expressed as a quotient of two integers 𝑝 and 𝑞, where 𝑞 ≠ 0. Examples are
fractions, repeating decimals, and terminating decimals.
𝑝
We write, 𝑸 = {𝑥: 𝑥 𝑐𝑎𝑛 𝑏𝑒 𝑟𝑒𝑝𝑟𝑒𝑠𝑒𝑛𝑡𝑒𝑑 𝑏𝑦 𝑞
, 𝑝 𝑎𝑛𝑑 𝑞 𝑎𝑟𝑒 𝑖𝑛𝑡𝑒𝑔𝑒𝑟𝑠, 𝑞 ≠ 0}

4. The set of irrational numbers, denoted by 𝑸′, consists of numbers whose decimal
representations are non-terminating and non-repeating that cannot be expressed
as the quotient of two integers. Examples: 𝜋, 𝑒, √2

5. The set of real numbers, denoted by 𝑹, is the union of 𝑸 and 𝑸′

6. The set of complex numbers, denote by 𝑪, are numbers of the form 𝑎 + 𝑏𝑖, where
a and b are real numbers and 𝑖 2 = −1. In symbols,
𝑪 = {𝑎 + 𝑏𝑖: 𝑎, 𝑏 𝑎𝑟𝑒 𝑟𝑒𝑎𝑙 𝑛𝑢𝑚𝑏𝑒𝑟𝑠 𝑎𝑛𝑑 𝑖 2 = −1}
Note that the set 𝑵 = {1, 2, 3, … } of positive integers is also known as the set of natural
numbers.

Property of and for the exclusive use of SLU. Reproduction, storing in a retrieval system, distributing, uploading or posting online, or transmitting in any form or by any
means, electronic, mechanical, photocopying, recording, or otherwise of any part of this document, without the prior written permission of SLU, is strictly prohibited. 24
Elementary Properties of the Real Numbers
We investigate here properties of real numbers. Here are some rules concerning the
addition and multiplication of the real numbers 𝑅:
a. Associative law for addition and multiplication:
(𝑎 + 𝑏) + 𝑐 = 𝑎 + (𝑏 + 𝑐)
(𝑎𝑏)𝑐 = 𝑎(𝑏𝑐)
b. Commutative law for addition and multiplication:
𝑎 + 𝑏 = 𝑏 + 𝑎
𝑎𝑏 = 𝑏𝑎
c. Distributive law for addition and multiplication:
𝑎(𝑏 + 𝑐) = 𝑎𝑏 + 𝑎𝑐
d. Additive and multiplicative identities: There exists a zero element 0 and a unity
element 1 such that for any 𝑎 ∈ 𝑅,
𝑎 + 0 = 0 + 𝑎 = 𝑎
𝑎 · 1 = 1 · 𝑎 = 𝑎
The Real Line: Order and Inequalities
One important property of real numbers is that they can be represented graphically
by points on a straight line.

Let 𝑎 and 𝑏 be real numbers. We say that a is less than b, written as 𝑎 < 𝑏, if the
difference 𝑏 − 𝑎 is positive, meaning that 𝑎 lies to the left of the point 𝑏 on the number line.
The following additional notation and terminologies are used:
𝑎 > 𝑏, read: 𝑎 is greater than 𝑏
𝑎 ≤ 𝑏, means 𝑎 < 𝑏 𝑜𝑟 𝑎 = 𝑏, read: 𝑎 is less than or equal to 𝑏
𝑎 ≥ 𝑏, read: 𝑎 is greater than or equal to 𝑏
We say, 2 < 5, −6 < −1, 6 ≥ 3. The statement 2 < 𝑥 < 7 means 2 < 𝑥 and 𝑥 < 7,
hence 𝑥 lies between 2 and 7 on the real line 𝑅

Basic Properties of Inequality Relations:


1. Let 𝑎, 𝑏, 𝑐 be real numbers. Then,
 𝑎 ≤ 𝑎
 If 𝑎 ≤ 𝑏 and 𝑏 ≤ 𝑎, then 𝑎 = 𝑏.
 If 𝑎 ≤ 𝑏 and 𝑏 ≤ 𝑐, then 𝑎 ≤ 𝑐.
2. Law of Trichotomy: For any real numbers 𝑎 and 𝑏, exactly one of the following holds:
𝑎 > 𝑏, 𝑎 = 𝑏, 𝑎 < 𝑏
3. Let 𝑎, 𝑏, 𝑐 be real numbers such that 𝑎 ≤ 𝑏. Then
● 𝑎+𝑐 ≤ 𝑏+𝑐
● 𝑎𝑐 ≤ 𝑏𝑐 when 𝑐 > 0; but 𝑎𝑐 ≥ 𝑏𝑐 when 𝑐 < 0.

Property of and for the exclusive use of SLU. Reproduction, storing in a retrieval system, distributing, uploading or posting online, or transmitting in any form or by any
means, electronic, mechanical, photocopying, recording, or otherwise of any part of this document, without the prior written permission of SLU, is strictly prohibited. 25
D. Basic Concepts

Equal sets. Two sets are said to be equal if they have exactly the same elements.

Example 12. Given 𝐸 = {𝑥: 𝑥 2 − 3𝑥 + 2 = 0} and 𝐹 = {2, 1}, we say that 𝐸 and 𝐹 are equal
sets. We write 𝐸 = 𝐹 since each consists precisely of the elements 1 and 2. On the other
hand, the sets 𝐶 = {1, 3, 5} and 𝐷 = {1, 2, 3} are not equal, because they have different
elements. We write 𝐶 ≠ 𝐷.

Finite and Infinite sets. A finite set is a set that has a finite (definite or fixed) number of
elements, meaning that we can list all their elements. An infinite set is a set that is not finite.
It has an infinite number of elements. Examples of infinite sets are the set of even whole
numbers and the set of integers greater than 2000.

Example 13. 𝑆 = {𝑥: 𝑥 𝑖𝑠 𝑎 𝑤ℎ𝑜𝑙𝑒 𝑛𝑢𝑚𝑏𝑒𝑟, 2000 < 𝑥 < 2005} is a finite set. Here, the elements
are 2001, 2002, 2003 and 2004. 𝑀 = {𝑥: 𝑥 𝑖𝑠 𝑎𝑛 𝑖𝑛𝑡𝑒𝑔𝑒𝑟, 2000 < 𝑥 < 3000} is also finite. The
elements of this set are the numbers 2001, 2002, 2003, …, 2999.

The Numbers of Elements of a Set. The cardinality of a set A, denoted by |A|, is the
number of elements in set A. A set has cardinality ℵ0 if and only if it is countably infinite, there
is a one-to-one correspondence between the set and the set of natural numbers. The symbol
ℵ0 (aleph-naught, aleph-zero, or the German term aleph-null) is the cardinality of all natural
numbers and is an infinite cardinality. Also, the cardinality of the set of real numbers
(cardinality of the continuum c) is 2ℵ0 .

Example 14. If 𝑆 = { 1, 3, 5, 7, 9 }, then the cardinality of set 𝑆 is 5, written as |𝑆| = 5 . If 𝐴 =


{ 1001, 1002, 1003, … , 3000 }, then cardinality of set 𝐴 is 2000, written as |𝐴| = 2000.

Equivalent Sets. Two sets are said to be equivalent if they both have the same
cardinality. Note that all equal sets are equivalent, but not all equivalent sets are equal sets.

Example 15. Sets 𝐶 = {1, 3, 5} and 𝐷 = {1, 2, 3} are equivalent sets since |𝐶 | = |𝐷| = 3 but
𝐶 ≠ 𝐷. Equal sets 𝐸 = {𝑥: 𝑥 2 − 3𝑥 + 2 = 0} and 𝐹 = {2, 1} are also equivalent sets since |𝐸 | =
|𝐷| = 2.

Universal and Empty Sets. The universal set, denoted by 𝑈 is the set that contains all
the elements being considered. Examples are the following:
1. In plane geometry, the universal set consists of all the points in the plane
2. In human population studies, the universal set consists of all the people in the
world

Property of and for the exclusive use of SLU. Reproduction, storing in a retrieval system, distributing, uploading or posting online, or transmitting in any form or by any
means, electronic, mechanical, photocopying, recording, or otherwise of any part of this document, without the prior written permission of SLU, is strictly prohibited. 26
The symbol 𝜙 𝑜𝑟 { } represents the empty set, more commonly known as the null set,
which is the set that has no elements at all. Thus, |𝜙| = 0. There is only one empty set: If L and
M are both empty, then 𝐿 = 𝑀 = 𝜙 since they have exactly the same elements, namely,
none.

Joint and Disjoint Sets. Two sets are said to be joint if they have at least one common
element. Otherwise, they are disjoint.

Example 16. Consider sets 𝐶 = {1, 3, 5} and 𝐷 = {1, 2, 3}. We say that sets 𝐶 and 𝐷 are joint
sets since 1 and 3 are elements common to them. If we consider sets 𝐺 = {6, 7, 9} and 𝐶 =
{1, 3, 5}, then we say that sets 𝐶 and 𝐺 are disjoint sets because they have no common
elements.

Subsets, Proper Subset, Improper Subset. Sets of things are often further subdivided.
For instance, owls are a particular type of bird, so every owl is also a bird. We express this in
the language of sets by saying that the set of owls is a subset of the set of birds. A set, say 𝐴,
is called a subset of another set, say 𝐵, if every element of 𝐴 can be found in 𝐵. This is denoted
by 𝐴 ⊆ 𝐵 (read this as “𝐴 is a subset of 𝐵”). If 𝐴 is a subset of 𝐵, and there is an element of 𝐵
that is not in 𝐴, then 𝐴 is called a proper subset of 𝐵. This is denoted by 𝐴 ⊂ 𝐵. If 𝐴 is any set,
then 𝐴 is the improper subset of 𝐴. Any other subset of 𝐴 is a proper subset of 𝐴.

Example 17. If 𝐴 = {2, 4, 6} and 𝐵 = {0, 1, 2, 3, 4, 5, 6}, then 𝐴 ⊆ 𝐵 because every element of A
is an element of B. Consider sets 𝐶 = {1, 3, 5} and 𝐷 = {1, 2, 3}. We say that C is not a subset
of D, denoted by 𝐶 ⊈ 𝐷, because 5 is an element of C that is not found in D. Also, we say
that 𝐷 ⊈ 𝐶.

The following properties should be noted:


1. Every set 𝐴 is a subset of the universal set 𝑈 because by definition, all the
elements of 𝐴 are elements of 𝑈. The empty set is a subset of 𝐴. In symbols, ∅ ⊆
𝐴 ⊆ 𝑈.
2. Every set is a subset of itself because the elements of 𝐴 belong to 𝐴 (the
improper subset of 𝐴. In symbols, 𝐴 ⊆ 𝐴.
3. If 𝐴 ⊆ 𝐵 and 𝐵 ⊆ 𝐶, then 𝐴 ⊆ 𝐶.
4. 𝐴 = 𝐵 if and only if 𝐴 ⊆ 𝐵 and 𝐵 ⊆ 𝐴.

Property of and for the exclusive use of SLU. Reproduction, storing in a retrieval system, distributing, uploading or posting online, or transmitting in any form or by any
means, electronic, mechanical, photocopying, recording, or otherwise of any part of this document, without the prior written permission of SLU, is strictly prohibited. 27
Example 18: Suppose that 𝑈 = {𝑥: 𝑥 ∈ 𝑍}. Consider the following sets as examples:
𝐴 = {𝑥: 𝑥 ∈ 𝑍, −2 ≤ 𝑥 ≤ 4} 𝐷 = {𝑥: 𝑥 ∈ 𝑖𝑠 𝑎𝑛 𝑒𝑣𝑒𝑛 𝑝𝑟𝑖𝑚𝑒 }
𝐵 = {𝑥: 𝑥 ∈ 𝑍, 𝑥 ≤ 0 } 𝐸 = {−1, 0, 1, 2, 3}
𝐶 = {𝑥: 𝑥 ∈ 𝑍, 1 ≤ 𝑥 ≤ 7} 𝐹 = {−2, −1, 0, 1, 2, 3, 4}
Find the following:
Solution:
Before we answer the questions, let us list first the elements of the sets above. We
have:
𝐴 = {−2, −1, 0, 1, 2, 3, 4} 𝐷 = {2}
𝐵 = {… , −2, −1, 0} 𝐸 = {−1, 0, 1, 2, 3}
𝐶 = {1, 2, 3, 4, 5, 6, 7} 𝐹 = {−2, −1, 0, 1, 2, 3, 4}

a. Which sets are finite? Which are infinite?


Answer:
 The finite sets are 𝐴, 𝐶, 𝐷, 𝐸 and 𝐹.
 𝐵 is an infinite set.
b. What is the cardinality of the sets 𝐴, 𝐵, 𝐶, 𝐷, 𝐸, 𝐹?
Answer:
|𝐴| = 7 |𝐶| = 7 |𝐸| = 5
|𝐵| = ℵ0 |𝐷| = 1 |𝐹| = 7
c. Which pair of sets is equal?
Answer: 𝐴 and 𝐹 are equal sets.
d. Which pair of sets is equivalent?
Answer: 𝐴, 𝐶 and 𝐹 are equivalent sets.
e. Identify pairs of joint and disjoint sets.
Answer:
 Joint sets: 𝐴&𝐵, 𝐴&𝐶, 𝐴&𝐷, 𝐴&𝐸, 𝐴&𝐹, 𝐵&𝐸, 𝐵&𝐹, 𝐶&𝐷, 𝐶&𝐸, 𝐶&𝐹,
𝐷&𝐸, 𝐷&𝐹, 𝐸&𝐹
 Disjoint sets: 𝐵&𝐶, 𝐵&𝐷
f. Which set is a proper subset of 𝐴?
Answer:
❏ 𝐷 ⊂ 𝐴 because 2 belongs to 𝐴.
❏ 𝐸 ⊂ 𝐴 because -1, 0, 1, 2, and 3 are all in 𝐴.
g. Which set is an improper subset of 𝐴?
Answer:
Since 𝐴 = 𝐹, we say that 𝐹 is an improper subset of 𝐴.

Property of and for the exclusive use of SLU. Reproduction, storing in a retrieval system, distributing, uploading or posting online, or transmitting in any form or by any
means, electronic, mechanical, photocopying, recording, or otherwise of any part of this document, without the prior written permission of SLU, is strictly prohibited. 28
E. Set Operations

Union of Sets. For two sets 𝐴 and 𝐵, the union of 𝑨 and 𝑩, denoted by 𝐴 ∪ 𝐵, is
defined as the set which contains all distinct elements found on either A or B.
In symbols, 𝐴 ∪ 𝐵 = {𝑥: 𝑥 ∈ 𝐴 𝑜𝑟 𝑥 ∈ 𝐵}.

Example 19: Let 𝐴 = {1, 2, 3, 4, 5} and 𝐵 = {3, 4, 6, 7}. Then, 𝐴 ∪ 𝐵 = {1, 2, 3, 4, 5, 6, 7}.

Intersection of Sets. For two sets 𝐴 and 𝐵, the intersection of 𝑨 and 𝑩, denoted by 𝐴 ∩
𝐵, is defined as the set which contains all elements which can be found on both 𝐴 and 𝐵.
In symbols, 𝐴 ∩ 𝐵 = {𝑥: 𝑥 ∈ 𝐴 𝑎𝑛𝑑 𝑥 ∈ 𝐵}.

Example 20: Let 𝐴 = {1, 2, 3, 4, 5} and 𝐵 = {3,4, 6, 7}. Getting the common elements of 𝐴
and 𝐵 gives 𝐴 ∩ 𝐵 = {3, 4}.
1.
Complement of a Set. The complement of a set 𝐴 is the set containing elements of
the universal set 𝑈 which are not in 𝐴.
In symbols, 𝐴𝑐 = {𝑥: 𝑥 ∉ 𝐴, 𝑥 ∈ 𝑈}.
Other notations for the complement of 𝐴 are 𝐴′ or 𝐴.

Example 21: Let 𝐴 = {1, 2, 3, 4, 5} and 𝑈 = {1, 2, 3, 4, 5, 6, 7, 8, 9, 10}. Removing the elements of
𝐴 from the elements of the universal set we obtain, 𝐴𝐶 = {6, 7, 8, 9, 10}.

Difference of Sets. The difference of the sets 𝐴 and 𝐵, denoted by 𝐴\𝐵 (or 𝐴 − 𝐵) is the
set of elements which belong to 𝐴 but not in 𝐵.
In symbols, 𝐴\𝐵 = {𝑥: 𝑥 ∈ 𝐴, 𝑥 ∉ 𝐵}.

Example 22: Let 𝐴 = {1, 2, 3, 4, 5} and 𝐵 = {3, 4, 6, 7}. Removing the elements of 𝐵 from the set
𝐴, we obtain 𝐴\𝐵 = {1, 2, 5}.

Cartesian Product of Sets. Let 𝐴 and 𝐵 be two sets. The product set or Cartesian
product of 𝐴 and 𝐵, denoted by 𝐴 × 𝐵, is the set of ordered pairs (𝑎, 𝑏) such that 𝑎 ∈ 𝐴 and
𝑏 ∈ 𝐵.
In symbols, 𝐴 × 𝐵 = {(𝑎, 𝑏): 𝑎 ∈ 𝐴 , 𝑏 ∈ 𝐵}.
Note here that 𝐴 × 𝐵 ≠ 𝐵 × 𝐴 and the cardinality of the product set of 𝐴 and 𝐵,
denoted by |𝐴 × 𝐵|, is |𝐴 × 𝐵| = |𝐴| ∙ |𝐵|.

Example 23: Let 𝐴 = {1, 2, 3, 4, 5} and 𝐵 = {3,4}. Then


𝐴 × 𝐵 = {(1,3), (1,4), (2,3), (2,4), (3,3), (3,4), (4,3), (4,4), (5,3), (5,4)}
𝐵 × 𝐴 = {(3,1), (3,2), (3,3), (3,4), (3,5), (4,1), (4,2), (4,3), (4,4), (4,5)}
Note that |𝐴 × 𝐵| = 5 × 2 = 10.

Property of and for the exclusive use of SLU. Reproduction, storing in a retrieval system, distributing, uploading or posting online, or transmitting in any form or by any
means, electronic, mechanical, photocopying, recording, or otherwise of any part of this document, without the prior written permission of SLU, is strictly prohibited. 29
Example 24: Consider the following sets:
𝑈 = {1, 2, 3, 4, 5, 6, 7, 8, 9, 10} 𝐴 = {2, 4, 6, 8, 10}
𝐵 = {1, 3, 6, 9} 𝐶 = {4, 7, 9, 10}
List the elements of the following sets:
a. (𝐴 ∪ 𝐵)𝑐 ∪ 𝐶
b. 𝐵 ∩ (𝐶 ∪ 𝐴𝐶 )
c. 𝐴𝑐 \(𝐴 ∪ 𝐵)𝑐
d. 𝐴𝐶 × [𝐴𝑐 \(𝐴 ∪ 𝐵)𝑐 ]
Solution:

a. (𝐴 ∪ 𝐵)𝑐 ∪ 𝐶
Since (𝐴 ∪ 𝐵) = {1, 2, 3, 4, 6, 8, 9, 10}, it follows that (𝐴 ∪ 𝐵)𝑐 = {5, 7}. Hence
(𝐴 ∪ 𝐵)𝑐 ∪ 𝐶 = {4, 5, 7, 9, 10}.
b. 𝐵 ∩ (𝐶 ∪ 𝐴𝐶 )
Since 𝐴𝐶 = {1, 3, 5, 7, 9}, we have (𝐶 ∪ 𝐴𝐶 ) = {1, 3, 4, 5, 7, 9, 10}. Thus
𝐵 ∩ (𝐶 ∪ 𝐴𝐶 ) = {1, 3, 9}.
c. 𝐴𝑐 \(𝐴 ∪ 𝐵)𝑐
From (a), we obtain (𝐴 ∪ 𝐵)𝑐 = {5, 7}.
Removing the elements of (𝐴 ∪ 𝐵)𝑐 from 𝐴𝐶 = {1, 3, 5, 7, 9}, we obtain
𝐴𝑐 \(𝐴 ∪ 𝐵)𝑐 = {1, 3, 9}.
d. 𝐴𝐶 × [𝐴𝑐 \(𝐴 ∪ 𝐵)𝑐 ]
We know that 𝐴𝐶 = {1, 3, 5, 7, 9} and 𝐴𝑐 \(𝐴 ∪ 𝐵)𝑐 = {1, 3, 9}. Then
𝐴𝐶 × [𝐴𝑐 \(𝐴 ∪ 𝐵)𝑐 ] = {(1,1), (1,3), (1,9), (3,1), (3, 3), (3, 9), (5, 1), (5, 3), (5, 9), (7, 1),
(7, 3), (7, 9), (9,1), (9, 3), (9,9)}

F. Laws on Sets and Set Operations


Commutative Law. The order in which the sets appear in a union or intersection
operation does not affect the result.
 𝐴∪𝐵 = 𝐵∪𝐴
 𝐴∩𝐵 = 𝐵∩𝐴
Associative Law. The grouping of operations involving union or intersection operations
does not affect the result.
 𝐴 ∪ (𝐵 ∪ 𝐶) = (𝐴 ∪ 𝐵) ∪ 𝐶
 𝐴 ∩ (𝐵 ∩ 𝐶) = (𝐴 ∩ 𝐵) ∩ 𝐶
Identity Laws
 𝐴∪∅ = 𝐴
 𝐴∩𝑈 = 𝐴
Distributive Laws
 𝐴 ∪ (𝐵 ∩ 𝐶 ) = (𝐴 ∪ 𝐵) ∩ (𝐴 ∪ 𝐶)
 𝐴 ∩ (𝐵 ∪ 𝐶 ) = (𝐴 ∩ 𝐵) ∪ (𝐴 ∩ 𝐶)
 (𝐴 ∪ 𝐵) ∩ 𝐶 = (𝐴 ∩ 𝐶) ∪ (𝐵 ∩ 𝐶)

Property of and for the exclusive use of SLU. Reproduction, storing in a retrieval system, distributing, uploading or posting online, or transmitting in any form or by any
means, electronic, mechanical, photocopying, recording, or otherwise of any part of this document, without the prior written permission of SLU, is strictly prohibited. 30
 (𝐴 ∩ 𝐵) ∪ 𝐶 = (𝐴 ∪ 𝐶) ∩ (𝐵 ∪ 𝐶)
De Morgan’s Laws
 (𝐴 ∪ 𝐵)𝐶 = 𝐴𝐶 ∩ 𝐵𝐶
 (𝐴 ∩ 𝐵)𝐶 = 𝐴𝐶 ∪ 𝐵𝐶

G. Venn Diagrams

Diagrams make mathematics easier because they help us


to see the whole situation at a glance. The English mathematician
John Venn (1834–1923) began using diagrams to represent sets.
His diagrams are now called Venn diagrams.
A Venn diagram is a pictorial representation of sets where
enclosed areas in the plane represent sets. In a Venn diagram,
the universal set is generally drawn as a large rectangle, and then
subsets of the universal set are represented by circles within this
rectangle.
Suppose 𝑆 is a subset of 𝑇. In The Venn diagram, we place the circle representing 𝑆
inside the circle representing 𝑇. Say for instance, let 𝑆 = { 0, 1, 2 } and 𝑇 = { 0, 1, 2, 3, 4 }. Using
the Venn diagram, we illustrate 𝑆 ⊂ 𝑇 as follows:

𝑻
3
4 𝐒
0
2 1

We can also illustrate the hierarchy of numbers using a Venn diagram.


Imaginary
Numbers

ℕ W ℤ ℚ ℚ′

Property of and for the exclusive use of SLU. Reproduction, storing in a retrieval system, distributing, uploading or posting online, or transmitting in any form or by any
means, electronic, mechanical, photocopying, recording, or otherwise of any part of this document, without the prior written permission of SLU, is strictly prohibited. 31
If A and B are arbitrary sets, it is possible that some elements are in A but not in B, some
elements are in B but not in A, some in both A and B, and some are in neither A nor B. We
illustrate these possibilities using a Venn diagram.

A. Illustration of disjoint sets B. Union of A and B

C. Intersection of A and B D. Complement of set A

E. Difference of A and B Note that the shaded region represents


the elements of the set in B, C, D, and E.

Problem Solving with Venn Diagrams


Let us look at a few examples that demonstrate how Venn diagrams can make
problem-solving much easier.

Example 25: A travel agent surveyed 100 people to find out how many of them had visited
the cities of Melbourne and Brisbane. Thirty-one people had visited Melbourne, 26 people
had been to Brisbane, and 12 people had visited both cities. Draw a Venn diagram to find
the number of people who had visited
a. Melbourne or Brisbane
b. Brisbane but not Melbourne
c. only one of the two cities
d. Neither city

Property of and for the exclusive use of SLU. Reproduction, storing in a retrieval system, distributing, uploading or posting online, or transmitting in any form or by any
means, electronic, mechanical, photocopying, recording, or otherwise of any part of this document, without the prior written permission of SLU, is strictly prohibited. 32
Solution:

Let: M be the set of people who had visited Melbourne,


B be the set of people who had visited Brisbane
U be the universal set that represents the set of people surveyed

First, let us construct the Venn diagram:


 Consider the intersection; there were 12 people who had visited both Melbourne
and Brisbane, this means that |𝑀 ∩ 𝐵| = 12.
 There were 31 people who visited Melbourne. Hence, there were
|𝑀\𝐵| = 31 − 12 = 19 people who had visited Melbourne only.
 There were 26 people who had visited Brisbane. Hence, there were |𝐵\𝑀| = 26 −
12 = 14 people who had visited Brisbane only
 Since the travel agent surveyed 100 people, there were
100 − 12 − 19 − 14 = 55 people who had visited neither Melbourne nor Brisbane.

The Venn diagram below illustrates the information that we obtained:

We are now ready to answer the given questions:


a. The number of people who had visited Melbourne or Brisbane. This includes those
who had visited both cities, Melbourne only, and Brisbane only, or M ∪ B
Answer: 12 + 19 + 14 = 45

b. The number of people who had visited Brisbane but not Melbourne. This means
that we consider those who had visited Brisbane only, not including the
intersection.
Answer: 14
c. The number of people who had visited only one of the two cities. This includes only
those who had visited Melbourne only plus Brisbane only; the intersection is not
included.
Answer: 19 + 14 = 33
d. The number of people who had visited neither city: (We consider here the
number outside the two circles.)
Answer: 55

Property of and for the exclusive use of SLU. Reproduction, storing in a retrieval system, distributing, uploading or posting online, or transmitting in any form or by any
means, electronic, mechanical, photocopying, recording, or otherwise of any part of this document, without the prior written permission of SLU, is strictly prohibited. 33
Example 26: Consider the following data for 120 mathematics students:
65 study French, 20 study French and German
45 study German, 25 study French and Russian
42 study Russian, 15 study German and Russian
8 study all three languages
Answer the following:
a. Find the number of students studying at least one of the three languages.
b. Find the number of students studying: exactly one language and exactly two
languages.
Solution:
Let: F be the set of students studying French
G be the set of students studying German
R be the set of students studying Russian
U be the universal set representing all mathematics students

First, let us construct the Venn diagram:


 Consider the intersection of the 3 sets = 8 of them are studying the three
languages
 Those who are studying German and Russian only = 15 – 8 = 7
 Those who are studying French and Russian only = 25 – 8 = 17
 Those who are studying French and German only = 20 – 8 = 12
 Those who are studying French only = 65 – 17 – 12 – 8 = 28
 Those who are studying German only = 45 – 7 – 12 – 8 = 18
 Those who are studying Russian only = 42 – 17 – 7 – 8 = 10
 Those who are not studying the 3 languages
120 – 10 – 18 – 28 – 12 – 17 – 7 – 8 = 20

The Venn diagram below illustrates the information that we obtained:

F 12 G
18
28
8
17 7
10
R 20

Property of and for the exclusive use of SLU. Reproduction, storing in a retrieval system, distributing, uploading or posting online, or transmitting in any form or by any
means, electronic, mechanical, photocopying, recording, or otherwise of any part of this document, without the prior written permission of SLU, is strictly prohibited. 34
We are now ready to answer the given questions:
a. The number of students studying at least one of the three languages. (at least
one means greater than or equal to 1)
Answer:
1 language = 28 + 18 + 10 = 56
2 languages = 12 + 17 + 7 = 36
3 languages = 8__
Total = 100
b. The number of students studying exactly one language:
Answer: 28 + 18 + 10 = 56
The number of students studying exactly two languages:
Answer: 12 + 17 + 7 = 36

UNIT 2: RELATIONS AND FUNCTIONS

A. Relation
A relation is defined as a set of ordered pairs. More precisely, consider the
Cartesian product 𝐴 × 𝐵 = {(𝑎, 𝑏): 𝑎 ∈ 𝐴, 𝑏 ∈ 𝐵} of two sets 𝐴 and 𝐵. A relation R from 𝐴 to 𝐵 is
a subset of 𝐴 × 𝐵. For each pair 𝑎 ∈ 𝐴 and 𝑏 ∈ 𝐵, exactly one of the following is true:
I. (𝑎, 𝑏) ∈ 𝑅; we say “𝑎 is 𝑅-related to 𝑏” written 𝑎𝑅𝑏.
II. (𝑎, 𝑏) ∉ 𝑅; we say “𝑎 is not 𝑅-related to 𝑏”.
The following are examples of relations because they consist of a set of ordered pairs:
{(−2,5), (−1, 0), (2, −3)}
{(−1,0), (0, −3), (2, −3), (3,0), (4,5)}
{(3,0), (4,5)}
{(−2,5), (−1, 0), (0, −3), (1, −4), (2, −3), (3,0), (4,5)}
Some relations are very special and are used at almost all levels of mathematics. Set
inclusion ⊆, “less than”, “parallel to” are some relations used in mathematics.

Example 27 Let 𝐴 = {𝑎, 𝑏} and 𝐵 = {1,2,3}. Then


𝑅1 = {(𝑎, 1), (𝑏, 2), (𝑎, 3)}
𝑅2 = {(𝑎, 1), (𝑎, 2), (𝑎, 3)}
𝑅3 = {(𝑏, 1), (𝑏, 2)}
are relations from 𝐴 to 𝐵. We could say that 𝑎𝑅1 1, 𝑎𝑅2 3, and 𝑏𝑅3 2.

The domain of a relation 𝑅 from a set 𝐴 to a set 𝐵 is the set of all first elements of the
ordered pairs which belong to 𝑅, and since these first elements come from set 𝐴 then the
domain of 𝑅 is a subset of 𝐴. The range of a relation 𝑅 is the set of all second elements of the
ordered pairs which belong to 𝑅, and so it is a subset of 𝐵.

Property of and for the exclusive use of SLU. Reproduction, storing in a retrieval system, distributing, uploading or posting online, or transmitting in any form or by any
means, electronic, mechanical, photocopying, recording, or otherwise of any part of this document, without the prior written permission of SLU, is strictly prohibited. 35
B. Function
A function is a relation for which each value from the set of the first components of
the ordered pairs is associated with exactly one value from the set of second components
of the ordered pair.

Example 28: The following relation is a function:


{(−1, 0), (0, −3), (1, −4), (2, −3), (3,0), (4,5)}
We have the following sets of first components (i.e. the first number from each
ordered pair) and second components (i.e. the second number from each ordered pair).
1𝑠𝑡 𝑐𝑜𝑚𝑝𝑜𝑛𝑒𝑛𝑡𝑠: {−1, 0, 1, 2, 3, 4} 2𝑛𝑑 𝑐𝑜𝑚𝑝𝑜𝑛𝑒𝑛𝑡𝑠: {0, −4, −3, 0,5}
For the set of second components, notice that the “−3” occurred in two ordered pairs but
we only listed it once.
To see why this relation is a function, pick any value from the set of first components.
Now, go back up to the relation, find every ordered pair in which this number is the first
component, and list all the second components from those ordered pairs. The list of
second components will consist of exactly one value. That is, every first component is
paired with a unique value for the second component. Therefore, this relation is a
function.

Example 29: The following relation is not a function:


{(6, 10)(−7, 3)(0, 4)(6, −4)}
Here is the list of first and second components
1𝑠𝑡 𝑐𝑜𝑚𝑝𝑜𝑛𝑒𝑛𝑡𝑠: {6, −7, 0} 2𝑛𝑑 𝑐𝑜𝑚𝑝𝑜𝑛𝑒𝑛𝑡𝑠: {10, 3, 4, −4}
The list of second components associated with 6 has two values, so this relation is not a
function.

“Working Definition” of Function


A function can be thought of as an equation for 𝑦 in terms of 𝑥 for which any 𝑥 value
that can be plugged into or substituted into the equation will yield exactly one 𝑦 value out
of the equation.
The phrase “𝑥 that can be plugged into” implies that not all 𝑥 values can be
substituted into an equation. Further, when dealing with functions, we will always assume
that both 𝑥 and 𝑦 will be real numbers.

Function Notation
Let’s start off with the following quadratic equation: 𝑦 = 𝑥 2 − 5𝑥 + 3. You can use any
process to verify that this is a function. Since this is a function, we will denote it as follows,
𝑓(𝑥) = 𝑥 2 − 5𝑥 + 3.

So, we replaced the variable 𝑦 with the notation 𝑓(𝑥). This is read as “𝑓 of 𝑥.” Note
that there is nothing special about the 𝑓 we used here. We could just have easily used any
of the following letters to represent the function:

Property of and for the exclusive use of SLU. Reproduction, storing in a retrieval system, distributing, uploading or posting online, or transmitting in any form or by any
means, electronic, mechanical, photocopying, recording, or otherwise of any part of this document, without the prior written permission of SLU, is strictly prohibited. 36
𝑔(𝑥) = 𝑥 2 − 5𝑥 + 3 ℎ(𝑥) = 𝑥 2 − 5𝑥 + 3 𝑅(𝑥) = 𝑥 2 − 5𝑥 + 3

Evaluating functions
Evaluating a function is means determining the 𝑦 value for a given value of 𝑥. Let’s
take the function 𝑓 (𝑥) = 𝑥 2 − 5𝑥 + 3 and evaluate its value at 𝑥 = 4.
Now, when we say the value of the function we are asking what the value of the
equation is for that particular value of 𝑥, denoted by 𝑓(4):
𝑓(4) = (4)2 − 5(4) + 3 = 16 − 20 + 3 = −1

Example 30: Given 𝑓(𝑥) = 𝑥 2 − 2𝑥 + 8 and 𝑔(𝑥) = √𝑥 + 6, evaluate each of the following.
a. 𝑓(3) and 𝑔(3)
b. 𝑓(−10) and 𝑔(−10)
c. 𝑓(0)
d. 𝑓(𝑡)
e. 𝑓(𝑡 + 1) and 𝑓(𝑥 + 1)
f. 𝑓(𝑥 3 )
g. 𝑔(𝑥 2 − 5)

Solutions:
(a) 𝑓(3) and 𝑔(3)
𝑓(3) = (3)2 − 2(3) + 8 = 9 − 6 + 8
𝑓 (3) = 11
𝑔(3) = √3 + 6 = √9
𝑔(3) = 3 (we take only the positive square root)
(b) 𝑓(−10) and 𝑔(−10)
𝑓(−10) = (−10)2 − 2(−10) + 8 = 100 + 20 + 8
𝑓(−10) = 128
𝑔(−10) = √−10 + 6
𝑔(−10) = √−4, thus 𝑔(−10) is undefined since √−4 is imaginary.
(c) 𝑓(0)
𝑓(0) = (0)2 − 2(0) + 8
𝑓(0) = 8 Note that in this case this is pretty
(d) 𝑓(𝑡) much the same thing as our
𝑓(𝑡) = 𝑡 2 − 2𝑡 + 8 original function, except this time
we’re using 𝑡 as a variable.
(e) 𝑓(𝑥 + 1)
𝑓(𝑥 + 1) = (𝑥 + 1)2 − 2(𝑥 + 1) + 8 = 𝑥 2 + 2𝑥 + 1 − 2𝑥 − 2 + 8
𝑓(𝑥 + 1) = 𝑥 2 + 7
(f) 𝑓(𝑥 3 )
𝑓(𝑥 3 ) = (𝑥 3 )2 − 2(𝑥 3 ) + 8
𝑓(𝑥 3 ) = 𝑥 6 − 2𝑥 3 + 8

Property of and for the exclusive use of SLU. Reproduction, storing in a retrieval system, distributing, uploading or posting online, or transmitting in any form or by any
means, electronic, mechanical, photocopying, recording, or otherwise of any part of this document, without the prior written permission of SLU, is strictly prohibited. 37
(g) 𝑔(𝑥 2 − 5)
𝑔(𝑥 2 − 5) = √𝑥 2 − 5 + 6
𝑔(𝑥 2 − 5) = √𝑥 2 + 1

Piecewise Functions
We’ve seen an example of a piecewise function even if we didn’t call it a function
(or a piecewise function) at the time. Recall the mathematical definition of absolute value.
x if x  0
x 
 x if x  0
This is a function and if we use function notation we can write it as follows,

This is also an example of a piecewise function. A piecewise function is nothing more


than a function that is broken into pieces and which piece you use depends upon value of
𝑥.

Example 31: Given

Evaluate each of the following.


1. 𝑔(−6)
2. 𝑔(−4)
3. 𝑔(1)
4. 𝑔(15)
5. 𝑔(21)
Solution:
Now, to evaluate each of these functions values, the first thing that we need to do is
determine which inequality the 𝑡 value satisfies, and it will only satisfy a single inequality.
When we determine which inequality the number satisfies, we use the expression associated
with that inequality.
(a) 𝑔(−6)
In this case, −6 satisfies the first inequality, so we’ll use the first expression for this
evaluation.
𝑔(−6) = 3(−6)2 + 4 = 112
(b) 𝑔(−4)
𝑔(−4) = 3(−4)2 + 4 = 52
(c) 𝑔(1)
𝑔(1) = 10

Property of and for the exclusive use of SLU. Reproduction, storing in a retrieval system, distributing, uploading or posting online, or transmitting in any form or by any
means, electronic, mechanical, photocopying, recording, or otherwise of any part of this document, without the prior written permission of SLU, is strictly prohibited. 38
(d) 𝑔(15)
𝑔(15) = 10
(e) 𝑔(21)
𝑔(21) = 1 − 6(21) = −125

Domain and Range


The domain is the set of all 𝑥 values that we can plug into a function and get back a
real number. At this point, that means that we need to avoid division by zero and taking
square roots of negative numbers.

Example 32: Determine the domain of each of the following functions.


x3
a  g x  
x  3 x  10
2

b  f x   5  3x
7x  8
c  hx  
x2  4

d  Rx   102 x  5
x  16
Solution:
The domains for these functions are all the values of 𝑥 for which we don’t
divide by zero or the square root of a negative number. If we remember
these two ideas finding the domains will be pretty easy.
x3
a  g x  
x  3 x  10
2

There is a possibility that we’ll have a division by zero here. We now solve for
the zeros of the denominator (values that will make 𝑥 2 + 3𝑥 − 10 zero).
𝑥 2 + 3𝑥 − 10 = (𝑥 + 5)(𝑥 − 2) = 0
𝑥 = −5, 𝑥 = 2
So, we will get division by zero if we plug in 𝑥 = −5 or 𝑥 = 2. That means that
we’ll need to avoid those two numbers. However, all the other values of 𝑥will work
since they don’t give division by zero. Thus,
Domain : All real numbers except 𝑥 = −5 and 𝑥 = 2. We can also use
𝑅 − {−5,2} or 𝑅\{−5,2}.
b  f x   5  3x
We have a square root in the problem, so we’ll need to worry about taking the
square root of a negative number.
This one is going to work a little differently from the previous example. In that
part, we determined the value(s) of 𝑥 to avoid. In this case, it will be just as easy to get
the domain directly. To avoid square roots of negative numbers, all that we need to
do is require that

Property of and for the exclusive use of SLU. Reproduction, storing in a retrieval system, distributing, uploading or posting online, or transmitting in any form or by any
means, electronic, mechanical, photocopying, recording, or otherwise of any part of this document, without the prior written permission of SLU, is strictly prohibited. 39
5 − 3𝑥 ≥ 0
This is a fairly simple linear inequality that we should be able to solve at this
point.
5
5  3x  x
3
5
The domain of this function is : Domain : x 
3
7x  8
c  hx  
x2  4
In this case, we’ve got a fraction, but notice that the denominator will never
be zero for any real number since 𝑥 2 is guaranteed to be positive or zero and adding
4 onto this will mean that the denominator is always at least 4. In other words, the
denominator won’t ever be zero. So, all we need to do then is worry about the square
root in the numerator. To do this we’ll require
8 8
7x  8  0  7 x  8  x Domain : x  
7 7
10 x  5
d  Rx  
x 2  16
We’ve got both a square root and division by zero to worry about in this final
part. Let’s take care of the square root first since this will probably put the largest
restriction on the values of 𝑥. So, to keep the square root happy (i.e. no square root of
negative numbers) we’ll need to require that,
1
10 x  5  0  10 x  5  x
2
1
So, at the least we’ll need to require that x  in order to avoid problems with
2
the square root. Now, let’s see if we have any division by zero problems. Again, to do
this simply set the denominator equal to zero and solve.
𝑥 2 − 16 = (𝑥 − 4)(𝑥 + 4) = 0 ⇒ 𝑥 = −4, 𝑥 = 4
Now, notice that 𝑥 = −4 doesn’t satisfy the inequality we need for the square
root and so that value of 𝑥 has already been excluded by the square root. On the
other hand, 𝑥 = 4 does satisfy the inequality. This means that it is okay to plug 𝑥 = 4
into the square root, however, since it would give division by zero we will need to
avoid it. Thus,
1
Domain : x  except x  4
2

Property of and for the exclusive use of SLU. Reproduction, storing in a retrieval system, distributing, uploading or posting online, or transmitting in any form or by any
means, electronic, mechanical, photocopying, recording, or otherwise of any part of this document, without the prior written permission of SLU, is strictly prohibited. 40
Operations on Functions
Functions can be added, subtracted, multiplied, and divided. If 𝑓 (𝑥) and 𝑔(𝑥) are
two functions, then for all 𝑥 in the domain of both functions, the sum, difference, product,
and quotient are defined as follows.
1. (𝑓 + 𝑔)(𝑥) = 𝑓(𝑥) + 𝑔(𝑥)
2. (𝑓 − 𝑔)(𝑥) = 𝑓(𝑥) − 𝑔(𝑥)
3. (𝑓𝑔)(𝑥) = 𝑓(𝑥) ∙ 𝑔(𝑥)
𝑓 𝑓(𝑥)
4. (𝑔) (𝑥) = 𝑔(𝑥) , 𝑔(𝑥) ≠ 0

Example 33: Consider the functions 𝑓 (𝑥) = 2𝑥 + 3 and 𝑔(𝑥) = 𝑥 2 − 1. Find the following
function values:
a. (𝑓 + 𝑔)(4)
b. (𝑓 − 𝑔)(5)
Solution:
a. (𝑓 + 𝑔)(4)
Consider 𝑓(𝑥) = 2𝑥 + 3 then 𝑓(4) = 2(4) + 3 = 11
Also, 𝑔(𝑥) = 𝑥 2 – 1, then 𝑔(4) = 42 − 1 = 15
Hence,
(𝑓 + 𝑔)(4) = 𝑓 (4) + 𝑔(4) = 26

b. (𝑓 − 𝑔)(5)
Consider 𝑓(𝑥) = 2𝑥 + 3 then 𝑓(5) = 2(5) + 3 = 13
Also,𝑔(𝑥) = 𝑥 2 – 1, then 𝑔(5) = 52 − 1 = 24
Hence,
(𝑓 − 𝑔)(5) = 𝑓 (5) − 𝑔(5) = −11

Example 34: Find (𝑓𝑔)(𝑥) and (𝑓 /𝑔)(𝑥)


a. (𝑓𝑔)(𝑥)
b. (𝑓 /𝑔)(𝑥)
Solution:
a. (𝑓𝑔)(𝑥)
(𝑓𝑔)(𝑥) = 𝑓(𝑥) ∙ 𝑔(𝑥)
= (2𝑥 + 3)(𝑥 2 − 1)
= 2𝑥 3 + 3𝑥 2 − 2𝑥 − 3

b. (𝑓 /𝑔)(𝑥)
𝑓 𝑓(𝑥) 2𝑥 + 3
( ) (𝑥) = = 𝑥 2− 1
𝑔 𝑔(𝑥)
𝑓 2𝑥 + 3
Note that the function (𝑔) (𝑥) = 𝑥 2− 1
is defined on all values of 𝑥 except when
𝑥 = −1 and 𝑥 = 1.

Property of and for the exclusive use of SLU. Reproduction, storing in a retrieval system, distributing, uploading or posting online, or transmitting in any form or by any
means, electronic, mechanical, photocopying, recording, or otherwise of any part of this document, without the prior written permission of SLU, is strictly prohibited. 41
Composition of Functions
The term "composition of functions" (or "composite function") refers to the combining
of two or more functions in a manner where the output from one function becomes the input
for the next function. Formally we define the composition of functions as follows:
Consider functions 𝑓: 𝐴 → 𝐵 and 𝑔: 𝐵 → 𝐶, where the target set 𝐵 of 𝑓 is the domain of
𝑔. Let 𝑎 ∈ 𝐴. Then the image 𝑓(𝑎) under 𝑓 is in B which is the domain of 𝑔. Accordingly, we
can find the image of 𝑓(𝑎) under the function 𝑔, i.e. 𝑔(𝑓 (𝑎)). We have a rule which assigns
to each element 𝑎 ∈ 𝐴, an element 𝑔(𝑓(𝑎)) in 𝐶. This gives rise to a well-defined function from
𝐴 to 𝐶, which is called the composition of 𝒇 and 𝒈, denoted by 𝑔 ∘ 𝑓. That is, if 𝑓: 𝐴 → 𝐵 and
𝑔: 𝐵 ⟶ 𝐶, then we have
𝑔 ∘ 𝑓 ∶ 𝐴 ⟶ 𝐶 by (𝑔 ∘ 𝑓 )(𝑎) = 𝑔(𝑓 (𝑎)) for 𝑎 ∈ 𝐴
We illustrate the composition of functions 𝑓 and 𝑔 as follows

Example 35: Consider the following illustration:

Find (𝑔 ∘ 𝑓)(𝑎), (𝑔 ∘ 𝑓)(𝑏), (𝑔 ∘ 𝑓)(𝑐)


Solution:
Using the illustration above, we have
 (𝑔 ∘ 𝑓)(𝑎) = 𝑔(𝑓(𝑎)) = 𝑔(𝑦) = 𝑡
 (𝑔 ∘ 𝑓)(𝑏) = 𝑔(𝑓 (𝑏)) = 𝑔(𝑧) = 𝑟
 (𝑔 ∘ 𝑓)(𝑐) = 𝑔(𝑓(𝑐)) = 𝑔(𝑦) = 𝑡

Example 36: Let the functions 𝑓 and 𝑔 be defined by 𝑓 (𝑥) = 2𝑥 + 1 and 𝑔(𝑥) = 𝑥 2 − 2.
Find (𝑔 ∘ 𝑓)(𝑥) and (𝑓 ∘ 𝑔)(𝑥).
Solution:
a. (𝑔 ∘ 𝑓)(𝑥)
(𝑔 ∘ 𝑓)(𝑥) = 𝑔(𝑓 (𝑥))
Substituting the value of 𝑓 (𝑥) = 2𝑥 + 1, we obtain
𝑔(𝑓(𝑥)) = 𝑔(2𝑥 + 1) = (2𝑥 + 1)2 − 2 = 4𝑥 2 + 4𝑥 − 1
b. (𝑓 ∘ 𝑔)(𝑥)
(𝑓 ∘ 𝑔)(𝑥) = 𝑓(𝑔(𝑥))
Substituting the value of 𝑔(𝑥) = 𝑥 2 − 2, we obtain
𝑓(𝑔(𝑥)) = 𝑓 (𝑥 2 − 2) = 2(𝑥 2 − 2) + 1 = 2𝑥 2 − 3

Property of and for the exclusive use of SLU. Reproduction, storing in a retrieval system, distributing, uploading or posting online, or transmitting in any form or by any
means, electronic, mechanical, photocopying, recording, or otherwise of any part of this document, without the prior written permission of SLU, is strictly prohibited. 42
Inverse of a Function

The inverse of the function 𝑓 is denoted by 𝑓 −1 and is pronounced "f inverse". The
inverse of a function does not mean the reciprocal of a function.
A function normally tells you what 𝑦 is if you know what 𝑥 is. The inverse of a function
will tell you what 𝑥 had to be to get that value of 𝑦.
An inverse of a function is a function that "reverses" another function, that is, if the
function 𝑓 applied to an input 𝑥 gives a result of 𝑦, then applying its inverse function 𝑓 −1 to 𝑦
gives the result 𝑥, and vice versa. That is,
𝒇(𝒙) = 𝒚 if and only if 𝒇−𝟏 (𝒚) = 𝒙
We say that a function 𝒇−𝟏 is the inverse of 𝒇 if
 for every 𝑥 in the domain of 𝑓, (𝑓 −1 ∘ 𝑓)(𝑥) = 𝑥, and
 for every 𝑥 in the domain of 𝑓 −1 , (𝑓 ∘ 𝑓 −1 )(𝑥) = 𝑥
The domain of 𝑓 is the range of 𝑓 −1 and the range of 𝑓 is the domain of 𝑓 −1

Given the function 𝑓 (𝑥), we can find the inverse function, 𝑓 −1 (𝑥) by following these steps:
1. First, replace 𝑓 (𝑥) with 𝑦.
2. Replace every 𝑥 with a 𝑦 and replace 𝑦 with 𝑥.
3. Solve the equation from Step 2 for 𝑦 and replace 𝑦 with 𝑓 −1 (𝑥)
4. Verify your work by checking that (𝑓 −1 ∘ 𝑓)(𝑥) = 𝑥, and (𝑓 ∘ 𝑓 −1 )(𝑥) = 𝑥

Example 37: Find the inverse of the following functions:


1. 𝑓 (𝑥) = 3𝑥 − 2
2. 𝑔(𝑥) = √𝑥 − 3
𝑥+4
3. ℎ(𝑥) = 2𝑥−5
Solution:
1. 𝑓 (𝑥) = 3𝑥 − 2
𝑓 (𝑥) = 3𝑥 − 2
𝑦 = 3𝑥 − 2 Replace 𝑓 (𝑥) with 𝑦
𝑥 = 3𝑦 − 2 Replace every 𝑥 with a 𝑦 and replace every 𝑦 with an 𝑥
𝑥+2
3
=𝑦 Solve the equation from Step 2 for 𝑦
𝒙+𝟐
Hence, 𝒇−𝟏 (𝒙) = 𝟑 . To check, let us recall the composition of functions and show
that (𝑓 −1 ∘ 𝑓)(𝑥) = 𝑥, and (𝑓 ∘ 𝑓 −1 )(𝑥) = 𝑥:
3𝑥−2+2
 (𝑓 −1 ∘ 𝑓)(𝑥) = 𝑓 −1 (𝑓(𝑥)) = 𝑓 −1 (3𝑥 − 2) = 3 = 𝑥
𝑥+2 𝑥+2
 (𝑓 ∘ 𝑓 −1 )(𝑥) = 𝑓(𝑓 −1 (𝑥)) = 𝑓 (
3
) = 3(
3
)−2 = 𝑥

Property of and for the exclusive use of SLU. Reproduction, storing in a retrieval system, distributing, uploading or posting online, or transmitting in any form or by any
means, electronic, mechanical, photocopying, recording, or otherwise of any part of this document, without the prior written permission of SLU, is strictly prohibited. 43
2. 𝑔(𝑥) = √𝑥 − 3
𝑔(𝑥) = √𝑥 − 3
𝑦 = √𝑥 − 3 Replace 𝑔(𝑥) with 𝑦
𝑥 = √𝑦 − 3 Replace every 𝑥 with a 𝑦 and replace every 𝑦 with an 𝑥
2
𝑥 +3=𝑦 Solve the equation from Step 2 for 𝑦
Hence, 𝒈−𝟏 (𝒙) = 𝒙𝟐 + 𝟑. You may also check (𝑔−1 ∘ 𝑔)(𝑥) = 𝑥, and (𝑔 ∘ 𝑔−1 )(𝑥) = 𝑥

𝑥+4
3. ℎ (𝑥) =
2𝑥−5

𝑥+4
ℎ(𝑥) = 2𝑥−5
𝑥+4
𝑦= Replace ℎ (𝑥) with 𝑦
2𝑥−5

𝑦+4
𝑥 = 2𝑦−5 Replace every 𝑥 with a 𝑦 and replace every 𝑦 with an 𝑥
Solve the equation from Step 2 for 𝑦
𝑦+4
𝑥 = 2𝑦−5
2𝑥𝑦 − 5𝑥 = 𝑦 + 4 Multiply both sides by 2𝑦 − 5
−5𝑥 − 4 = −2𝑥𝑦 + 𝑦 Combine all terms containing 𝑦
−5𝑥 − 4 = 𝑦(−2𝑥 + 1) Factor out 𝑦
−5𝑥−4 5𝑥+4
= 𝑦 or =𝑦
−2𝑥+1 2𝑥−1
𝟓𝒙+𝟒
Hence, 𝒉−𝟏 (𝒙) = . You may also check (ℎ −1 ∘ ℎ )(𝑥) = 𝑥, and (ℎ ∘ ℎ −1 )(𝑥) = 𝑥
𝟐𝒙−𝟏

Learning Reinforcement 3

Directions: Write your solutions and answers on a clean sheet of


paper, or you may print this page and answer there. Submit the
image of your HANDWRITTEN SOLUTIONS as a single pdf file in the
submission bin for this activity in the Classroom. You may use
image scanning apps on your phone (CamScanner or Tap
Scanner) to save several images into one pdf file, or place your
images in a document and save them as a pdf file.

1. Specify the set 𝐴 by listing its elements, where


𝐴 = {𝑥: 𝑥 𝑖𝑠 𝑎 𝑤ℎ𝑜𝑙𝑒 𝑛𝑢𝑚𝑏𝑒𝑟 𝑙𝑒𝑠𝑠 𝑡ℎ𝑎𝑛 100 𝑎𝑛𝑑 𝑑𝑖𝑣𝑖𝑠𝑖𝑏𝑙𝑒 𝑏𝑦 16}.
2. Specify the set 𝐵 in set-builder form by giving a written description of its elements,
where 𝐵 = {0, 1, 4, 9, 16, 25}.
3. Consider
𝐴 = {𝑚, 𝑎, 𝑡, ℎ} 𝐶 = {𝑥: 𝑥 = 3𝑛, 1 ≤ 𝑛 ≤ 4, 𝑛 ∈ 𝑁}
𝐵 = {𝑠, 𝑡, 𝑒, 𝑚} 𝐷 = {𝑥: 𝑥 = 2𝑛, 1 ≤ 𝑛 ≤ 6, 𝑛 ∈ 𝑁}
a. What is 𝐴 ∩ 𝐵? b. What is 𝐶 ∪ 𝐷?

Property of and for the exclusive use of SLU. Reproduction, storing in a retrieval system, distributing, uploading or posting online, or transmitting in any form or by any
means, electronic, mechanical, photocopying, recording, or otherwise of any part of this document, without the prior written permission of SLU, is strictly prohibited. 44
5. Solve the following problem using a Venn diagram: Consider the following data
among 110 students in the college dormitory: 30 students are on a list A (taking
Accounting); 35 students are on a list B (taking Biology); and 20 students are on both
lists. Find the number of students:
a. on list A or B
b. on exactly one of the two lists
c. on neither list
5
6. Let 𝑔(𝑥) = . Find the domain of 𝑔(𝑥).
𝑥−4
2𝑥 2−3𝑥+7
7. Let 𝑓(𝑥) = 5𝑥 2−2𝑥
. Find 𝑓(3).
8. Suppose 𝑓 (𝑥) = √𝑥 − 3 and 𝑔(𝑥) = 𝑥 2 + 5. Which of the following is a formula for
(𝑓 ∘ 𝑔)(𝑥)?
9. Suppose 𝑓 (𝑥) = 𝑥 2 + 3 and 𝑔(𝑥) = 𝑥 − 2. Which of the following is a formula for
(𝑓 − 𝑔)(𝑥)?
10. Using the figure on the right and 𝑈 = {𝑎, 𝑏, 𝑐, 𝑑, 𝑒, 𝑓, 𝑔, ℎ} being the universal set, find the
elements of the set (𝑋 ∩ 𝑌) ∪ 𝑍.

11. consider the following sets:


𝑈 = {−5, −4, −3, −2, −1,0,1,2,3,4,5},
𝐴 = {𝑥: 𝑥 2 = 25},
𝐵 = {−5, 0,1,2,3,4, 5}
𝐶 = {𝑥: 𝑥 2 − 5𝑥 + 6 = 0}.
a. What is 𝐴 ∩ 𝐵?
b. Which is equal to 𝐵\𝐶?
c. Which set is equal to (𝐵\𝐴)𝑐 ?
d. What is the cardinality of a countably infinite set?

Well done! It’s time to answer Quiz 1.

Property of and for the exclusive use of SLU. Reproduction, storing in a retrieval system, distributing, uploading or posting online, or transmitting in any form or by any
means, electronic, mechanical, photocopying, recording, or otherwise of any part of this document, without the prior written permission of SLU, is strictly prohibited. 45
MODULE 4: REASONING AND PROBLEM SOLVING

Our target learning outcomes for this module are: a) solve problems involving patterns
and recreational puzzles using inductive and deductive reasoning; b) use different types of
reasoning to justify statements and arguments made about mathematics and mathematical
concepts; c) solve problems involving patterns and recreational problems following Polya’s
four steps; d) organize one’s methods and approaches for proving and solving problems.

Solve the VIRAL MATH PROBLEM − 6 ÷ 2(1 + 2) =


What is your answer to this problem? Why? Justify your answer.

UNIT 1: REASONING

A. Inductive and Deductive Reasoning

Inductive reasoning.
It is the process of forming or reaching a conclusion by examining specific examples.
Note that the conclusion that is formed by using inductive reasoning is often called a
conjecture, since it may or may not be correct.

Example 38: Inductive reasoning to predict a number. Given the sequence 1, 3, 6, 10, 15,
__. What is the next number?
Solution
The difference between the first two numbers is 2. The second and third numbers differ
by 3; the third and fourth numbers differ by 4. It appears that the difference between any
two consecutive numbers is always one greater than the preceding difference. Since 10 and
15 differ by 5, we predict that the next number in the list will be 6 greater than 15. Hence, the
next number is 21.

Example 39: Inductive reasoning to make a conjecture: Consider the following procedure:
Pick a number. Multiply the number by 8; add 6 to the product; divide the sum by 2; and,
subtract 3 from the quotient.
Solution
Using any number (an integer, i.e.) and following the outlined procedure or steps, we
conjecture that the given procedure will produce a resulting number that is four times the
original number.

Counterexamples. It is one case found for which a statement is not true. It is used to
verify if a statement is a false statement.

Property of and for the exclusive use of SLU. Reproduction, storing in a retrieval system, distributing, uploading or posting online, or transmitting in any form or by any
means, electronic, mechanical, photocopying, recording, or otherwise of any part of this document, without the prior written permission of SLU, is strictly prohibited. 46
Example 40: Statement: For all x, x2 > x.
Solution
Consider x = 1. For x = 1, we have 1 2 = 1. Since 1 is not greater than 1, a
counterexample is found. Thus, the statement “for all x, x2> x” is a false statement.

Example 41: Statement: For all x, x/x = 1.


Solution
Consider x = 0. For x = 0, 0/0 ≠ 1; 0/0 is undefined. Hence, the statement “for all x,
x/x = 1” is a false statement.

Deductive reasoning
It is the process of reaching a conclusion by applying general assumptions,
procedures, or principles.

Example 42: Use deductive reasoning to show that the following procedure produces a
number that is four times the original number.
Procedure: Pick a number. Multiply the number by 8, add 6 to the product, divide the sum
by 2, and subtract 3 from the quotient.
Solution
Let n represent the original number.
Multiply the number by 8: 8n
Add 6 to the product: 8n + 6

Divide the sum by 2:


Subtract 3 from the quotient: 4n + 3 – 3 = 4n

Starting with n and performing the procedure, we ended with 4n. The
procedure produces a number that is four times the original number.

Remember! Inductive: from specific to general.


Deductive: from general to specific.

B. Logic and KenKen Puzzles


Video viewing and doing problem solving:
Can you solve Einstein’s Riddle? https://www.youtube.com/watch?v=1rDVz_Fb6HQ

Logic puzzles
Logic puzzles can be solved by using deductive reasoning and a chart that is used to
display given information in a visual manner.

Property of and for the exclusive use of SLU. Reproduction, storing in a retrieval system, distributing, uploading or posting online, or transmitting in any form or by any
means, electronic, mechanical, photocopying, recording, or otherwise of any part of this document, without the prior written permission of SLU, is strictly prohibited. 47
Example 43: Each of four neighbors, Sean, Maria, Sarah, and Brian, has a different
occupation (editor, banker, chef, or dentist). From the following clues, determine the
occupation of each neighbor.
Solution
Clues:
1. Maria gets home from work after the banker and the dentist.
2. Sarah, who is the last to get home from work, is not the editor.
3. The dentist and Sarah leave for work at the same time.
4. The banker leaves next door to Brian.

Procedure:
1. From clue 1, Maria is not the banker nor the dentist. In the following chart, we write ×1
under the Banker and Dentist column for Maria, to indicate the possibility is eliminated
because of clue 1.
Editor Banker Chef Dentist
Sean
Maria ×1 ×1
Sarah
Brian
2. From clue 2, Sarah is not the editor. Also, Sarah is the last to get home from work and from
clue 1, the banker is not the last to get home from work. Hence, Sarah is also not the
banker. We write ×2 under the editor and banker column for Sarah.
Editor Banker Chef Dentist
Sean
Maria ×1 ×1
Sarah ×2 ×2
Brian
3. From clue 3, Sarah is not the dentist for she and the dentist leave for work at the same
time. We write ×3 under the Dentist column for Sarah in the following chart and we
conclude that Sarah is the Chef. Correspondingly, we get to see that since Sarah is the
Chef, we place an ×3 mark under the Chef column for Maria and conclude that she is
the Editor. Now, we also place an ×3 mark under the columns Chef and Editor for Sean
and Brian.
Editor Banker Chef Dentist
Sean ×3 ×3
Maria ×1 ×3 ×1
Sarah ×2 ×2 ×3
Brian ×3 ×3

Property of and for the exclusive use of SLU. Reproduction, storing in a retrieval system, distributing, uploading or posting online, or transmitting in any form or by any
means, electronic, mechanical, photocopying, recording, or otherwise of any part of this document, without the prior written permission of SLU, is strictly prohibited. 48
4. From clue 4, Brian is not the banker. We place an ×4 mark under the Banker column for
Brian. Since there are three × ’𝑠 under the Banker column, then Sean must be the Banker.
Sean could not be the Dentist so we place an ×4 mark for Sean under the Dentist column.
Since there are three × ’𝑠 under the Dentist column, Brian must be the Dentist and we
place a check mark in that box.
Editor Banker Chef Dentist
Sean ×3 ×3 ×4
Maria ×1 ×3 ×1
Sarah ×2 ×2 ×3
Brian ×3 ×4 ×3
From the given clues (and using deductive reasoning), Sean is the banker, Maria is
the editor, Sarah is the Chef, and Brian is the dentist.

Kenken puzzles
Watch the video: KenKen 4x4 Introduction and Strategy@
https://www.youtube.com/watch?v=uYTrrsxJqb0

KenKen is an arithmetic-based logic puzzle that was invented by the Japanese


mathematics teacher Tetsuya Miyamoto in 2004. The noun "ken" has "knowledge" and
"awareness" as synonyms. Hence, KenKen translates as knowledge squared, or awareness
squared.

Rules for Solving KenKen Puzzles


1. For an n x n puzzle, fill in each square of the grid with one of the numbers 1, 2, 3, ...
,n. Thus, in a 3x3 puzzle, use the numbers 1 to 3. In a 4x4 puzzle, use the numbers 1
to 4. In a 5x5 puzzle, use the numbers 1 to 5, and so on.
2. Do not repeat a number in any row or column. Our goal is to fill in the whole grid
with numbers, making sure no number is repeated in any row or column.
3. The numbers in each heavily outlined set of squares, called cages, must combine
(in some order) to produce the target number in the top left corner of the cage
using the mathematical operation indicated.
4. Cages with just one square should be filled in with the target number. If you see a
single-cell cage with just a number and no operator, it means that the value in
that cell is the target number. Such single-cell cages work like in Sudoku puzzles.
You won’t see these in every puzzle, but when you see one, you should start there.
5. A number can be repeated within a cage as long as it is not in the same row or
column.

Property of and for the exclusive use of SLU. Reproduction, storing in a retrieval system, distributing, uploading or posting online, or transmitting in any form or by any
means, electronic, mechanical, photocopying, recording, or otherwise of any part of this document, without the prior written permission of SLU, is strictly prohibited. 49
KenKen Puzzles Example

Figure a Figure b
First fill in single box cages, called "freebies" Write the possible numbers for each box
with the target number. considering the cage and operations.

Property of and for the exclusive use of SLU. Reproduction, storing in a retrieval system, distributing, uploading or posting online, or transmitting in any form or by any
means, electronic, mechanical, photocopying, recording, or otherwise of any part of this document, without the prior written permission of SLU, is strictly prohibited. 50
Figure c. Eliminate numbers 2 and 1 Figure d. From Figure c it shows that the circled
that are with the same column and numbers 4 found their location.
row of the freebies 2 and 1.

Figure e. Eliminate # 4 that are with Figure f:


the same column and row of the result
in Figure d.

Property of and for the exclusive use of SLU. Reproduction, storing in a retrieval system, distributing, uploading or posting online, or transmitting in any form or by any
means, electronic, mechanical, photocopying, recording, or otherwise of any part of this document, without the prior written permission of SLU, is strictly prohibited. 51
Figure g: Eliminate the numbers that are with
the same column and row of the result in Figure f.

The final result then will be:

C: LOGIC

When someone tells you, “I fail to see your logic,” perhaps the person you are talking
to does not understand your reasoning or sees your arguments as invalid. How exactly do
we determine whether our statements are valid or not? This is the focus of our next section.
Merriam Webster defines logic as a science that deals with the principles and criteria
of validity of inference and demonstration: the science of the formal principles of reasoning.
It allows us to analyze arguments and determine whether it is valid or invalid. Logic is used
by lawyers and judges, computer programmers and more. It is an important tool in

Property of and for the exclusive use of SLU. Reproduction, storing in a retrieval system, distributing, uploading or posting online, or transmitting in any form or by any
means, electronic, mechanical, photocopying, recording, or otherwise of any part of this document, without the prior written permission of SLU, is strictly prohibited. 52
mathematical proofs. In this context, we will see that logic includes a language for
expressing complicated compound statements, a concise notation for writing them, and a
methodology for objectively reasoning about their validity. Gottfried Wilhelm Leibniz (1646-
1716), Augustus De Morgan (1806-1871) and George Boole (1815-1864) are some of the first
mathematicians to study symbolic logic as we know it today, advancing it as a
mathematical discipline instead of merely being a part of philosophy. In particular, Boole
published two works on logic, The Mathematical Analysis of Logic and An Investigation of
the Laws of Thought.

LOGIC STATEMENTS: SIMPLE AND COMPOUND

In logic, a statement is a declarative sentence that is either true or false, but not both
true and false. Let us take a look at the following sentences, and identify if it is considered as
a statement:
1. Benguet is a province in the Philippines.
2. How are you?
3. 99 + 2 is a prime number.
4. 𝑥 2 − 4𝑥 = 12.
5. Open the door.
6. 7055 is a large number.
7. In the year 2022, the president of the Philippines will be a woman.
8. 𝑥 > 3.

In the examples above, numbers 1, 3, 4, 7, and 8 are considered as statements, but


the others are not. Note that although we may not know if sentences 3 and 7 are true or
false, they cannot be true and false. Statements 4 and 8 are called open statements, and
their truth or falsity depends on a particular value of 𝑥. Examples 2 and 5 are not declarative
sentences, while the word “large” in example 6 is not a precise term, so we cannot
determine if indeed the sentence is true or false. The famous paradox “This is a false
statement” is not a statement (think about it!).
A simple statement is a statement that conveys a single idea, while a compound
statement conveys two or more ideas. Compound statements are formed by connecting
simple statements with words or phrases such as “and,” “or,” “if... then...” and “if and only if.”
These, together with “not,” are called logical connectives. George Boole used symbols such
as 𝑝, 𝑞, 𝑟, and 𝑠 to represent simple statements and the symbols ∧,∨, ∼, →, and ↔ to represent
connectives. Using these, we can write any logic statement in its symbolic form. The table
below summarizes the different statements, their types, connectives, and symbolic form.

Property of and for the exclusive use of SLU. Reproduction, storing in a retrieval system, distributing, uploading or posting online, or transmitting in any form or by any
means, electronic, mechanical, photocopying, recording, or otherwise of any part of this document, without the prior written permission of SLU, is strictly prohibited. 53
Statement Connective Symbolic Form Type of Statement
not 𝑝 not ∼𝑝 negation
𝑝 and 𝑞 and 𝑝∧𝑞 conjunction
𝑝 or 𝑞 or 𝑝∨𝑞 disjunction
If 𝑝, then 𝑞 If... then 𝑝→𝑞 conditional
𝑝 if and only if 𝑞 if and only if 𝑝↔𝑞 biconditional

Let us have some illustrations on writing statements in their symbolic form. Consider the
following simple statements and their symbols:

𝑝: Today is Friday.
𝑞: It is raining.
𝑟: I am going to a movie.
𝑠: I am not going to the basketball game.

Example 44: Here are some compound statements and their corresponding symbolic
form:
Solution
1. Today is Friday and it is raining.
SYMBOLIC FORM: 𝑝 ∧ 𝑞
2. It is not raining and I am going to a movie.
SYMBOLIC FORM: ∼ 𝑞 ∧ 𝑟
3. I am going to the basketball game or I am going to a movie.
SYMBOLIC FORM: ∼ 𝑠 ∨ 𝑟
4. If it is raining, then I am not going to the basketball game.
SYMBOLIC FORM: 𝑞 → 𝑠

Example 45: Let us also show illustrations on doing the reverse procedure, translating the
symbolic form into words. Suppose we have the following simple statements and their
symbols:
𝑝: The game will be played in Atlanta.
𝑞: The game will be shown on CBS.
𝑟: The game will not be shown on ESPN.
𝑠: The Dodgers are favored to win.
Solution
Then the following statements in the symbolic form will be translated in this manner:
1. 𝑞 ∧ 𝑝
IN WORDS: The game will be shown on CBS and will be played in Atlanta.
2. ∼ 𝑟 ∧ 𝑠
IN WORDS: The game will be shown on ESPN, and the Dodgers are favored to win.

Property of and for the exclusive use of SLU. Reproduction, storing in a retrieval system, distributing, uploading or posting online, or transmitting in any form or by any
means, electronic, mechanical, photocopying, recording, or otherwise of any part of this document, without the prior written permission of SLU, is strictly prohibited. 54
3. 𝑠 ↔∼ 𝑝
IN WORDS: The Dodgers are favored to win if and only if the game will not be played
in Atlanta.

In compound statements containing more than two simple statements, grouping


symbols are used to indicate which simple statements are grouped together. When in
symbols, parenthesis or brackets are used. If it is given in words, a comma is used: all
statements before the comma are grouped together, and all statements after the comma
are also grouped together. The table below gives particular examples of groupings in
symbols and in words.

Symbolic Form English Sentence


𝑝 ∧ (𝑞 ∨∼ 𝑟) 𝑝, and 𝑞 or not 𝑟
(𝑝 ∧ 𝑞) ∨ 𝑟 𝑝 and 𝑞, or 𝑟
(𝑝 ∧∼ 𝑞) → (𝑟 ∨ 𝑠) If 𝑝 and not 𝑞, then 𝑟 or 𝑠

Example 46: Suppose we are given the following simple statements:


𝑝: You get a promotion.
𝑞: You complete the training.
𝑟: You will receive a bonus.
Then the symbolic form (𝑝 ∧ 𝑞) → 𝑟 in words will be “If you get a promotion and complete
the training, then you will receive a bonus.” On the other hand, the statement “If you do
not complete the training, then you will not get a promotion and you will not receive a
bonus” will be represented by ~𝑞 → (~𝑝 ∧ ~𝑟).

TRUTH VALUES AND TRUTH TABLES

The truth value of a simple statement is either true (T) or false (F). The truth value of a
compound statement depends on the truth values of its simple statements and its
connectives. A truth table is a table that shows the truth value of a compound statement for
all possible truth values of its simple statements. We take note of the following for the
negation, conjunction, and disjunction:

1. The negation of a true statement is false, while the negation of a false statement is
true.
2. Both statements must be true for the conjunction to be true. If at least one statement
is false, then the conjunction will be false.
3. A disjunction is true if either statement is true. If both statements are false, then the
disjunction is false.

Property of and for the exclusive use of SLU. Reproduction, storing in a retrieval system, distributing, uploading or posting online, or transmitting in any form or by any
means, electronic, mechanical, photocopying, recording, or otherwise of any part of this document, without the prior written permission of SLU, is strictly prohibited. 55
These are summarized in the following truth tables:

Truth Table for a Truth Table for a Conjunction Truth Table for a Disjunction
Negation 𝑝 𝑞 𝑝∧𝑞 𝑝 𝑞 𝑝∨𝑞
𝑝 ∼𝑝 T T T T T T
T F T F F T F T
F T F T F F T T
F F F F F F

Example 47: Let us look at some examples of determining the truth value of a compound
statement:
1. 7 ≥ 5.
This statement is a disjunction: It can be written as 7 > 5 or 7 = 5. Since the first
statement is true, it means that disjunction 7 ≥ 5 is also true.
2. 5 is a whole number, and 5 is an even number.
The first simple statement is true, and the second is false, so the conjunction is false.
3. 2 is a prime number, and 2 is an even number.
Since both simple statements are true, the conjunction is true.
4. 21 is a rational number, and 21 is a natural number.
This conjunction is also true.
5. 4 ≤ 9.
This disjunction is true.
6. −7 ≥ −3.
This disjunction is false since −7 > −3 is false, and −7 = −3 is also false.

Construction of Truth Table


For combinations of simple arbitrary statements, constructing a truth table allows us
to determine the truth value of the compound statement given any combination of the truth
values of the simple statements. Note that if a compound statement has 𝑛 simple statements,
then the truth table to be constructed will have 2𝑛 rows so that all possibilities are covered.
We will show two ways to construct the truth table.

Example 48: Let us first consider the compound statement ∼ 𝑝 ∨ (𝑝 ∧ 𝑞).


Procedure
There are two simple statements, 𝑝 and 𝑞, so there must be 22 = 4 rows in the truth
table. We start off with the following standard truth table form. Notice that in the column of
the simple statement 𝑞, the truth values T and F alternate, while in the column for 𝑝, there are
two rows of T is followed by two rows of F. This ensures all combinations of truth values are
enumerated.

Property of and for the exclusive use of SLU. Reproduction, storing in a retrieval system, distributing, uploading or posting online, or transmitting in any form or by any
means, electronic, mechanical, photocopying, recording, or otherwise of any part of this document, without the prior written permission of SLU, is strictly prohibited. 56
𝑝 𝑞
T T
T F
F T
F F

In the first method of constructing truth tables, we add a column to the truth table for
each particular connective that appears in the symbolic form. In the example above, we
start with a column for ∼ 𝑝 and identify its truth value for each row. This is followed by a
column for 𝑝 ∧ 𝑞, and finally a column for ∼ 𝑝 ∨ (𝑝 ∧ 𝑞) whose truth values are based on the
previous two columns. This last column gives the truth values of the compound statement ∼
𝑝 ∨ (𝑝 ∧ 𝑞).
𝑝 𝑞 ∼𝑝 𝑝∧𝑞 ∼ 𝑝 ∨ (𝑝 ∧ 𝑞)
T T F T T
T F F F F
F T T F T
F F T F T
① ② ③

Based on column 3, we can say that if 𝑝 is false and 𝑞 is true, then


∼ 𝑝 ∨ (𝑝 ∧ 𝑞) will be true. Other conclusions can be made by referring to the remaining rows
of the truth table.
In the second method of constructing a truth table, instead of having a separate
column for each connective, we simply place the statement's truth value under the symbol
of the connective, as shown in the table below. Here we number the columns as we
evaluate the truth values under each connective and shade the final column to indicate
the truth values of the compound statement given. Notice that compared to the first
method, we have rearranged the columns in a truth table. In a way, this alternative
procedure results in a more compact solution, especially in compound statements that
involve many signs of grouping. Of course, you are free to use either method based on your
preference.
𝑝 𝑞 ∼ 𝑝 ∨ (𝑝 ∧ 𝑞)
T T F T T
T F F F F
F T T T F
F F T T F
① ③ ②

Property of and for the exclusive use of SLU. Reproduction, storing in a retrieval system, distributing, uploading or posting online, or transmitting in any form or by any
means, electronic, mechanical, photocopying, recording, or otherwise of any part of this document, without the prior written permission of SLU, is strictly prohibited. 57
Example 49: Let us consider the compound statement [𝑟 ∧ (∼ 𝑝 ∨ 𝑞)] ∧ (𝑟 ∨∼ 𝑞)
Procedure
This time, there will be eight rows in the standard truth table form. In the 𝑟 column, the
T and F alternate; in the 𝑞 column, T and F appear two at a time; and finally in the 𝑝 column,
four rows of T are followed by four rows of F. This ensures all combinations of truth values are
covered, and usually helps us remember how to fill up the rows! The table below gives us the
results:
𝑝 𝑞 𝑟 [𝑟 ∧ (∼ 𝑝 ∨ 𝑞)] ∧ (𝑟 ∨ ∼ 𝑞)
T T T T F T T T F
T T F F F T F F F
T F T F F F F T T
T F F F F F F T T
F T T T T T T T F
F T F F T T F F F
F F T T T T T T T
F F F F T T F T T
③ ① ② ⑥ ⑤ ④

When constructing truth tables, remember to arrange your simple statements


alphabetically and evaluate going out for nested signs of grouping from the inside.

Equivalent Statements
Truth tables can be used to show that compound statements are equivalent. Two
statements are said to be equivalent if they have the same truth value for all possible truth
values of their simple statements. The symbol denotes equivalent statements ≡.

Example 50: Verify that ∼ (𝑝 ∨∼ 𝑞) and ∼ 𝑝 ∧ 𝑞 are equivalent statements, that is, ∼
(𝑝 ∨∼ 𝑞) ≡ ∼ 𝑝 ∧ 𝑞.
Solution
𝑝 𝑞 ∼ (𝑝 ∨∼ 𝑞) ∼ 𝑝∧𝑞
𝑝
T T F F
T F F F
F T T T
F F F F
The example above is an application of De Morgan’s Laws for statements: For any
statements 𝑝 and 𝑞,
∼ (𝑝 ∧ 𝑞) ≡ ∼ 𝑝 ∨∼ 𝑞 and ∼ (𝑝 ∨ 𝑞) ≡ ∼ 𝑝 ∧∼ 𝑞.

Property of and for the exclusive use of SLU. Reproduction, storing in a retrieval system, distributing, uploading or posting online, or transmitting in any form or by any
means, electronic, mechanical, photocopying, recording, or otherwise of any part of this document, without the prior written permission of SLU, is strictly prohibited. 58
Another example of equivalent statements is ∼ (∼ 𝑝) ≡ 𝑝 since the negation of a
negation is the original statement. We will encounter more equivalent statements when we
discuss conditional and biconditional statements.

CONDITIONAL STATEMENTS

The conditional statement is a statement of the form “If 𝑝, then 𝑞” or in symbols, 𝑝 → 𝑞.


It shows an implication; that is, given that a situation 𝑝 will happen (antecedent of
hypothesis), then another situation 𝑞 will happen (consequent or hypothesis). Most theorems
in mathematics are in the form of a conditional. There are many other ways to express 𝑝 →
𝑞, namely:

If 𝑝, 𝑞. 𝑞 is necessary for 𝑝.
𝑝 implies 𝑞. 𝑞 is a necessary condition for 𝑝.
𝑞, if 𝑝. 𝑝 only if 𝑞.
𝑞 when 𝑝. 𝑞 whenever 𝑝.
𝑝 is sufficient for 𝑞. 𝑞 follows from 𝑝.
𝑝 is a sufficient condition for 𝑞.

A conditional statement can also be expressed in terms of negations, conjunctions


and disjunctions based on the following statements equivalent to 𝑝 → 𝑞:

𝑝 →𝑞 ≡∼ 𝑝∨𝑞 and 𝑝 → 𝑞 ≡ ∼ (𝑝 ∧∼ 𝑞)

Using these equivalent statements, we can see that the negation of 𝑝 → 𝑞 is 𝑝 ∧∼ 𝑞. Note that
the negation is no longer a conditional statement! To illustrate, consider the statement “If
they pay me money, I will sign the contract.” Its negation will be, “They pay (or paid) me
money and I will not sign the contract.” Note that “but” can also be used for a conjunction,
so the negation can also be written as “They pay (or paid) me money but I will not sign the
contract” to emphasize the contrast in the two simple statements.
The conditional is false if the antecedent is true and the consequent is false.
Otherwise, the conditional is true. This is shown in the truth table for 𝑝 → 𝑞:

𝑝 𝑞 𝑝→𝑞
T T T
T F F
F T T
F F T

Property of and for the exclusive use of SLU. Reproduction, storing in a retrieval system, distributing, uploading or posting online, or transmitting in any form or by any
means, electronic, mechanical, photocopying, recording, or otherwise of any part of this document, without the prior written permission of SLU, is strictly prohibited. 59
To explain this, let us consider a conditional statement that perhaps your
parents/guardians have said to you to convince you to do well in school: “If you pass this
test, then I will treat you to your favorite restaurant.” We analyze the truth value of this
conditional statement.
1. If you passed the test and you were treated to your favorite restaurant, then you can
say that your parents/guardians were truthful when they said the conditional. Thus 𝑝 →
𝑞 is true in the first row of the truth table.
2. If you passed the test and you were not treated to your favorite restaurant, then you
would say that parents/guardians were not truthful when they said the conditional.
Thus 𝑝 → 𝑞 is false in the second row of the truth table.
3. If you did not pass the test, then regardless of being treated to the restaurant or not,
you cannot say that the conditional is false. This is because it does not indicate what
might or might not happen if you did not pass the test. Thus, 𝑝 → 𝑞 is true in rows 3 and
4 of the truth table.

Construct the truth table for the following compound statements involving the
conditional for your practice: (1) [𝑝 ∧ (𝑝 → 𝑞)] → 𝑞; (2) [𝑝 ∧ (𝑞 ∨∼ 𝑝)] → ∼ 𝑝.

If we switch and/or negate the antecedent and consequent of a conditional


statement, we can form other conditionals such as the converse, inverse, and
contrapositive. Given a conditional 𝑝 → 𝑞, we have the following:
converse: 𝑞→𝑝
inverse: ∼𝑝→∼𝑞
contrapositive: ∼𝑞 →∼𝑝

Note that the conditional statement is equivalent to its contrapositive, while the converse is
equivalent to the inverse:

𝑝 → 𝑞 ≡∼ 𝑞 → ∼ 𝑝 and 𝑞 →𝑝 ≡∼ 𝑝→∼𝑞

Example 51: Let us consider the conditional statement “If I get the job, then I will rent the
apartment.” Its converse is “If I rent the apartment, then I will get the job.” The inverse will
be “If I don’t get the job, then I will not rent the apartment.” The contrapositive will be “If I
don’t rent the apartment, then I will not get the job.” In addition, the negation of the
conditional is “I get the job and I will not rent the apartment.”

Example 52: Practice on this: State the converse, inverse, contrapositive and negation of
the conditional statement “Whenever I do yoga, I feel calm.” Be careful with your choice
of antecedent and consequent! Refer to the different ways to express 𝑝 → 𝑞 previously
discussed.

Property of and for the exclusive use of SLU. Reproduction, storing in a retrieval system, distributing, uploading or posting online, or transmitting in any form or by any
means, electronic, mechanical, photocopying, recording, or otherwise of any part of this document, without the prior written permission of SLU, is strictly prohibited. 60
BICONDITIONAL STATEMENTS

A biconditional statement is a two-way conditional statement: 𝑝 if and only if 𝑞, or in


symbols, 𝑝 ↔ 𝑞, is the conjunction of two conditional statements:

𝑝 ↔ 𝑞 ≡ (𝑝 → 𝑞) ∧ (𝑞 → 𝑝)

Using this equivalence, the truth table of the biconditional can be constructed, as shown
below. Here, we can see that a biconditional statement is true if its simple statements have
the same truth value.
𝑝 𝑞 𝑝↔𝑞
T T T
T F F
F T F
F F T

Example 53: Let us consider some biconditional statements involving the variable 𝑥 and
determine its truth value.
1. 𝑥 + 4 = 7 if and only if 𝑥 = 3.
2. 𝑥 2 = 36 if and only if 𝑥 = 6.
3. 𝑥 > 7 if and only if 𝑥 > 6.
Solution:
Recall that it is necessary to substitute a value for 𝑥 in order to know the truth value of
the open statement. In this case, a biconditional is a true statement no matter what value
of 𝑥 is substituted. A biconditional becomes false if there is at least one value of 𝑥 that makes
it false.
1. 𝑥 + 4 = 7 if and only if 𝑥 = 3.
From the first equation, we know that 𝑥 = 3. Thus to determine the truth value of the
biconditional, we need to check two possibilities: (a) 𝑥 = 3; (b) 𝑥 ≠ 3.

𝑥+4= 7 𝑥=3 𝑥+4=7↔𝑥=3


Case (a): 𝑥 = 3 T T T
Case (b): 𝑥 ≠ 3 F F T

We can see that no matter what value is substituted into 𝑥, both equations have the
same truth value. Thus, the biconditional is true.

2. 𝑥 2 = 36 if and only if 𝑥 = 6.
Here, we only need to note that if we substitute 𝑥 = −6, the first equation holds but
the second does not. Thus, the biconditional statement is false.

Property of and for the exclusive use of SLU. Reproduction, storing in a retrieval system, distributing, uploading or posting online, or transmitting in any form or by any
means, electronic, mechanical, photocopying, recording, or otherwise of any part of this document, without the prior written permission of SLU, is strictly prohibited. 61
3. 𝑥 > 7 if and only if 𝑥 > 6.
What do you think?

TAUTOLOGIES AND SELF-CONTRADICTIONS

A tautology is a statement that is always true. A self-contradiction is a statement that


is always false.

Example 54: 𝑝 ∨ (∼ 𝑝 ∨ 𝑞) is a tautology and 𝑝 ∧ (∼ 𝑝 ∧ 𝑞) is a self-contradiction, as shown in


the truth table below.

𝑝 𝑞 𝑝 ∨ (∼ 𝑝 ∨ 𝑞) 𝑝 ∧ (∼ 𝑝 ∧ 𝑞)
T T T T F T T T F F F T
T F T T F F F T F F F F
F T F T T T T F F T T T
F F F T T T F F F T F F

Example 55: The open statement 𝑥 + 2 = 5 is neither a tautology nor a self-contradiction


since it is true when 𝑥 = 3 and false when 𝑥 ≠ 3.

SYMBOLIC ARGUMENTS

An argument consists of a set of statements called premises and another statement


called the conclusion. The conclusion is preceded by the word “therefore.” An argument is
valid if the conclusion is true whenever all the premises are assumed to be true. It is invalid if
it is not a valid argument. The symbolic form of an argument is written as

𝑃𝑟𝑒𝑚𝑖𝑠𝑒 1
𝑃𝑟𝑒𝑚𝑖𝑠𝑒 2

𝐿𝑎𝑠𝑡 𝑃𝑟𝑒𝑚𝑖𝑠𝑒
∴ 𝐶𝑜𝑛𝑐𝑙𝑢𝑠𝑖𝑜𝑛

The symbol ∴ represents the word “therefore.” An argument can also be written as a
conditional statement whose antecedent is the conjunction of all the premises and whose
consequent is the conclusion:

[𝑃𝑟𝑒𝑚𝑖𝑠𝑒 1 ∧ 𝑃𝑟𝑒𝑚𝑖𝑠𝑒 2 ∧ … ∧ 𝐿𝑎𝑠𝑡 𝑃𝑟𝑒𝑚𝑖𝑠𝑒] → 𝐶𝑜𝑛𝑐𝑙𝑢𝑠𝑖𝑜𝑛

Property of and for the exclusive use of SLU. Reproduction, storing in a retrieval system, distributing, uploading or posting online, or transmitting in any form or by any
means, electronic, mechanical, photocopying, recording, or otherwise of any part of this document, without the prior written permission of SLU, is strictly prohibited. 62
The validity of an argument can be determined using a truth table. We proceed as
follows:
1. Write the argument in symbolic form.
2. Construct a truth table that shows the truth value of each premise and the truth value
of the conclusion for all combinations of truth values of the simple statements.
3. If the conclusion is true in every row of the truth table in which all the premises are
true, the argument is valid. If the conclusion is false in any row in which all of the
premises are true, the argument is invalid

Example 56: Let us consider the following argument: “If Aristotle was human, then Aristotle
was mortal. Aristotle was human. Therefore, Aristotle was mortal.”
Solution:
1. Symbolic Form
If we use the symbol ℎ for the statement “Aristotle was human,” and 𝑚 for “Aristotle
was mortal,” then we obtain the following:
Symbolic form: Conditional:
ℎ→𝑚 [(ℎ → 𝑚 ) ∧ ℎ] → 𝑚

∴ 𝑚

2. Construct a Truth Table


First Premise Second Premise Conclusion
ℎ 𝑚 ℎ→𝑚 ℎ 𝑚
T T T T T
T F F T F
F T T F T
F F T F F

3. Validity of the Argument


We disregard rows 2, 3, and 4 in our analysis because the premises are not both true.
Since only the first row has both premises true, and the conclusion is also true, then the
argument is valid.

Property of and for the exclusive use of SLU. Reproduction, storing in a retrieval system, distributing, uploading or posting online, or transmitting in any form or by any
means, electronic, mechanical, photocopying, recording, or otherwise of any part of this document, without the prior written permission of SLU, is strictly prohibited. 63
Example 57: Consider the following argument: “If it rains, then the game will not be played.
It is not raining. Therefore, the game will be played.”
Solution:
Let 𝑟 represent the statement “It rains” and 𝑔 the statement “The game will be
played.”

1. Then the symbolic form and the truth table for the argument are given by
𝑟 →∼𝑔
∼𝑟
∴ 𝑔
2. Construct a Truth Table

First Premise Second Premise Conclusion


𝑔 𝑟 𝑟 →∼𝑔 ∼𝑟 𝑔
T T F F T
T F T T T
F T T F F
F F T T F

3. Validity of the Argument


For this argument, we disregard rows 1 and 3 since the premises are not both true.
Looking at row 4, we see that the conclusion is false even if both premises are true. This
means that the argument is invalid.

Example 58: Suppose we are given the following argument: “If I am going to run the
marathon, then I will buy new shoes. If I buy new shoes, then I will not buy a television.
Therefore, if I buy a television, I will not run the marathon.” Let us use the following symbols:
𝑚 for “I am going to run the marathon,” 𝑠 for “I will buy new shoes,” and 𝑡 for “I will buy a
television.”
Solution:

1. The symbolic form and truth table for this argument are below:

𝑚→ 𝑠
𝑠 →∼ 𝑡
∴ 𝑡 →∼ 𝑚

Property of and for the exclusive use of SLU. Reproduction, storing in a retrieval system, distributing, uploading or posting online, or transmitting in any form or by any
means, electronic, mechanical, photocopying, recording, or otherwise of any part of this document, without the prior written permission of SLU, is strictly prohibited. 64
2. Construct a Truth Table

First Premise Second Premise Conclusion


𝑚 𝑠 𝑡 𝑚→ 𝑠 𝑠 →∼ 𝑡 𝑡 →∼ 𝑚
T T T T F F
T T F T T T
T F T F T F
T F F F T T
F T T T F T
F T F T T T
F F T T T T
F F F T T T

3. Validity of the Argument

In the highlighted rows, it can be seen that the conclusion is true whenever both
premises are true. Thus, the argument is valid.

UNIT 2: PROBLEM SOLVING

Video viewing: Polya explains the problem solving technique


https://www.youtube.com/watch?v=h0gbw-Ur_do&t=1028s

INTRODUCTION

Every day we encounter problems. For sure, everyone has felt at least once in his or
her life how wonderful it is if we could solve a problem at hand, preferably without much
difficulty or even with some difficulties. Unfortunately, problem-solving is an art at this point,
for there are no universal approaches one can take to solving problems. Basically, one must
explore possible avenues to a solution one by one until one comes across the right path to
a solution. However, in general, as one gains experience in solving problems, one develops
one's own techniques and strategies, though they are often intangible.
In this section, we are going to learn problem-solving and get a glimpse of strategies
that are often used by experts. They are based on the work of Polya. George Polya, known
as the father of modern problem solving, did extensive studies and wrote numerous
mathematical papers and three books about problem-solving. In 1945, he published the
book “How to Solve It: a new aspect of mathematical method” which quickly became his
most prized publication. In his book, he identifies four basic steps of problem-solving and
reveals how the mathematical method of demonstrating a proof or finding an unknown can
be of help in attacking any problem that can be “reasoned” out.

Property of and for the exclusive use of SLU. Reproduction, storing in a retrieval system, distributing, uploading or posting online, or transmitting in any form or by any
means, electronic, mechanical, photocopying, recording, or otherwise of any part of this document, without the prior written permission of SLU, is strictly prohibited. 65
Polya’s famous four-step method for problem solving

First Step: Understand the problem


Guide questions:
❏ Do you understand all the words used in stating the problem?
❏ What are you asked to find or show?
❏ Can you restate the problem in your own words?
❏ Can you think of a picture or diagram that might help you understand the
problem?
❏ Is there enough information to enable you to find a solution?
If so, then:
1. What is the unknown?
2. What is the data?
3. What is the condition?
4. Is it possible to satisfy the condition?
5. Is the condition sufficient to determine the unknown?
6. Or is it insufficient? Or redundant? Or contradictory?
7. Draw a figure. Introduce suitable notation.
8. Separate the various parts of the condition. Write them down.

Second Step: Devise a plan


Find the connection between the data and the unknown and obtain a plan of the
solution. Successful problem solvers use a variety of techniques when they attempt to solve

Property of and for the exclusive use of SLU. Reproduction, storing in a retrieval system, distributing, uploading or posting online, or transmitting in any form or by any
means, electronic, mechanical, photocopying, recording, or otherwise of any part of this document, without the prior written permission of SLU, is strictly prohibited. 66
a problem. Polya mentioned that there are many reasonable ways to solve problems. Here
are some strategies used:
a. Working Backwards. This is the strategy that “advertises” itself by stating the end
conditions of the problem and asking to find the starting conditions.
b. Logical Reasoning. Problems involving logical reasoning often include a
substantial amount of data that, at first glance, appears overwhelming and
confusing. In this strategy you need to draw logical conclusions from such data.
c. Finding a pattern. A powerful problem-solving strategy for problems is examining
data to see if a pattern exists then use it to solve the problem. Used if a series of
numbers or events are in the problem.
d. Creating a visual representation. If the situation described in the problem is difficult
to visualize, a diagram, using simple symbols or pictures, may enable us to see the
situation more easily.
e. Solve a similar but simpler problem. This is used when it is not possible to gather,
explore, understand, relate, conjecture and to analyze data, in a meaningful way.
In this case, it would be better to formulate an equivalent problem but simpler
form.
f. Organizing Data. This is used if a problem presents an excessive amount of data.
A table or a list are used to organize the data in a problem.
g. Considering Extreme Cases. In this strategy, the existence of extreme positions is
often the key to understanding existence results. By considering extremes, we may
be changing variables in the problem, but only those that do not affect the actual
problem situation.
h. Adopt a Different Point of View. A problem can be solved in a more efficient and
interesting manner if we approach it from a different point of view. That is, instead
of considering the problem in the most direct and obvious manner, a different
approach may yield the answer quickly and more efficiently.
i. Accounting all possibilities.Used to solve problems that indicate different
possibilities. Make an organized list that shows all the possibilities
j. Write an Equation. One of the first steps in working with problems is to translate the
problem into symbolic terms where all key concepts are identified and defined to
come up with an equation. This is effectively applicable for algebraic problems.
k. Intelligent Guessing and Testing. This strategy is used when no other strategy is
immediately obvious. Here we make a guess (and it must be an intelligent guess,
not just an uninformed stab at the problem) and then proceeds to test that guess
within the conditions of the problem. We repeat the process until we succeed.
The skill of choosing an appropriate strategy is best learned by solving many
problems.

Property of and for the exclusive use of SLU. Reproduction, storing in a retrieval system, distributing, uploading or posting online, or transmitting in any form or by any
means, electronic, mechanical, photocopying, recording, or otherwise of any part of this document, without the prior written permission of SLU, is strictly prohibited. 67
Third Step: Carry out the plan
In this step we solve the problem applying the plan we devised. All we need in this
step is care and patience, given that we have the necessary skills. If it continues not to work,
discard it and use another plan.

Fourth Step: Look back

This step is the time to reflect and look back at what we have done, what worked,
and what didn’t. Can you use the result, or the method, for some other problem?

Example 59: In a room with 10 people, everyone shakes hands with everybody else exactly
once. How many handshakes are there?
Solution:

Step 1: Understanding the Problem:


Unknown: number of handshakes in a room
Given: 10 people shakes hands with everybody exactly once
Step 2: Devise a plan:
❏ Visual representation
❏ Accounting for all possibilities
❏ Looking for a pattern
❏ Organizing data
Step 3: Carry out the plan:
a. Visual Representation (Draw a Diagram)
Let us use our visual representation strategy, by drawing a diagram. The 10
points (no 3 of each are collinear) represent the 10 people. Begin with the person
represented by point A.

We join A to each of the other 9 points, indicating the first 9 handshakes


that take place.

Property of and for the exclusive use of SLU. Reproduction, storing in a retrieval system, distributing, uploading or posting online, or transmitting in any form or by any
means, electronic, mechanical, photocopying, recording, or otherwise of any part of this document, without the prior written permission of SLU, is strictly prohibited. 68
Now, from B there are 8 additional handshakes (because since A has
already shaken hands with B). Similarly, from C there will be 7 lines drawn to the
other, from D there will be 6 additional lines or handshakes, and so on. When we
reach point I, there is only one remaining handshake to be made, namely, I with
J, since I has already shaken hands with A, B, C, D, E, F, G, H. Thus, the sum of the
handshakes equals 9 + 8 + 7 + 6 + 5 + 4 + 3 + 2 + 1 = 45. In general, this is same as
using the formula for sum of the first n natural numbers, where 𝑛 > 2. (Notice that
the final drawing will be a decagon with all of its diagonals drawn.)

b. Accounting all possibilities


Consider the grid shown below, which indicates persons A, B, C, . . . , H, I, J,
shaking hands with one another. The diagonal with 𝑥’𝑠 indicates that people
cannot shake hands with themselves.

The remaining cells indicate doubly with all other handshakes (i.e., A
shakes hands with B and B shakes hands with A). thus we take the total number
of cells (102 ) minus those on diagonal (10) and divide the result by 2. In this
100−10
case, we have = 45.
2
𝑛2 −𝑛
In a general case for the 𝑛 × 𝑛 grid, the number would be , which
2
𝑛(𝑛−1)
is equivalent to the formula .
2

Property of and for the exclusive use of SLU. Reproduction, storing in a retrieval system, distributing, uploading or posting online, or transmitting in any form or by any
means, electronic, mechanical, photocopying, recording, or otherwise of any part of this document, without the prior written permission of SLU, is strictly prohibited. 69
c. Adopting a different point of view

Let’s now examine the problem by adopting a different point of view.


Consider the room with 10 people, each of whom will shake 9 other people’s
hands. This seems to indicate that there are 10 × 9 or 90 handshakes, but we must
divide by 2 to eliminate the duplication.

d. Finding for a Pattern


Let’s try to solve the problem by looking for a pattern. The table shown
below lists the number of handshakes occurring in a room as the number of people
increases.

Number of people in Number of handshakes Total number of total


room for additional person handshakes in room
1 0 0
2 1 1
3 2 3
4 3 6
5 4 10
6 5 15
7 6 21
8 7 28
9 8 36
10 9 45

The third column, the total number of handshakes, gives a sequence of


numbers known as the triangular numbers, whose successive differences increase
by one each time. It is, therefore, possible to continue the table until we reach the
corresponding sum for the ten people. Alternatively, we note that the pattern at
each entry is half the product of the number of people and the number of
handshakes for additional people.

e. Organizing Data
We can approach a problem by careful use of an organizing data strategy.
The chart below shows each of the people in the room and the number of people
they have to shake with each time, given that they already shaken the hands of
their predecessors and don’t shake their own hands. If there are 10 people in the
room one will shake hands with the other 9 people, making 9 handshakes; and if
there are 9 people in the room one will shake hands with the other 8 people,
making 8 handshakes and so on, until we reach person number 2, who only has
one person’s hand to shake, and person number 1 has no other hands to shake.
Again, the sum is 45.

Property of and for the exclusive use of SLU. Reproduction, storing in a retrieval system, distributing, uploading or posting online, or transmitting in any form or by any
means, electronic, mechanical, photocopying, recording, or otherwise of any part of this document, without the prior written permission of SLU, is strictly prohibited. 70
Organizing data
No. of people 10 9 8 7 6 5 4 3 2 1
No. of handshakes 9 8 7 6 5 4 3 2 1 0

Step 4. Look Back: After reviewing, we can say:


There are 45 handshakes in the room with 10 people.

Example 60: Evelyn, Harley, and Al play a certain game. The player who loses each round
must give each of the other players as much money as the player has that time. In round
1, Evelyn loses and gives Henry and Al as much money as they have. In Round 2, Henry
loses, and gives Evelyn and Al as much money they each then have. Al loses in round 3
and gives Evelyn and Henry as much money as they each have. They decided to quit at
this point and discover that they each have ₱24. How much money did each start with?
Solution:

Step 1: Understand the problem


Unknown: Amount of money Evelyn, Henry, and Al each had before they started to
play.
Given: Each has 24 pesos at the end of the game. The player who loses each round
must give each of the other players as much money as the player has at that time

Step 2: Devise a plan


Consider this question from the problem “How much money did they start with?” This
is a sign the problem is best solved by working backwards.

Step 3. Carry out a plan


We apply working backwards.

Evelyn Henry Al
End of round 3 24 24 24
End of round 2
End of round 1
Start

Since Al lost at the end of round 3 and the player who loses each round must
give each of the other players as much money as the player has that time. This means
that Al gave half of Evelyn’s money and half of Henry’s money. Evelyn’s money at the
end of round 2 was 12 and so with Henry.

Property of and for the exclusive use of SLU. Reproduction, storing in a retrieval system, distributing, uploading or posting online, or transmitting in any form or by any
means, electronic, mechanical, photocopying, recording, or otherwise of any part of this document, without the prior written permission of SLU, is strictly prohibited. 71
Evelyn Henry Al
End of round 3 24 24 24
End of round 2 24 − 12 = 12 24 − 12 = 12 24 + 12 + 12 = 48
End of round 1
Start

In round 2, Henry lost, he gave half of Evelyn’s money and half of Al’s money.
Evelyn’s money at the end of round 1 was 6 and Al’s money at the end of Round 1
was 24.

Evelyn Henry Al
End of round 3 24 24 24
End of round 2 24 − 12 = 12 24 − 12 = 12 24 + 12 + 12 = 48
End of round 1 12 − 6 = 6 12 + 6 + 24 = 42 48 − 24 = 24
Start 6 + 21 + 12 = 39 42 − 21 = 21 24 − 12 = 12

In round 1, Evelyn lost, she gave half of Henry’s money and half of Al’s money.
Henry’s money at the start was 21 and Al’s money at the start was 12.

Step 4. Look Back:


At the end of round 1 Evelyn lost so she gave Henry ₱21 and gave Al ₱12 leaving her
₱6.
Evelyn Henry Al
Start 39 21 12
End of round 1 39 − 21 − 12 = 6 21 + 21 = 42 12 + 12 = 24
End of round 2
End of round 3

At the end of round 2 Henry lost so he gave Evelyn ₱6 and gave Al ₱24 leaving him
₱12.
Evelyn Henry Al
Start 39 21 12
End of round 1 39 − 21 − 12 = 6 21 + 21 = 42 12 + 12 = 24
End of round 2 6 + 6 = 12 42 − 6 − 24 = 12 24 + 24 = 48
End of round 3

At the end of round 3 Al lost so he gave Evelyn ₱12 and gave Al ₱12 leaving him ₱24.

Property of and for the exclusive use of SLU. Reproduction, storing in a retrieval system, distributing, uploading or posting online, or transmitting in any form or by any
means, electronic, mechanical, photocopying, recording, or otherwise of any part of this document, without the prior written permission of SLU, is strictly prohibited. 72
Evelyn Henry Al
Start 39 21 12
End of round 1 39 − 21 − 12 = 6 21 + 21 = 42 12 + 12 = 24
End of round 2 6 + 6 = 12 42 − 6 − 24 = 12 24 + 24 = 48
End of round 3 12 + 12 = 24 12 + 12 = 24 48 − 12 − 12 = 24
At the end of round 3 it shows that they ₱24 each.
This proves our answer that Evelyn began with ₱39, Henry began with ₱21 and Al
began with ₱12.

Learning Reinforcement 4

Directions: Write your solutions and answers on a clean sheet of


paper, or you may print this page and answer there. Submit the
image of your HANDWRITTEN SOLUTIONS as a single pdf file in the
submission bin for this activity in the Classroom. You may use
image scanning apps on your phone (CamScanner or Tap
Scanner) to save several images into one pdf file, or place your
images in a document and save them as a pdf file.

1. Four friends (including Adam) were at the company


picnic. Each friend had a different drink (one was juice), with a different mix added
(one had vanilla). From the clues provided can you figure out which friend had which
drink and what was added to his/her drink?
CLUES
• Wilma did not drink water mixed with her lemon.
• Only the men drank soda or water.
• Mary was not the one who had water & lime.
• David did not mix his drink with lime.
• The woman who had strawberry did not have tea.
2.

Property of and for the exclusive use of SLU. Reproduction, storing in a retrieval system, distributing, uploading or posting online, or transmitting in any form or by any
means, electronic, mechanical, photocopying, recording, or otherwise of any part of this document, without the prior written permission of SLU, is strictly prohibited. 73
3. Let 𝑝 and 𝑞 be the statements
𝑝: I bought a lottery ticket this week.
𝑞: I won the jackpot on Friday.
Express each of these statements as a sentence:
a.) 𝑝 ↔ 𝑞
b.) ∼ 𝑝 ∨ (𝑝 ∧ 𝑞)
4. Let 𝑝, 𝑞, and 𝑟 be the statements
𝑝: Grizzly bears have been seen in the area.
𝑞: Hiking is safe on the trail.
𝑟: Berries are ripe along the trail.
Write the following statements in symbolic form.
a.) If berries are ripe along the trail, then hiking is safe if and only if grizzly bears have
not been seen in the area.
b.) Grizzly bears have not been seen in the area and hiking on the trail is safe, but
berries are ripe along the trail.
c.) Hiking is not safe on the trail whenever grizzly bears have been seen in the area
and berries are ripe along the trail.
5. Determine whether these biconditionals are true or false.
a.) 1 + 1 = 3 if and only if monkeys can fly.
b.) 0 > 1 if and only if 2 > 1.
6. Write each of the propositions in the form “𝑝 if and only if 𝑞”:
a.) For you to get a passing mark in this course, it is necessary and sufficient that you
learn how to solve mathematics problems.
b.) If you read the newspaper every day, you will be informed, and conversely.
7. Write the proposition “I come to class whenever there is going to be a quiz” in the
form “if p then q” then state its converse, contrapositive, and inverse.
a. Conditional “if p then q”
b. Converse:
c. Contrapositive:
d. Inverse:
8. Let 𝑝, 𝑞, and 𝑟 be true, false and false, respectively. Determine the truth value of the
following?
a.) (𝑝 → 𝑞) ∧∼ 𝑟
b.) 𝑞 ↔ (𝑝 ∧ 𝑟)
9. Determine the validity of an argument: A polygon is regular or a polygon has a side
which is longer than another side of the polygon. If a polygon is regular then all the
sides of the polygon are congruent. If a polygon has one side which is longer than
another side of the polygon, then an interior angle of the polygon has measure
greater than one of the interior angles of the polygon. Therefore, either the polygon
have congruent sides or an interior angle of the polygon has a measure greater than
one of the interior angles of the polygon.

Property of and for the exclusive use of SLU. Reproduction, storing in a retrieval system, distributing, uploading or posting online, or transmitting in any form or by any
means, electronic, mechanical, photocopying, recording, or otherwise of any part of this document, without the prior written permission of SLU, is strictly prohibited. 74
10. Solve the following problems following Polya’s Four Step Method and employing the
different strategies discussed. Write the problem and present the solution neatly
and organized.
5
a. Frosia delivers prescriptions for the local pharmacy. On Tuesday, she delivered 9 of
3
the prescriptions already in the delivery van and then 4
of the remaining
2
prescriptions. After picking up 10 more prescriptions at the store, she delivered of
3
those she had with her. She then picked up an additional 12 prescriptions and
7
delivered 8 of those she had in the van. Finally, she up 3 more and then delivered
the remaining 5 prescriptions. How many prescriptions did she deliver?
b. Joyce invited 17 friends to a dinner party at her house last Friday evening. She
gave each guest a card with a number from 2 through 18, reserving number 1 for
herself. When she had everyone paired off at the dinner table, she noticed that
the sum of each couple’s numbers was perfect square. What number did Joyce’s
partner have?

Property of and for the exclusive use of SLU. Reproduction, storing in a retrieval system, distributing, uploading or posting online, or transmitting in any form or by any
means, electronic, mechanical, photocopying, recording, or otherwise of any part of this document, without the prior written permission of SLU, is strictly prohibited. 75
MODULE 5: MATHEMATICS OF FINANCE

Our target learning outcomes are a) Use mathematical concepts and tools in other
areas such as finance and business; b) Differentiate compound interest from simple interest;
c) Apply the interest and annuity formulas to cases of loans, credits, stocks bonds, property
purchases, and investment problems

UNIT 1: SIMPLE INTEREST AND COMPOUND INTEREST

Important Terms
Principal. It refers to the original sum of money borrowed in a loan or put into an
investment.
Interest is the charge for the privilege of borrowing money. From the investor’s
viewpoint, interest is the income from an invested amount at a given rate for a given time.

From the debtor’s viewpoint, interest is the money paid for the use of borrowed
money.

Interest Rate. The interest rate is the amount a lender charges for the use of money
expressed as a percentage of the principal. The interest rate is typically noted on an annual
basis known as the annual interest rate.
Time. This is the period from the beginning when the money was borrowed (or
invested) to the period when the money should be returned with the additional amount
(interest). This is also called the term of loan or term of investment.

Property of and for the exclusive use of SLU. Reproduction, storing in a retrieval system, distributing, uploading or posting online, or transmitting in any form or by any
means, electronic, mechanical, photocopying, recording, or otherwise of any part of this document, without the prior written permission of SLU, is strictly prohibited. 76
A. SIMPLE INTEREST
PowerPoint 14: Simple interest

It refers to the interest paid on the original principal. It is also characterized by a fixed
amount earned over time. Usually, simple interest is associated with loans and investments
which are short term in nature.
The formula in computing for the simple interest is given by
𝐼 = 𝑃𝑟𝑡
where: I – the Interest
P– the Principal
r – the rate of interest
t – the time period

We have to note that the time t should be expressed in years. Unless otherwise stated,
it will be assumed that the interest rate is an annual interest.

Example 61: Find the simple interest earned in an account where ₱4,500 is on deposit for
4 years at 3 1/4% annual interest.
Solution:
1
Given: 𝑃 = ₱4,500, 𝑡 = 4, 𝑟 = 3 4 % = 0.0325
Unknown: 𝐼
Solution:
𝐼 = 𝑃𝑟𝑡 = ₱4,500(0.0325)(4) = ₱585
Answer: 𝐼 = ₱585

Example 62: Find the simple interest for a loan of ₱12,400 due at the end of 8 1/4 years at
4 1/2% annual interest.
Solution:
1 1
Given: 𝑃 = ₱12,400, 𝑡 = 8 = 8.25 , 𝑟 = 4 % = 0.045
4 2
Unknown: 𝐼
Solution: 𝐼 = 𝑃𝑟𝑡 = 12400(0.045)(8.25) = ₱4,603.5
Answer: 𝐼 = ₱4,603.5

Example 63: Find the principal necessary to earn ₱814 in simple interest if the money is to
be left on deposit for 5 years and 3 months and earns 5 1/2% annual interest.
Solution:
3 1
Given: 𝐼 = ₱814, 𝑡 = 5 12 = 5.25, 𝑟 = 5 2 % = 0.055
Unknown: 𝑃
𝐼
Solution: Using 𝐼 = 𝑃𝑟𝑡, we obtain 𝑃 = 𝑟𝑡
814
𝑃 = 0.055(5.25) = ₱2,819.05
Answer: 𝑃 = ₱2,819.05

Property of and for the exclusive use of SLU. Reproduction, storing in a retrieval system, distributing, uploading or posting online, or transmitting in any form or by any
means, electronic, mechanical, photocopying, recording, or otherwise of any part of this document, without the prior written permission of SLU, is strictly prohibited. 77
Example 64: Find the time necessary for a deposit of ₱11,500 to earn ₱3,450 in simple
interest if the money is to earn 3 3/4% annual interest.
Solution:
3
Given: 𝑃 = ₱11,500, 𝐼 = ₱3,450, 𝑟 = 3 4 % = 0.0375
Unknown: 𝑡
𝐼
Solution: Using 𝐼 = 𝑃𝑟𝑡, we obtain 𝑡 = 𝑃𝑟
𝐼 3,450
𝑡= = = 8
𝑃𝑟 11,500(0.0375)
Answer: 8 𝑦𝑒𝑎𝑟𝑠

Converting time to years if the period of a loan (or investment) with an annual interest
rate is given in days
1. Exact method: t = number of days /365
2. Ordinary method: t = number of days /360
Note that the ordinary method is used by most businesses. Unless otherwise specified,
we use the ordinary method.

Example 65: Calculate the simple interest due on a 45-day loan of ₱3500 if the annual
interest rate is 8%.
Solution:
45
Given: 𝑡 = 360 𝑑𝑎𝑦𝑠, 𝑃 = ₱3,500, 𝑟 = 0.08
Unknown: 𝐼
45
Solution: 𝐼 = 𝑃𝑟𝑡 = 3,500(0.08) (360) = ₱35
Answer: 𝐼 = ₱35
*We have used the ordinary method as we convert the time in days into years as
there is no method specified in the problem.

Example 66: Calculate the simple interest due on a 120-day loan of ₱7,000 using the exact
method if the annual interest rate is 5.25%.
Solution:
120
Given: 𝑡 = 365 𝑑𝑎𝑦𝑠, 𝑃 = ₱7,000, 𝑟 = 0.0525
Unknown: 𝐼
120
Solution: 𝐼 = 𝑃𝑟𝑡 = 7,000(0.0525) (365) = ₱120.82
Answer: 𝐼 = ₱120.82

Time in between two dates . When time is given between two dates, the time in days
is determined using:
 Actual time uses the exact number of days in each month
 Approximate time assumes 30 days per month for all months

Property of and for the exclusive use of SLU. Reproduction, storing in a retrieval system, distributing, uploading or posting online, or transmitting in any form or by any
means, electronic, mechanical, photocopying, recording, or otherwise of any part of this document, without the prior written permission of SLU, is strictly prohibited. 78
Methods of computing for the simple interest when time is given between two dates

1. Ordinary Interest for actual number of days


𝑎𝑐𝑡𝑢𝑎𝑙 𝑡𝑖𝑚𝑒
𝐼𝑂 = 𝑃𝑟 ( )
360
This is referred to as the Banker’s Rule.

2. Ordinary Interest for approximate number of days


𝑎𝑝𝑝𝑟𝑜𝑥𝑖𝑚𝑎𝑡𝑒 𝑡𝑖𝑚𝑒
𝐼𝑂 = 𝑃𝑟 ( )
360
3. Exact Interest for actual number of days
𝑎𝑐𝑡𝑢𝑎𝑙 𝑡𝑖𝑚𝑒
𝐼𝑒 = 𝑃𝑟 ( )
365
4. Exact interest for approximate number of days
𝑎𝑝𝑝𝑟𝑜𝑥𝑖𝑚𝑎𝑡𝑒 𝑡𝑖𝑚𝑒
𝐼𝑒 = 𝑃𝑟 ( )
365
Note: Use the Banker’s rule unless otherwise specified. When we count the number of days
in between two dates we do not include the first day but we include the last day. Take note
that February may either have 29 or 28 days depending on if it is a leap year or not.

Example 67: Calculate the simple interest due on a loan of ₱2000, at 6.5% simple interest,
which was availed on July 12, 2019, and to be repaid on December 12, 2019. Use the four
methods of computing for simple interest.
Solution:
Given: 𝑃 = ₱2,000, 𝑟 = 0.065
𝑡: July 12, 2019 → December 12, 2019
Unknown: 𝐼 using the four different methods
Solution: First we count the number of days between July 12, 2019 and December
12, 2019 using actual time and approximate time.

Actual Time Approximate Time


July 31-12=19 July 30-12=18
August 31 August 30
September 30 September 30
October 31 October 30
November 30 November 30
December 12 December 12
Total 153 Total 150

a) Ordinary Interest for actual number of days (Banker’s Rule)


𝑎𝑐𝑡𝑢𝑎𝑙 𝑡𝑖𝑚𝑒 153
𝐼𝑂 = 𝑃𝑟 ( ) = 2,000(0.065) ( ) = ₱55.25
360 360

Property of and for the exclusive use of SLU. Reproduction, storing in a retrieval system, distributing, uploading or posting online, or transmitting in any form or by any
means, electronic, mechanical, photocopying, recording, or otherwise of any part of this document, without the prior written permission of SLU, is strictly prohibited. 79
b) Ordinary Interest for approximate number of days
𝑎𝑝𝑝𝑟𝑜𝑥𝑖𝑚𝑎𝑡𝑒 150
𝐼𝑂 = 𝑃𝑟 ( ) = 2,000(0.065) ( ) = ₱54.17
360 360
c) Exact Interest for actual number of days
𝑎𝑐𝑡𝑢𝑎𝑙 𝑡𝑖𝑚𝑒 153
𝐼𝑒 = 𝑃𝑟 ( ) = 2,000(0.065) ( ) = ₱54.49
365 365
d) Exact Interest for approximate number of days
𝑎𝑝𝑝𝑟𝑜𝑥𝑖𝑚𝑎𝑡𝑒 𝑡𝑖𝑚𝑒 150
𝐼𝑒 = 𝑃𝑟 ( ) = 2,000(0.065) ( ) = ₱53.42
365 365
Note: Among the four methods of computing simple interest, the Banker’s rule will usually
give the highest interest.

Final Amount. The sum of the principal and the interest which is accumulated at a
certain time. Final amount could be the Future Value or the Maturity Value
 Future Value (of an investment). A term used to refer to the total amount on
deposit after the interest earned has been added to the principal.
 Maturity Value (of the loan). A term used to refer to the total amount to be
repaid to the lender where the amount is the interest due on the loan plus the
principal.
The formula in computing for the Final Amount is given as:
𝐹 =𝑃+𝐼
𝐹 = 𝑃 + 𝑃𝑟𝑡
𝐹 = 𝑃(1 + 𝑟𝑡)
where: F - the Final Amount.
P - the Principal.
I – the Interest.

Example 68: Calculate the maturity value of simple interest, eight-month loan of ₱8,000 if
the interest rate is 9.75%.
Solution:
8 2
Given: 𝑡 = 12 = 3 , 𝑃 = ₱8,000, 𝑟 = 0.0975
Unknown: 𝐹
2
Solution: 𝐹 = 𝑃 (1 + 𝑟𝑡) = 8,000 [1 + 0.0975 (3)] = ₱8,520
Answer: 𝐹 = ₱8,520

Example 69: What principal will accumulate to ₱135,000 in 2 years at 15% simple interest?
Solution:
Given: 𝐹 = ₱135,000, 𝑡 = 2, 𝑟 = 0.15
Unknown: 𝑃
𝐹
Solution: Using 𝐹 = 𝑃(1 + 𝑟𝑡) we obtain 𝑃 = 1+𝑟𝑡 .
𝐹 135,000
𝑃 = 1+𝑟𝑡 = 1+(2)(0.15) = ₱103,846.15
Answer: 𝑃 = ₱103,846.15

Property of and for the exclusive use of SLU. Reproduction, storing in a retrieval system, distributing, uploading or posting online, or transmitting in any form or by any
means, electronic, mechanical, photocopying, recording, or otherwise of any part of this document, without the prior written permission of SLU, is strictly prohibited. 80
Example 70: How many years are needed for ₱5,700 to accumulate to P6,555 at 3.5%
simple interest?
Solution:
Given: 𝑃 = ₱5,700, 𝐹 = ₱6,555 𝑟 = 0.035
Unknown: 𝑡
𝐹
( −1)
Solution: Using 𝐹 = 𝑃(1 + 𝑟𝑡) we obtain 𝑡 = 𝑃
𝑟
.

𝐹 6,555
( −1) ( −1)
𝑃 5,700
𝑡= 𝑟
= 0.035
= 4.29
Answer: 𝑡 = 4.29 years

B. COMPOUND INTEREST
PowerPoint 15: Compound Interest

It is the interest resulting from the periodic addition of simple interest to the principal
to create a new principal every now and then. Interest is charged (or paid) on interest as
well as on principal. The compound interest is the sum by which the original principal has
been increased by the end of the term.
The total accumulated amount at the end of the period which is the original principal
plus the compound interest is called the compound amount or final amount.
The formula in computing for the compound amount is given by

𝑟
𝐹 = 𝑃(1 + 𝑖)𝑛 where 𝑖 = and 𝑛 = 𝑚𝑡, or
𝑚

𝑟 𝑚𝑡
) 𝐹 = 𝑃 (1 +
𝑚
and the formulas for the compound interest and the principal are
𝐼 =𝐹−𝑃 𝑃 = 𝐹(1 + 𝑖)−𝑛
where: F – the Final amount or Compound amount
P – the Principal
I – the Compound Interest
i - the interest rate per period or periodic rate
t – time or term of investment, expressed in years
n – number of conversion periods (or compounding periods) for the
whole term.
m – the number of times interest is computed per year;
the number of conversion periods per year;
r – the nominal rate of interest per annum or year.

Property of and for the exclusive use of SLU. Reproduction, storing in a retrieval system, distributing, uploading or posting online, or transmitting in any form or by any
means, electronic, mechanical, photocopying, recording, or otherwise of any part of this document, without the prior written permission of SLU, is strictly prohibited. 81
*Conversion or Compounding periods
Conversion/Compounding Period Value of m
Annually 1
Semi-annually 2
Quarterly 4
Monthly 12

Simple Interest and Compound Interest Compared


To better understand the difference between simple and compound interest, let us
use the following information.
Given: P=₱1,000,
𝑟 = 0.01,
𝑚 = 1,
𝑡=3
Simple Interest
Year Principal (amount Interest for the period Total interest for the
earning an interest) covered period covered

1 ₱1,000 ₱100 ₱100

2 1,000 100 100

3 1,000 100 100


Final Amount at the end of 3 years where F = P (1 +rt) ₱1,000 + ₱300
F = 1000 [1 +(0.1)(3)] = ₱1,300 = ₱1,300

*The same ₱1,000 earns an interest per period.


Compound Interest
Year Principal (amount earning Interest for the period Total interest for the
an interest) covered period covered

1 ₱1000 ₱100 (a) ₱100

2 1000 100 (b)

100 (a) 10 (c) ₱110

3 ₱1000 ₱100 (d)

100 (a) 10 (e)

100 (b) 10 (f)

100 (c) 1(g) ₱121

Property of and for the exclusive use of SLU. Reproduction, storing in a retrieval system, distributing, uploading or posting online, or transmitting in any form or by any
means, electronic, mechanical, photocopying, recording, or otherwise of any part of this document, without the prior written permission of SLU, is strictly prohibited. 82
Final Amount at the end of 3 years where 𝐹 = 𝑃(1 + 𝑖)𝑛
F  10001  0.1  ₱ 1,331 ₱1000+₱331=₱1,331
3

*The same ₱1,000 earns an interest per period but the interests also earn interest.
Comparing simple and compound interest there is a ₱31 discrepancy. Said amount
is the accumulated interest of the interests.

Example 71: Find the compound amount and interest if ₱1,000 is invested at 2%
compounded quarterly for 1 year and 6 months.
Solution:
Given: 𝑃 = ₱1,000, 𝑟 = 0.02, 𝑚 = 4, 𝑡 = 1.5
Unknown: 𝐹, 𝐼
𝑟 𝑚𝑡 0.02 4(1.5)
Solution: 𝐹 = 𝑃 (1 + 𝑚) = 1,000 (1 + 4
) = ₱1,030.38
𝐼 = 𝐹 − 𝑃 = 1,030.38 − 1,000 = ₱30.38
Answer: 𝐹 = ₱1,030.38 and 𝐼 = ₱30.38

Cash Value. In some business transactions involving money, especially during the
onset of the purchase of an expensive good or service, a down payment is required and in
most cases, the purchaser makes financing arrangements to cover the remaining amount
owed to the seller. (Investopedia). In the case that we want to compute for the cash value
then we use
CASH VALUE = Down payment(DP) + Present value of all future payments
If no down payment is required, then
CASH VALUE = Present value of all future payments

Example 72: An equipment is purchased on installment by your company. Your company


paid an amount of ₱205,000 and owes a balance of ₱500,000 to be paid at the end of
seven years. Find the cash value of the equipment if money is worth 5% compounded
quarterly.
Solution:
Given: 𝐷𝑃 = ₱205,000, 𝐹 = ₱500,000, 𝑟 = 0.05, 𝑚 = 4, 𝑡 = 7
Unknown: 𝐶𝑎𝑠ℎ 𝑉𝑎𝑙𝑢𝑒
To solve for the cash value, we have to determine the present value 𝑃 of the future
payment of ₱500,000.

𝑟 −𝑚𝑡 0.05 −4(7)


𝑃 = 𝐹 (1 + ) = 500,000 (1 + ) = ₱353,109.26
𝑚 4
Then , 𝐶𝑎𝑠ℎ 𝑣𝑎𝑙𝑢𝑒 = ₱205,000 + ₱353,109.26 = ₱558,109.26
Answer: 𝐶𝑎𝑠ℎ 𝑣𝑎𝑙𝑢𝑒 = ₱558,109.26

Property of and for the exclusive use of SLU. Reproduction, storing in a retrieval system, distributing, uploading or posting online, or transmitting in any form or by any
means, electronic, mechanical, photocopying, recording, or otherwise of any part of this document, without the prior written permission of SLU, is strictly prohibited. 83
UNIT 2: ANNUITY (or SIMPLE ANNUITY)

An annuity refers to a sequence of equal payments made at equal intervals of time.


This identifies the elements of an annuity as follows:
a. Sequences or series of payments
b. Payments are of equal amounts
c. Payments are made at equal intervals of time.
The concept of annuity applies to premiums of life insurance, periodic payments of
rentals, purchase of car or house, interest payment on bonds, or payment of household
appliances purchased on installment.
The term of an annuity refers to the period of time from the beginning of the first
payment interval up to the last payment interval. The size or value of each payment is called
the periodic payment. The time between successive periodic payments is called the
payment interval.
The Present Value of an Annuity, denoted by A, is the value of the annuity at the
beginning of its term.
The Amount of an annuity or Sum of an Annuity, denoted by S, is the value of the
annuity at the end of its term.

Classification of (Simple) Annuity Certain


1. Ordinary annuity – annuity where the periodic payments are made at the end of
each payment interval.
2. Annuity Due – annuity where the periodic payments are made at the beginning of
each payment interval.
3. Deferred Annuity – annuity where the first periodic payment is not made at the
beginning nor at the end of the first payment interval but at some later date.

A. Ordinary Annuity
An annuity where the periodic payments are made at the end of payment intervals.

Formulas for Ordinary Annuity


1. Final Amount of Ordinary Annuity
𝑟 𝑚𝑡
(1+𝑖)𝑛−1 (1+ ) −1
𝑚
𝑆𝑜𝑟𝑑 = 𝑅 [ ] = 𝑅[ 𝑟 ]
𝑖
𝑚

Property of and for the exclusive use of SLU. Reproduction, storing in a retrieval system, distributing, uploading or posting online, or transmitting in any form or by any
means, electronic, mechanical, photocopying, recording, or otherwise of any part of this document, without the prior written permission of SLU, is strictly prohibited. 84
2. Present Value of Ordinary Annuity
𝑟 −𝑚𝑡
1−(1+𝑖)−𝑛 1−(1+ )
𝑚
𝐴𝑜𝑟𝑑 = 𝑅 [ 𝑖
] = 𝑅[ 𝑟 ]
𝑚

where: 𝑆 – the Sum or Amount of annuity


𝐴 – the Present value of an annuity
𝑅 – Periodic payment
𝑡 – the term of the annuity
𝑖 – the periodic rate
𝑟 – the rate of interest of an annuity
𝑚 – number of conversion periods.

Example 73: A man deposits ₱12,200 every end of 6 months in an account paying 2%
compounded semi-annually. What amount is in the account at the end of 9 years and 6
months?
Solution:
Given: 𝑅 = ₱12,200, 𝑚 = 2, 𝑟 = 0.02, 𝑡 = 9.5𝑦𝑟𝑠
Unknown: 𝑆𝑜𝑟𝑑
Solution:
𝑟 𝑚𝑡 0.02 2(9.5)
(1 + ) − 1 (1 + ) −1
𝑚 2
𝑆𝑜𝑟𝑑 = 𝑅 [ 𝑟 ] = 12,200 [ ] = ₱253,892.92
0.02
𝑚 2
Answer: 𝑆𝑜𝑟𝑑 = ₱253,892.92

Example 74: A home video entertainment set is offered for sale for ₱18,000 down payment
and ₱1,800 every three months for the balance, for 18 months. If interest is to be computed
at 10% converted quarterly, what is the cash price equivalent of the set?
Solution:
18
Given: 𝐷 = ₱18,000, 𝑅 = ₱1,800, 𝑡 = = 1.5, 𝑟 = 0.10, 𝑚 = 4
12
Unknown: 𝐶𝑎𝑠ℎ 𝑉𝑎𝑙𝑢𝑒
Solution: To find the cash value, we need to solve first for the present value of future
payments. As the future payments are in the form of an ordinary annuity, we simply
solve for 𝐴𝑜𝑟𝑑 .
𝑟 −𝑚𝑡 0.10 −4(1.5)
1 − (1 + 𝑚 ) 1 − (1 + 4 )
𝐴𝑜𝑟𝑑 = 𝑅 [ 𝑟 ] = 1,800 [ ] = ₱9,914.63
0.10
𝑚 4
Now for the cash value,
𝐶𝑎𝑠ℎ 𝑉𝑎𝑙𝑢𝑒 = 𝐷 + 𝐴𝑜𝑟𝑑 = ₱18,000 + ₱9,914.63 = ₱27,914.63
Answer: 𝐶𝑎𝑠ℎ 𝑉𝑎𝑙𝑢𝑒 = ₱27,914.63

Property of and for the exclusive use of SLU. Reproduction, storing in a retrieval system, distributing, uploading or posting online, or transmitting in any form or by any
means, electronic, mechanical, photocopying, recording, or otherwise of any part of this document, without the prior written permission of SLU, is strictly prohibited. 85
Example 75: How much monthly deposit must be made for 5 years and 5 months in order
to accumulate ₱120,000 at 3% compounded monthly?
Solution:
5
Given: 𝑡 = 5 12 , 𝑆𝑜𝑟𝑑 = ₱120,000, 𝑟 = 0.03, 𝑚 = 12
Unknown: 𝑅
𝑟 𝑚𝑡
(1+ ) −1
Solution: From 𝑆𝑜𝑟𝑑 = 𝑅 [ 𝑚
𝑟 ] we can obtain
𝑚

𝑟 𝑚𝑡
−1
𝑟
(1+ ) −1
𝑚 𝑚
𝑅 = 𝑆𝑜𝑟𝑑 [ 𝑟 ] = 𝑆𝑜𝑟𝑑 [ 𝑟 𝑚𝑡
].
𝑚 (1+ ) −1
𝑚

𝑟 0.03

Then , 𝑅 = 𝑆𝑜𝑟𝑑 [ 𝑚
𝑟 𝑚𝑡
] = 120,000 [ 12
5 ] = ₱1,702.52
(1+ ) −1 0.03 12(5+12)
𝑚 (1+ ) −1
12

Answer: 𝑅 = ₱1,702.52

Example 76: Bebong wants to buy a car worth ₱740,000. He can pay 40% of the price as
down payment and the balance payable every end of the month for 60 months, how
much must he pay monthly at 9% compounded monthly?
Solution:
Given: 𝐶𝑎𝑠ℎ 𝑝𝑟𝑖𝑐𝑒 = ₱740,000, 𝐷𝑃 = ₱740,000 (0.4) = ₱296,000
60
𝑡= = 5, 𝑟 = 0.09, 𝑚 = 12
12
Unknown: 𝑅
𝑟 −𝑚𝑡 𝑟
1−(1+ )
Solution: From 𝐴𝑜𝑟𝑑 = 𝑅 [ 𝑚
𝑟 ] we can obtain 𝑅 = 𝐴𝑜𝑟𝑑 [ 𝑚
𝑟 −𝑚𝑡
].
𝑚 1−(1+ )
𝑚

Since 𝐶𝑎𝑠ℎ 𝑝𝑟𝑖𝑐𝑒 = 𝐷 + 𝐴𝑜𝑟𝑑 , this implies that


𝐴𝑜𝑟𝑑 = 𝐶𝑎𝑠ℎ 𝑝𝑟𝑖𝑐𝑒 − 𝐷 = ₱740,000 − ₱296,000 = ₱444,000
𝑟 0.09

Finally , 𝑅 = 𝐴𝑜𝑟𝑑 [ 𝑚
𝑟 −𝑚𝑡
] = 444,000 [ 12
0.09 −12(5)
] = ₱9,216.71
1−(1+ ) 1−(1+ )
𝑚 12

Answer: 𝑅 = ₱9,216.71

B. Annuity Due
An annuity where the periodic payments are made at the beginning of each
payment interval.

Property of and for the exclusive use of SLU. Reproduction, storing in a retrieval system, distributing, uploading or posting online, or transmitting in any form or by any
means, electronic, mechanical, photocopying, recording, or otherwise of any part of this document, without the prior written permission of SLU, is strictly prohibited. 86
Formulas for Annuity Due
1. Final Amount of Annuity Due
𝑟 𝑚𝑡+1
(1+𝑖)𝑛+1 −1 (1+ ) −1
𝑚
𝑆𝑑𝑢𝑒 = 𝑅 [ 𝑖
− 1] = 𝑅 [ 𝑟 − 1]
𝑚

2. Present Value of Annuity Due


𝑟 −𝑚𝑡+1
1−(1+𝑖)−𝑛+1 1−(1+ )
𝑚
𝐴𝑑𝑢𝑒 = 𝑅 [ 𝑖
+ 1] = 𝑅 [ 𝑟 + 1]
𝑚

Example 77: Mimay wants to have ₱95,000 for his traveling expenses four years from
now. How much must he save at the beginning of each quarter starting today, if he gets
0.6% compounded quarterly interest on his savings?
Solution:
Given: 𝑆𝑑𝑢𝑒 = ₱95,000, 𝑡 = 4, 𝑟 = 0.0006, 𝑚 = 4
Unknown: 𝑅
(1+𝑖)𝑛+1−1 (1+𝑖)𝑛−1
Solution: Consider 𝑆𝑑𝑢𝑒 = 𝑅 [ 𝑖
− 1] = 𝑅 (1 + 𝑖 ) [ 𝑖
].
0.0006
𝑆𝑑𝑢𝑒 𝑖 95,000,
Then we obtain 𝑅 = [ ]=
(1+𝑖) (1+𝑖)𝑛 −1 (1+0.0006)
[ 4
0.0006 4(4)
] = ₱5929.93
4 (1+ ) −1
4

Answer: 𝑅 = ₱5929.93

Learning Reinforcement 5

Directions: Write your solutions and answers on a clean sheet of


paper, or you may print this page and answer there. Submit the
image of your HANDWRITTEN SOLUTIONS as a single pdf file in the
submission bin for this activity in the Classroom. You may use
image scanning apps on your phone (CamScanner or Tap
Scanner) to save several images into one pdf file, or place your
images in a document and save them as a pdf file.

1. On February 19, 2020, Trisha borrowed ₱50,000 from a


community cooperative at 9% simple interest. The loan is payable on September 12,
2020. Compute the simple interest using the four methods. How much will be the
maturity value if the Banker’s Rule is used?
2. Mike deposited on March 12, 2020 at 10.5% simple interest. On September 8, 2020, the
fund accumulated to ₱12,630. How much was the amount originally invested?
3. Mr. Tan borrowed ₱15,000 from the faculty fund at 13.25% simple interest for 210 days.
How much would he repay at the end of the term if he is charged exact interest?

Property of and for the exclusive use of SLU. Reproduction, storing in a retrieval system, distributing, uploading or posting online, or transmitting in any form or by any
means, electronic, mechanical, photocopying, recording, or otherwise of any part of this document, without the prior written permission of SLU, is strictly prohibited. 87
4. How much money should be invested in an account that earns 9.5% interest,
compounded monthly, in order to have ₱30,000 in 5 years?
5. Accumulate ₱180,000 for 5 years at 13% compounded semi-annually. (Note:
accumulating an amount means finding its maturity value.)
6. At what interest rate will a principal of ₱9,500 accumulate to ₱15,000 in 2 years
compounded semi-annually?
7. Polly purchased a car. She paid ₱150,000 as down payment and pays ₱5,500 at the
end of each month for 48 months. If the interest is 7.8% compounded monthly, how
much was the car worth?
8. A house and lot are worth ₱4.3 million in cash. A buyer pays a 40% down payment
and agrees to pay the balance by equal payments at the end of each month for 10
years at the rate of 9% compounded monthly. How much will be the monthly
payment?
9. In order to have ₱1 million in a fund at the end of 15 years, how much must be
deposited in the fund every quarter if money can be invested at 10.5% compounded
quarterly?

Well done! It’s time to answer Quiz 2 and prepare for the MIDTERM
EXAMINATION!

Property of and for the exclusive use of SLU. Reproduction, storing in a retrieval system, distributing, uploading or posting online, or transmitting in any form or by any
means, electronic, mechanical, photocopying, recording, or otherwise of any part of this document, without the prior written permission of SLU, is strictly prohibited. 88
MODULE 6: DATA MANAGEMENT

Our target learning outcomes are a) Use a variety of statistical tools to process and
manage numerical data; b) Use the methods of linear regression and correlations to predict
the value of a variable given certain conditions; c) Advocate the use of statistical data in
making important decisions

UNIT 1: INTRODUCTION TO STATISTICS


PowerPoint 19: Definition of Statistics and terminologies

What is Statistics?
Statistics is the science of collection, organizing, presenting, analyzing, and
interpreting data to assist in making more effective decisions.

Why study statistics?


Data is everywhere. Statistical techniques are used to make many decisions that
affect our lives. No matter what your career, you will make professional decisions that involve
data. An understanding of statistical methods will help you make these decisions effectively

A. Divisions of Statistics
1. Descriptive Statistics. It deals with the methods of organizing, summarizing, and
presenting a mass of data to yield meaningful information. It includes anything done
to the data designed to summarize, or describe without any attempt to make
inference or conclusion about the gathered data.
Activities:
 Collect data; e.g., Survey
 Present data; e.g., Tables and graphs
 Summarize data; e.g., Sample mean
2. Inferential Statistics. It is concerned with generalizing about a population or other
groups of data based on the study of the sample. It comprises those methods
concerned with the analysis of a subset of data leading to predictions or inferences
about the entire set of data.
Activities:
 Estimation; e.g., Estimate the population mean weight using the
sample mean weight
 Hypothesis testing; e.g., Test the claim that the population mean
weight is 70 kg

Property of and for the exclusive use of SLU. Reproduction, storing in a retrieval system, distributing, uploading or posting online, or transmitting in any form or by any
means, electronic, mechanical, photocopying, recording, or otherwise of any part of this document, without the prior written permission of SLU, is strictly prohibited. 89
B. Population and Sample
1. Population. It consists of the totality of the observations with which we are concerned.
It refers to a group of a total number of people, objects, or reactions that can be
described as having a unique or combination of qualities. Population can be either
finite or infinite.
 Parameter is any numerical value describing a characteristic of a population
usually represented by Greek letters.
Examples:
 If we consider all math classes to be the population, then the
average number of points earned per student over all the math
classes is an example of a parameter.
 There are 35, 000 students enrolled in a university and 15 % of
them are enrolled in math. The figure of 15% is a parameter
because it is based on the entire population of all enrolled
students.
2. Sample . It refers to a finite number of objects selected from the population. It is a
collection of some elements in a population or is a representative of the entire
population.
 Statistic is any numerical values describing a characteristic of a sample and
usually represented by the ordinary letters of the English alphabets
Example:
 If we consider one math class to be a sample of the population
of all math classes, then the average number of points earned
by students in that one math class at the end of the term is an
example of a statistic. The statistic is an estimate of a population
parameter, in this case the mean.
 An institution polled 2.3 million adults in the Philippines and 80%
said that they would vote for the presidency. That figure of 80 %
is a statistic because it is based on a sample, not the entire
population of all adults in the Philippines.

An illustration below is given to differentiate population and sample.

Property of and for the exclusive use of SLU. Reproduction, storing in a retrieval system, distributing, uploading or posting online, or transmitting in any form or by any
means, electronic, mechanical, photocopying, recording, or otherwise of any part of this document, without the prior written permission of SLU, is strictly prohibited. 90
C. Sample Size Determination

The number of respondents or subjects to form a sample is termed as the sample


size.
1. Cochran (1977) presented a set of formulas that can be used to determine the
sample size.
For finite and known For an infinite or unknown
population size, N population size, N:

Estimating a
Population Mean

Estimating a
Population Proportion

Where n is the sample size, ,𝒁𝜶 is the two- tailed z- score corresponding to the level of
𝟐

significance, s is the known standard deviation, e is the margin of error, p is the past
estimate of the population proportion, and q=1-p
NOTE
a. The level of significance,𝛼 , can take any of the standard values namely, 0.01,
0.05, and 0.10. Theoretically, the level of significance is the probability of the
type 1 error in hypothesis testing.
b. The following table presents the values of 𝒁𝜶 corresponding to the standard
𝟐

values of 𝛼

𝛼 𝒁𝜶
𝟐

0.01 2.575
0.05 1.96
0.10 1.645

c. The standard deviation, s, can be estimated from a pilot data set or the value
can be adopted from a previous study that considered the same or similar
population.
d. In the same manner as s, p can be the past estimate of the population
proportion or can be computed from a pilot data set.

Property of and for the exclusive use of SLU. Reproduction, storing in a retrieval system, distributing, uploading or posting online, or transmitting in any form or by any
means, electronic, mechanical, photocopying, recording, or otherwise of any part of this document, without the prior written permission of SLU, is strictly prohibited. 91
2. Yamane’s Formula (Simplified Formula for Proportions)
If the behavior of the population is not certain or the researcher is not familiar
with the population’s behavior, Yaro Yamen’s formula (1980) or Taro Yamane’s
formula (1967) may be used. The formula is:

𝑁
𝑛=
1 + 𝑁𝑒 2

Where N is the population size and e is the margin of error.

Example 78: 41% of Jacksonville residents said that they had been in a hurricane. How
many adults should be surveyed to estimate the true proportion of adults who have been
in a hurricane, with a 95%confidence interval and 3% margin of error.
Solution:
41 % is a past estimate of population proportion. Unknown population size. Hence,
we use the following formula.
𝛼 =0.05
p=0.41
q=1-0.41=0.59
𝑍𝛼 = 1.96
2
2
( 𝒁𝜶 ) 𝑝𝑔
𝟐
𝑛≥
𝑒2

( 𝟏. 𝟗𝟔)2 (0.41)(0.59)
𝑛≥
(0.03)2
𝑛 ≥ 1,032.54 ≈ 1.033

Example 79: From a population of 10,000 individuals of a certain town, what sample size
is needed in order to get an accurate result for a certain study using a margin of error of
3% .
Solution:
𝑁
𝑛=
1 + 𝑁𝑒 2
10,000
𝑛=
1 + (10,000)(0.03)2
𝑛 = 1,000

Hence, the sample size needed in order to get an accurate result for a a certain
study using a margin of error of 3% is 1000 individuals.

Property of and for the exclusive use of SLU. Reproduction, storing in a retrieval system, distributing, uploading or posting online, or transmitting in any form or by any
means, electronic, mechanical, photocopying, recording, or otherwise of any part of this document, without the prior written permission of SLU, is strictly prohibited. 92
D. Sampling Techniques

Sampling is the process of selecting units, like people, organizations, or objects from
a population of interest in order to study and fairly generalize the results back to the
population from which the sample was taken. The two types of sampling are
1. Random Sampling Techniques
Members from the population are selected in such a way that each individual
member in the population has an equal chance of being selected.
a. Simple Random Sampling . Every case in the population being sampled has an equal
chance of being chosen. It is an equal probability sampling Method (EPSEM).
Basic Steps:
1. Make a list of the population units and number them from a 1 to N,
where N is the population size.
2. Select n random numbers from 1 to N using some random process.
3. Employ any of the following selection procedure:
 Draw lots
 Lottery
 Usage of gadgets like the calculator or computer to generate
Random Numbers
 Table of Random Numbers
b. Systematic Random Sampling. we select some starting point randomly and then
select every kth (such as every 50th) element in the population until the desired
sample size is achieved.
Basic Steps:
1. Construct the sampling frame
2. Determine the sample size
𝑁
3. Determine the sample interval, k: 𝑘 = 𝑛
4. Identify the random start using SRS, r: 1 ≤ 𝑟 ≤ 𝑘
5. Commencing on the random start, select every kth item until the
desired sample size is reached.

c. Stratified Random Sampling. We subdivide the population into at least two different
subgroups (or strata) so that subjects within the same subgroup share the same
characteristics (such as gender or age bracket), then we draw a sample from each
subgroup (or stratum).

Property of and for the exclusive use of SLU. Reproduction, storing in a retrieval system, distributing, uploading or posting online, or transmitting in any form or by any
means, electronic, mechanical, photocopying, recording, or otherwise of any part of this document, without the prior written permission of SLU, is strictly prohibited. 93
We use Proportional Allocation to draw a sample from each stratum to
reach the desired sample size.

Example 80: Suppose a school has five departments composed of the following number
of students. Determine the number of students to be part of the sample when the
researcher needs 363 respondents.
Solution:

Department 𝑁ℎ 𝑛ℎ
Business Administration (BA) 1,500 140
Management(M) 1,200 112
Finance(F) 850 80
Entrepreneurship(E) 200 19
Culinary Arts(CA) 150 14
Total 3,900

1,500
𝑛𝐴𝐵 = (363) = 139.62 ≈ 140
3,900
1,200
𝑛𝑀 = (363) = 111.69 ≈ 112
3,900
850
𝑛𝐹 = (363) = 79.12 ≈ 80
3,900
200
𝑛𝐸 = (363) = 18.62 ≈ 19
3,900
150
𝑛𝐶𝐴 = (363) = 13.96 ≈ 14
3,900

Hence, 140 students from Business Administration, 112 students from


Management, 80 students from Finance, 19 students from Entrepreneurship, and 14
students from Culinary Arts are part of the sample.

Property of and for the exclusive use of SLU. Reproduction, storing in a retrieval system, distributing, uploading or posting online, or transmitting in any form or by any
means, electronic, mechanical, photocopying, recording, or otherwise of any part of this document, without the prior written permission of SLU, is strictly prohibited. 94
Sample Size Round-off
Rule: When the calculated sample size is not a whole number, it should be
rounded up to the next higher whole number. Rounding up a sample size calculation
for conservativeness ensures that your sample size will always be representative of the
population. For instance, A sample size calculation determined that 2006.083 data
points were necessary to represent the population. In this case, 2007 data points
samples should be taken.

d. Cluster Random Sampling. Divide the population into sections (or clusters), then
randomly select some of those clusters, and then choose all members from those
selected clusters.

e. Multi-Stage Sampling. This method uses several stages or phases in getting random
samples from the general population.
Commonly used if research is of National Scope.
 We divide the country to Regions
 Regions to Municipalities and Cities
 Municipalities and Cities to barangays
 Barangays to Sitios or sections

2. Non Random Sampling Techniques

a. Accidental or Haphazard or Convenience sampling. It is one of the most common


methods of sampling where methods done are normally biased since the researcher
considers his/her convenience in the collection of the data.
b. Purposive sampling. It is based on certain criteria laid down by the researcher. People
who satisfy the criteria are interviewed. The sub- categories of purposive sampling are:
1. Modal Instance Sampling. When we do modal instance sampling, we are
sampling most frequent cases. The problem with modal instance sampling is
identifying the “modal” case.

Property of and for the exclusive use of SLU. Reproduction, storing in a retrieval system, distributing, uploading or posting online, or transmitting in any form or by any
means, electronic, mechanical, photocopying, recording, or otherwise of any part of this document, without the prior written permission of SLU, is strictly prohibited. 95
2. Expert Sampling. It involves the assembling of a sample of persons with known
or demonstrable experience and expertise in some area.
3. Quota Sampling. Selecting items non randomly according to some fixed
quota.
4. Snowball Sampling. Begin by identifying someone who meets the criteria for
inclusion in your study. You ask them to recommend others who they may know
who also meet the criteria.

E. Statistical Data
It is the raw materials of research or any statistical investigations usually obtained by
counting or measuring items. Statistical data are usually obtained by counting or measuring
items. Data are categorized

1. According to Description:
a) Qualitative (Categorical) Data generally described by words or letters. They are not
as widely used as quantitative data because many numerical techniques do not
apply to the qualitative data. For example, it does not make sense to find an average
hair color and other attributes of the population.
 The gender (male, female) of survey respondents
 The numbers 24, 28, 17, 54, and 31 sewn on the shirts of the
basketball team are categorical data. These numbers are
substitutes for names. They do not count or measure anything.
Qualitative data can be separated into two subgroups:
1. Dichotomic takes the form of a word with two options, such as gender -
male or female.
2. Polynomic takes the form of a word with more than two options, such as
education - primary school, secondary school and university.
b) Quantitative (Numerical) Data are always numbers and are the result of counting
or measuring attributes of a population.
 The ages (in years) of survey respondents
 distance traveled
 number of children in a family,
Quantitative data can be separated into two subgroups:
1. Discrete is the result of counting. It is expressed as whole numbers and is
always exact.
 The numbers of eggs that hens lay are discrete data because
they represent counts.
 The number of students of a given ethnic group in a class.
 The number of books on a shelf.

Property of and for the exclusive use of SLU. Reproduction, storing in a retrieval system, distributing, uploading or posting online, or transmitting in any form or by any
means, electronic, mechanical, photocopying, recording, or otherwise of any part of this document, without the prior written permission of SLU, is strictly prohibited. 96
2. Continuous is the result of measuring. It is not necessarily whole numbers.
 The amounts of milk from cows are continuous data because
they are measurements that can assume any value over a
continuous span. During a year, a
a cow might yield an amount of milk that can be any value
between 0 and 7000 liters. It would be possible to get
5678.1234 liters because the cow is not restricted to the
discrete amounts of 0, 1, 2, . . . , 7000 liters.
 distance traveled
 weight of luggage

2. According to Source:
a. Primary data refers to the information which is gathered directly from an original
source or which are based on direct or first- hand experience using methods like
surveys, interviews, or experiments.
b. Secondary data refers to the information taken from published / unpublished
materials that have been previously gathered by other individuals, researcher’s or
agencies.

3. According to level of measurement


a. Nominal Scale. It involves categorizing cases according to the presence or absence
of some attribute. It is generally used for the purpose of classification. Data gathered
from variables measured at a nominal level can be categorized but cannot be
ranked, as there are no quantitative differences between and among them.
 gender
 religious affiliation
 eye color
b. Ordinal Scale. It is the simplest scale which orders people, objects, or events along
some continuum. Values of variables measured at the ordinal level offer at least a
rough indication of quantitative differences; they can also be categorized and
ranked, numbers are used only to place objects in order.
 year level
 job position
c. Interval Scale. It is the scale on which zero is arbitrary. It does not reflect the absence
of an attribute. Data gathered from variables measured at an interval scale can be
categorized, ranked, and can be added or subtracted.
 IQ Scores
 temperature
d. Ratio Scale. It possesses all of the characteristics of interval scales but has a true zero
point. Thus, a case where 0 is on a scale indicates the total absence of the property
being measured. For values at this level, differences and ratios are both meaningful.

Property of and for the exclusive use of SLU. Reproduction, storing in a retrieval system, distributing, uploading or posting online, or transmitting in any form or by any
means, electronic, mechanical, photocopying, recording, or otherwise of any part of this document, without the prior written permission of SLU, is strictly prohibited. 97
 Distances (in km) traveled by cars (0 km represents no distance traveled, and
400 km is twice as far as 200 km.)
 Prices of books(P0.00 does represent no cost, and a P300.00 book does cost
twice as much as a P150.00 book.)
 height
 weight

UNIT 2: METHODS OF DATA GATHERING AND PRESENTATION


PowerPoint 20: Methods of Data Gathering

A. Methods of Data Gathering


1. Interview (Direct) Method – a method of person-to-person exchange between the
interviewer and the interviewee.
Positive:
1. It provides consistent and more precise information since clarification
may be given by the interviewee.
2. Questions may be repeated or maybe modified to suit the interviewee’s
level of understanding.
Negative:
1. Time-consuming
2. Expensive
3. Limited field coverage
2. Questionnaire (Indirect) Method – in this method written responses are given to
prepared questions. A questionnaire is used to elicit answers to the problems of the
study. Questionnaires may be mailed or hand-carried.
Positive:
1. Inexpensive
2. Can cover a wide area in a shorter span of time.
3. Respondents may feel a greater sense of freedom to express views and
opinions because their anonymity is maintained.
Negative:
1. There’s a strong possibility of non-response, especially when
questionnaires are mailed.
2. Questions not easily understood may not be answered.
3. Observation Method – the investigator observes the behavior of the subject
/respondent. It is used when the subjects cannot talk or write.
Positive:
 The recording of behavior at the appropriate time and situation is
made possible.

Property of and for the exclusive use of SLU. Reproduction, storing in a retrieval system, distributing, uploading or posting online, or transmitting in any form or by any
means, electronic, mechanical, photocopying, recording, or otherwise of any part of this document, without the prior written permission of SLU, is strictly prohibited. 98
4. Experiment Method - this method is used when the objective is to determine the
cause-and-effect relationship of certain phenomena under controlled conditions. It
is usually used by scientific researchers.
5. Registration Method – this method of gathering information is enforced by law.
 registration of births
 deaths
 vehicles
 licenses
Positive:
1. Information is kept systematized.
2. Information is always made available to the public.

Characteristics of a Good Question


1. A good question is unbiased.
2. Questions must not be worded in a manner that influences the answer of a
respondent in a certain way, that is, to favor a certain response or be against it.
3. An unbiased question is stated in neutral language and there is no element of
pressure
4. A good question must be clear and simply stated.
5. It is easier to understand and a question that is simple and clear and is more likely to
be answered truthfully.
6. Questions must be precise
7. Questions must not be vague. The question should indicate clearly the manner on
how the answers must be given.
8. Good questionnaires lend themselves to easy analyses.

B. Methods of Data Presentation


PowerPoint 21: Methods of Presentation of Data

1. Textual Presentation – This type of presentation incorporates data in a set of narrative


sentences or paragraphs. It emphasizes and compares important figures. However, it
can be tedious to read especially if it consists of lengthy paragraphs and some figures
or words are repeated many times.

Property of and for the exclusive use of SLU. Reproduction, storing in a retrieval system, distributing, uploading or posting online, or transmitting in any form or by any
means, electronic, mechanical, photocopying, recording, or otherwise of any part of this document, without the prior written permission of SLU, is strictly prohibited. 99
2. Tabular Presentation – This is a systematic way of categorizing related data in rows
and columns. This methodical arrangement called statistical table presents data in a
more concise and greater detail than in textual or graphical form.

3. Graphical Method – This is a method of presenting quantitative data in pictorial form


produces a device which is often referred to as graph or chart. They have visual
appeal that can attract better and hold further, the reader’s interests.
Kinds of Graphs and Diagrams
1. Bar Graph. A bar graph uses bars of equal width to show frequencies of
categories of qualitative data. The vertical scale represents frequencies or
relative frequencies. The horizontal scale identifies the different categories of
qualitative data. It is best used for large changes over time/category.

2. Frequency Polygon. One type of statistical graph involves the class midpoints.
A frequency polygon uses line segments connected to points located directly
above class midpoint values. A variation of the basic frequency polygon is the
relative frequency polygon, which uses relative frequencies (proportions or
percentages) for the vertical scale. When trying to compare two data sets, it
is often very helpful to graph two relative frequency polygons on the same
axes.

Property of and for the exclusive use of SLU. Reproduction, storing in a retrieval system, distributing, uploading or posting online, or transmitting in any form or by any
means, electronic, mechanical, photocopying, recording, or otherwise of any part of this document, without the prior written permission of SLU, is strictly prohibited. 100
3. Ogive. Another type of statistical graph called an ogive (pronounced “oh-
jive”) involves cumulative frequencies. Ogives are useful for determining the
number of values below some particular value, as illustrated in Example 3. An
ogive is a line graph that depicts cumulative frequencies. An ogive uses class
boundaries along the horizontal scale, and cumulative frequencies along the
vertical scale.

4. Pie chart is a graph that depicts qualitative data as slices of a circle, in which
the size of each slice is proportional to the frequency count for the category.

Property of and for the exclusive use of SLU. Reproduction, storing in a retrieval system, distributing, uploading or posting online, or transmitting in any form or by any
means, electronic, mechanical, photocopying, recording, or otherwise of any part of this document, without the prior written permission of SLU, is strictly prohibited. 101
5. A stemplot (or stem-and-leaf plot) represents quantitative data by separating
each value into two parts: the stem (such as the leftmost digit) and the leaf
(such as the rightmost digit).

UNIT 3: DESCRIPTIVE STATISTICAL MEASURES

A. Measures of Central Tendencies


PowerPoint 22: Measures of Central Tendencies and other Locations

Numerical values that tend to locate in some sense the middle of a set of data
when arranged in increasing or decreasing order. The term average is often
associated with these measures mean, median, mode, midrange

1. Mean 𝝁 or x
a. Arithmetic Mean. It is obtained by adding all the observations and dividing the sum
by the number of observations, thus it is called computational average.
1. Population Mean: If 𝑥1 , 𝑥2 , ..., 𝑥𝑛 represents a finite population of size N,
the population mean is given by

2. Sample Mean: If 𝑥1 , 𝑥2 , ..., 𝑥𝑛 represents a finite sample of size n, the


sample mean is given by

Example 81: Suppose you chose ten people who entered the campus and whose ages
are as follows: 15 25 18 20 25 18 18 20 20 25 What is the mean age of this sample?
Solution:

The mean age of the sample is 20.40 .

Property of and for the exclusive use of SLU. Reproduction, storing in a retrieval system, distributing, uploading or posting online, or transmitting in any form or by any
means, electronic, mechanical, photocopying, recording, or otherwise of any part of this document, without the prior written permission of SLU, is strictly prohibited. 102
b. Weighted Mean. If the data values 𝑥1 , 𝑥2 , ..., 𝑥𝑘 have assigned weights 𝑤1 , 𝑤2 , ..., 𝑤𝑘 ,
respectively, the mean is given by

Example 82: :A student was taking 5 subjects last semester. Find his average if his final
grades were as follows:

Solution:

3(1.75) + 5(2.50) + 3(2.25) + 2(1.50) + 4(3.0)


̅=
𝒙 = 2.32
3+5+3+2+4

Characteristics of Mean
1. Interval and ratio measurements
2. All the scores or measurements are considered in the computation of the mean.
3. Very high or very low scores or measurements affect the mean.

2. Mode 𝝁̂ or ̂𝒙
It is the value in the distribution with the highest frequency. It locates the point
where the observation values occur with the greatest density. It can be used for
quantitative aw sell as qualitative data.
A data set can have one mode, more than one mode, or no mode.
 When two data values occur with the same greatest frequency, each
one is a mode and the data set is bimodal.
 When more than two data values occur with the same greatest
frequency, each is a mode and the data set is said to be multimodal.
 When no data value is repeated, we say that there is no mode.

Example 83: Observe the given ungrouped data below:


a. 1,2,3,4,5,6,7 (No Mode)
b. 15.2, 12.3, 4.6, 12.3, 6.5, 12.3, 5.5 (𝒙
̂=12.3)
c. 15,12,4,15,4,6,5 (𝒙̂=12 and 𝒙 ̂= 4)
d. 3,4,5,1,3,2,4,5,7,10 (𝒙
̂=3, 𝒙̂=4, and 𝒙 ̂= 5)

Property of and for the exclusive use of SLU. Reproduction, storing in a retrieval system, distributing, uploading or posting online, or transmitting in any form or by any
means, electronic, mechanical, photocopying, recording, or otherwise of any part of this document, without the prior written permission of SLU, is strictly prohibited. 103
Characteristics of Mode
1. It is very easy to compute but is seldom used because it is very unstable.
2. When a rough or quick estimate of a central value is wanted.
3. It is most appropriate for nominal scale as a measure of popularity.

3. Median
It is a value that divides the distribution into two equal parts (after arranging the
values in ascending or descending order). As such, it is a positional average. The median
is defined by

Example 84 During the first marking period, Nicole's math quiz scores were 90, 92, 93, 88,
95, 88, 97, 87, and 98. What was the median quiz score?
Solution:
Ordering the data from least to greatest, we get:

Since n =9 (odd),

The median quiz score is 92. (Four quiz scores were higher than 92 and four
were lower.)

Example 85 The ages of 10 college students are listed below. Find the median.
18, 24, 20, 35, 19, 23, 26, 23, 19, 20
Solution:
Ordering the data from least to greatest, we get:

Since n =10 (even),

The median age of the college students is 21.5

Characteristics of Median
1. Ordinal or ranked measurements
2. Only the middle scores or measurements are considered in the computation of the
median.

Property of and for the exclusive use of SLU. Reproduction, storing in a retrieval system, distributing, uploading or posting online, or transmitting in any form or by any
means, electronic, mechanical, photocopying, recording, or otherwise of any part of this document, without the prior written permission of SLU, is strictly prohibited. 104
3. Very high or very low scores do not affect the median.
4. When there are extreme cases, thus the distribution is markedly skewed.
5. When we desire to know whether the cases fall within the upper halves or the lower
halves of a distribution

B. Measures of Relative Location

These measures, also known as quantiles or fractiles, are values below which a specific
fraction or percentage of the observations in a given data set must fall. These are
percentiles, deciles, and quartiles

1. The Percentiles
Percentiles are values that divide a set of observations into 100 equal parts. These
values, denoted by 𝑃1 , 𝑃2 , … , 𝑃99 , are such that 1% of the data falls below 𝑃1 , 2% falls below
𝑃2 , …, and 99% falls below 𝑃99 . The 𝑘th percentile, 𝑃𝑘 (𝑘 = 1, 2, 3, … ,99), can be determined
using the following procedure:
a. Arrange the data in increasing order and compute the value of the index
𝑘
𝑖 = (100) 𝑛, where 𝑛 is the number of observations.
𝑥𝑖+𝑥𝑖+1
b. If 𝑖 is an integer, 𝑃𝑘 = . If 𝑖 is not an integer, use the rounded-up value for 𝑖
2
and take 𝑃𝑘 = 𝑥𝑖 . Note that 𝑥𝑖 here pertains to the score in the data set.

Example 86. As part of a quality-control study aimed at improving a production line, the
weights (in ounces) of 50 bars of soap are measured. The results are as follows, sorted from
smallest to largest. Find, the 43rd percentile and 10th percentile

11.6 12.6 12.7 12.8 13.1 13.3 13.6 13.7 13.8 14.1
14.3 14.3 14.6 14.8 15.1 15.2 15.6 15.6 15.7 15.8
15.8 15.9 15.9 16.1 16.2 16.2 16.3 16.4 16.5 16.5
16.5 16.6 17.0 17.1 17.3 17.3 17.4 17.4 17.4 17.6
17.7 18.1 18.3 18.3 18.3 18.5 18.5 18.8 19.2 20.3

Solution

a. 43rd Percentile
We compute the index 𝑖. Note that 𝑘=43 𝑎𝑛𝑑 𝑛=50
43
Then 𝑖 = (100) 50 = 21.5 ≈22 (𝑟𝑜𝑢𝑛𝑑 𝑢𝑝)
From the data set, 𝑥22 = 15.9
Hence, we have 𝑃43 = 𝑥22 = 15.9
Hence, 43% of the values lie below 15.9.

Property of and for the exclusive use of SLU. Reproduction, storing in a retrieval system, distributing, uploading or posting online, or transmitting in any form or by any
means, electronic, mechanical, photocopying, recording, or otherwise of any part of this document, without the prior written permission of SLU, is strictly prohibited. 105
b. 10th percentile 𝑃10

Notice that the data are already arranged in increasing order. We compute the
10
index 𝑖. Note that 𝑘 = 10 and 𝑛 = 50, so 𝑖 = ( ) 50 = 5.
100

.
𝑥5 +𝑥6
Since 𝑖 is an integer, 𝑃10 = From the data set, 𝑥5 = 13.1 and 𝑥6 = 13.3 (the fifth
2
𝑥5 +𝑥6 13.1.+13.3
and sixth values in the data set). Thus, we have 𝑃10 = = = 13.2.
2 2
This means that 10% of the bars of soap weigh less than 13.2 ounces.

2. The Deciles
Deciles are values that divide a set of observations into ten equal parts. These values,
denoted by 𝐷1 , 𝐷2 , … , 𝐷9 , are such that 10% of the data falls below 𝐷1 , 20% falls below 𝐷2 , …,
and 90% falls below 𝐷9 . The 𝑘th Decile, 𝐷𝑘 (𝑘 = 1, 2, … ,9), can be determined using the
following procedure:
a. Arrange the data in increasing order and compute the value of the index
𝑘
𝑖 = (10) 𝑛, where 𝑛 is the number of observations.
𝑥𝑖+𝑥𝑖+1
b. If 𝑖 is an integer, 𝐷𝑘 = 2
. If 𝑖 is not an integer, use the rounded-up value for 𝑖
and take 𝐷𝑘 = 𝑥𝑖 .

Example 87. Compute the 1st and 9th deciles of the data set from the previous example.

Solution

a. 9th Decile

Let us compute the index 𝑖 , given that k=9 and n=50


9
𝑖 = (10) 50 = 45
(𝑥45+𝑥46)
Since i is an integer, 𝐷9 = 2
. From the data set, 𝑥45 = 18.3 and 𝑥46 = 18.5
18.3+18.5
Thus, we have, 𝐷9 = 2
= 18.4
Hence, 90% of the bars of soap weigh less than 18.4 ounces. Also, 𝐷9 = 𝑃90

b. First Decile

1
Let us compute the index 𝑖 given that 𝑘 = 1 and 𝑛 = 50: 𝑖 = (10) 50 = 5.
𝑥5 +𝑥6
Since 𝑖 is an integer, 𝐷1 = . From the data set, 𝑥5 = 13.1 and 𝑥6 = 13.3. Thus, we
2
𝑥5 +𝑥6 13.1.+13.3
have 𝐷1 = = = 13.2.
2 2
This means that 10% of the bars of soap weigh less than 13.2 ounces.

Property of and for the exclusive use of SLU. Reproduction, storing in a retrieval system, distributing, uploading or posting online, or transmitting in any form or by any
means, electronic, mechanical, photocopying, recording, or otherwise of any part of this document, without the prior written permission of SLU, is strictly prohibited. 106
3. The Quartiles
Quartiles are values that divide a set of observations into four equal parts. These
values, denoted by 𝑄1 , 𝑄2 , and 𝑄3 , are such that 25% of the data falls below 𝑄1 , 50% falls
below 𝑄2 and 75% falls below 𝑄3 . The 𝑘th quartile, 𝑄𝑘 (𝑘 = 1, 2, 3), can be determined
using the following procedure:
a. Arrange the data in increasing order and compute the value of the index
𝑘
𝑖 = ( 4) 𝑛, where 𝑛 is the number of observations.
𝑥𝑖 +𝑥𝑖+1
c. If 𝑖 is an integer, 𝑄𝑘 = . If 𝑖 is not an integer, use the rounded-up value for 𝑖
2
and take 𝑄𝑘 = 𝑥𝑖 .

Example 88. Compute the 2nd and the 3rd quartiles of the data set from the previous
example.
Solution

a. 2nd quartile 𝑄2

2
Let us compute the index 𝑖 given that 𝑘 = 2 and 𝑛 = 50: 𝑖 = ( ) 50 = 25.
4

Since 𝑖 is an integer, 𝑄2 =
𝑥25 +𝑥26
2
. From the data set, 𝑥25 = 16.2 and 𝑥26 = 16.2.
𝑥25 +𝑥26 16.2+16.2
Thus, we have 𝑄2 = = = 16.2.
2 2
This means that 50% of the bars of soap weigh less than 16.2 ounces.
Note that the value of 𝑄2 represents the median of the data and that 𝑄2 = 𝑃50 = 𝐷5.

b. 3rd quartile, 𝑄3

3
We compute the index 𝑖. Note that 𝑘 = 3 and 𝑛 = 50: 𝑖 = (4) 50 = 37.5 ≈ 38(round up
since 37.5 is not an integer). Hence, we have 𝑄3 = 𝑥38 = 17.4.
This means that 75% of the bars of soap weigh less than 17.4 ounces. We also say that
𝑄3 = 𝑃75 .

UNIT 4: MEASURES OF VARIABILITY / DISPERSION


PowerPoint 23: Measures of Dispersion

The measures of central tendency and relative location do not by themselves give an
adequate description of the data. It is also very important for us to know how the
observations spread out from the average. The measures of variability/dispersion indicate
the extent to which individual items in a series are scattered about the average. It is used to
determine the extent of the scatter so that steps may be taken to control the existing
variation. After going through this unit, you are expected to know how to calculate

Property of and for the exclusive use of SLU. Reproduction, storing in a retrieval system, distributing, uploading or posting online, or transmitting in any form or by any
means, electronic, mechanical, photocopying, recording, or otherwise of any part of this document, without the prior written permission of SLU, is strictly prohibited. 107
descriptive measures to explain the consistency or variability of data using a scientific
calculator.

For example, let us consider the following measurements from two samples of data:

Sample A 0.97 1.00 0.89 1.03 1.11


Sample B 1.06 1.01 0.88 0.91 1.14

Both samples have the same mean 𝑥̅𝐴 = 𝑥̅ 𝐵 = 1.00. However, looking closely at the values,
the measurements for sample A are more uniform, or the values are close to each other
compared to sample B. This is what we will quantify in this unit. There are two general
classifications of the measures of variability: (1) measures of absolute dispersion and (2)
measures of relative dispersion.

Measures of Variability indicate the extent to which individual items in a series are
scattered about the average It is used to determine the extent of the scatter so that steps
may be taken to control the existing variation. General Classifications of Measures of
Variation are
A. Measures of Absolute Dispersion
B. Measures of Relative Dispersion

A. Measures of Absolute Dispersion: Expressed in the units of the observations. It cannot be


used to compare variations of two data sets when the averages of these data sets differ
or when the observations differ in units of measurement.
1. Range. It is the difference between the largest and smallest values. It gives an
idea of the spread of the data set but is affected by outliers and does not
consider all values in the data set.
𝑅𝑎𝑛𝑔𝑒 = 𝐻𝑖𝑔ℎ𝑒𝑠𝑡 𝑉𝑎𝑙𝑢𝑒 − 𝐿𝑜𝑤𝑒𝑠𝑡 𝑉𝑎𝑙𝑢𝑒
However, the range does not use the concept of deviation. It is a quick
but a rough measure of variation since it considers only the highest value and the
lowest value of the observations. It is affected by outliers but does not consider
all values in the data set. Thus, the range is not a very accurate measure of
variability.
2. Variance and Standard Deviation are the most common and useful measures of
variability. These two measures provide information about how the data vary
about the mean.
The variance 𝝈𝟐 or 𝒔𝟐 is a measure of variation which considers the
position of each observation relative to the mean of the set.

 Given a finite population 𝑋1 , 𝑋2 … 𝑋𝑁 , the population variance 𝜎 2 , which


is exact, can be calculated using any of the following formulas

Property of and for the exclusive use of SLU. Reproduction, storing in a retrieval system, distributing, uploading or posting online, or transmitting in any form or by any
means, electronic, mechanical, photocopying, recording, or otherwise of any part of this document, without the prior written permission of SLU, is strictly prohibited. 108
∑(𝑋𝑖 − 𝜇)2 𝑁 ∑ 𝑋𝑖 2 − (∑ 𝑋𝑖 )2
𝜎2 = 𝑜𝑟 𝜎 2 =
𝑁 𝑁2
The two formulas will generate the same result, but the second is
computationally more convenient because it eliminates the step of
computing the deviations from the mean. The second formula is
recommended in computing the variance since it does not require the
computation of the mean first and it also eliminates round off errors
caused by taking deviations from the mean.

 On the other hand, given a random sample 𝑥1 , 𝑥2 … 𝑥𝑛 , the sample


variance 𝑠 2 can be computed using

2
∑(𝑥𝑖 − 𝑥̅ )2 𝑛 ∑ 𝑥𝑖 2 − (∑ 𝑥𝑖 )2
2
𝑠 = 𝑜𝑟 𝑠 =
𝑛−1 𝑛(𝑛 − 1)

The standard deviation 𝝈 or s is the square root of the variance.

 The population standard deviation 𝜎 is the square root of the population


variance

𝑁 ∑ 𝑋𝑖 2 − (∑ 𝑋𝑖 )2
𝜎=√
𝑁2

 The sample standard deviation 𝑠 is the square root of the sample


variance.

𝑛 ∑ 𝑥𝑖 2 − (∑ 𝑥𝑖 )2
𝑠=√
𝑛(𝑛 − 1)

The variables in the abovementioned formulas are defined as follows:

𝜎 = population standard deviation 𝑋𝑖 𝑎𝑛𝑑 𝑥𝑖 = 𝑖 th observation


𝑠 = sample standard deviation 𝜇 = population mean
𝑥̅ = sample mean 𝑁 = population size
𝑛 = sample size

If the data are clustered around the mean, then the variance and the standard
deviation will be somewhat small. If the data are widely scattered about the mean, the
variance and the standard deviation will be somewhat large.

Property of and for the exclusive use of SLU. Reproduction, storing in a retrieval system, distributing, uploading or posting online, or transmitting in any form or by any
means, electronic, mechanical, photocopying, recording, or otherwise of any part of this document, without the prior written permission of SLU, is strictly prohibited. 109
Let us take note of the following:
1. The standard deviation unit is the same as that of the raw data, so it is preferable to
use the standard deviation as a measure of variability instead of the variance, whose
unit is the square of the unit of the raw data.
2. In the formula for 𝑠, we divide by the quantity 𝑛 − 1 (instead of 𝑛) to make the sample
variance an unbiased estimator of the population variance (an estimator is unbiased
if its average value is equal to the parameter it is estimating). Hence, it is critical to
determine if the data set is from a population or a sample because of the difference
in the formula to be used.
3. There is a big difference between (∑ 𝑥𝑖 )2 and ∑ 𝑥𝑖 2 ! The first, (∑ 𝑥𝑖 )2 , means we add up
all the 𝑥𝑖 values first, then square the sum. The second, ∑ 𝑥𝑖 2 , means we should square
each of the 𝑥𝑖 values first, then add them up! To illustrate, suppose our data set is 4,
3, 6, and 7. This gives us:
∑ 𝑥𝑖 = 4 + 3 + 6 + 7 = 20
(∑ 𝑥𝑖 )2 = (4 + 3 + 6 + 7)2 = 202 = 400
∑ 𝑥𝑖 2 = 42 + 32 + 62 + 72 = 110
See the difference there!

A scientific calculator can easily compute these quantities using the Statistics mode.
You can watch the video demonstration for the models Casio 991EX and Casio 991ES on the
following links, if you are not familiar with the statistical functions of your calculator.

https://www.loom.com/share/85e2345a662446c9bcab064213aeb381
https://www.loom.com/share/a08e8f19258148e0ba91ef945a39521a

Property of and for the exclusive use of SLU. Reproduction, storing in a retrieval system, distributing, uploading or posting online, or transmitting in any form or by any
means, electronic, mechanical, photocopying, recording, or otherwise of any part of this document, without the prior written permission of SLU, is strictly prohibited. 110
Example 89. A high school teacher at a small private school assigns trigonometry practice
problems to be worked via the net. Students must use a password to access the problems,
and the time of log-in and log-off is automatically recorded for the teacher. At the end of
the week, the teacher examines the amount of time each student spent working on the
assigned problems. The data is provided below in minutes. Find the Range, Standard
Deviation, and Variance for the above data.
15, 28 25 48 22 43 49 34 22 33 27 25 22 20 39

Solution
x 𝒙𝟐

15 225
28 784
25 625
48 2304
22 484
43 1849
49 2401
34 1156
22 484
33 1089
27 729
25 625
22 484
20 400
39 1521
452 (Total) 15160 (Total)

For the Standard deviation:

𝑛 ∑ 𝑥 𝑖 2 − (𝑥 𝑖 )2
𝑠=√
𝑛(𝑛 − 1)

15(15160) − (452)2
𝑠=√
15(15 − 1)
𝒔 = 𝟏𝟎. 𝟒𝟖𝟕𝟏𝟖

Property of and for the exclusive use of SLU. Reproduction, storing in a retrieval system, distributing, uploading or posting online, or transmitting in any form or by any
means, electronic, mechanical, photocopying, recording, or otherwise of any part of this document, without the prior written permission of SLU, is strictly prohibited. 111
For the variance:
The variance is 109.98095. The standard deviation is 10.48718. The variance is
the square of the standard deviation. 10.48718 squared is equal to 109.98095.

𝒔𝟐 = 𝟏𝟎𝟗. 𝟗𝟖𝟎𝟗𝟓

For Range:
The high score is 49; the low score is 15. Hence, the range is 49 - 15 = 34.

Uses of the Variance and Standard Deviation


1. Variances and standard deviations can be used to determine the spread of the data.
If the variance or standard deviation is large, the data are more dispersed. This
information is useful in comparing two (or more) data sets to determine which is more
(most) variable.
2. The measures of variance and standard deviation are used to determine the
consistency of a variable. For example, in the manufacture of fittings, such as nuts
and bolts, the variation in the diameters must be small, or the parts will not fit together.
3. The variance and standard deviation are used to determine the number of data
values that fall within a specified interval in a distribution.
4. Finally, the variance and standard deviation are used quite often in inferential
statistics.
5. These uses will be shown in later chapters of this textbook.

B. Measures of Relative Dispersion. It is used to compare variations in the dispersion of two


data sets when the averages of these data sets differ or when the observations differ in
units of measurement. It is unitless.
1. Coefficient of Variation. It indicates how large the standard deviation is in
relation to the mean. It can be used to compare variations for different variables
with different units. The larger the coefficient of variation, the more dispersed the
observations are.

Property of and for the exclusive use of SLU. Reproduction, storing in a retrieval system, distributing, uploading or posting online, or transmitting in any form or by any
means, electronic, mechanical, photocopying, recording, or otherwise of any part of this document, without the prior written permission of SLU, is strictly prohibited. 112
Example 90: If we have a standard deviation of 1.5 and a mean of 5, what is the coefficient
of variation?
Solution

𝑥̅ = 5
𝑠 = 1.5
1.5
𝐶𝑉 = ( ) 100% = 30 %
5

In other words, the standard deviation is 30% of the mean. When comparing two data
sets, the general rule of thumb you should follow is: The higher the coefficient of variation,
the higher the variability of the data set. This means that, when comparing two or more data
sets, the one with the highest coefficient of variability can be said to have the highest
variation.

Example 91: The mean of the number of sales of cars over a 3-month period is 87, and the
standard deviation is 5. The mean of the commissions is P261,250.00 and the standard
deviation is P38,650.00. Compare the variations of the two.
Solution

The coefficients of variation are

5
𝐶𝑉 = ( ) 100% = 5.70 %
87 Sales

P38,650.00
𝐶𝑉 = ( ) 100% = 14.80 %
P261,250.00 Commissions

Since the coefficient of variation is larger for commissions, the commissions are more
variable than sales.

UNIT 5: CORRELATION AND REGRESSION


PowerPoint 24: Correlation and Regression

A. Correlation
It measures the strength of the association or relationship between variables. the
variables are not designated as dependent or independent.
𝑐𝑜𝑟𝑟𝑒𝑙 𝑋 𝑎𝑛𝑑 𝑌=𝑐𝑜𝑟𝑟𝑒𝑙 𝑌 𝑎𝑛𝑑 𝑋
It is not define as causation (cause and effect relationship)
Assume that the association is linear, that one variable increases or decreases a fixed
amount for a unit increase or decrease in the other.

Property of and for the exclusive use of SLU. Reproduction, storing in a retrieval system, distributing, uploading or posting online, or transmitting in any form or by any
means, electronic, mechanical, photocopying, recording, or otherwise of any part of this document, without the prior written permission of SLU, is strictly prohibited. 113
Pearson Correlation Coefficient
 denoted by 𝑟
 use to measure the degree of linear association or relationship
 measured on a scale that varies from +1 through 0 𝑡𝑜 – 1
 formula is
n xy   x y
r
n x2   x
2
n y 2  y 
2
 

The value of r is interpreted as follows:

r Interpretation
1.0 Perfect positive/negative correlation
0.80-0.99 Very strong positive/ negative correlation
0.60-0.79 Strong positive/ negative correlation
0.40-0.59 Moderate positive/ negative correlation
0.20-0.39 Weak positive/ negative correlation
0.01-0.19 Very weak positive/ negative correlation
0.0 No correlation

Perfect positive Perfect negative


correlation correlation

No correlation Use other measures


of correlation

Property of and for the exclusive use of SLU. Reproduction, storing in a retrieval system, distributing, uploading or posting online, or transmitting in any form or by any
means, electronic, mechanical, photocopying, recording, or otherwise of any part of this document, without the prior written permission of SLU, is strictly prohibited. 114
Example 92: Given the following data on the number of hours of study (x) for an
examination and the scores (y)received by a random sample of 10 students, compute for
the Pearson correlation coefficient.
Solution

Student 𝒙 𝒚 ∑ 𝒙𝒚 ∑ 𝒚𝟐 ∑ 𝒙𝟐
1 8 56 448 3136 64
2 5 44 220 1936 25
3 11 79 869 6241 121
4 13 72 936 5184 169
5 10 70 700 4900 100
6 5 54 270 2916 25
7 18 94 1692 8836 324
8 15 85 1275 7225 225
9 2 33 66 1089 4
10 8 65 520 4225 64

n xy   x y
r
n x 2   x 2
n y 2  y 
2

106996  95652
r
101121  95 1045688  652
2 2

r  0.9625
There is a very strong positive linear relationship between the number of hours
of study (𝑥) for an examination and the scores (𝑦) received by a random sample of
10 students.

Property of and for the exclusive use of SLU. Reproduction, storing in a retrieval system, distributing, uploading or posting online, or transmitting in any form or by any
means, electronic, mechanical, photocopying, recording, or otherwise of any part of this document, without the prior written permission of SLU, is strictly prohibited. 115
Example 93: Consider the scores obtained in Math and Statistics by 10 students.

Student 1 2 3 4 5 6 7 8 9 10
Math 5 8 10 12 12 14 15 16 18 20
Score
Stat 2 7 8 9 10 12 14 10 16 12
Score
Solution

Student 1 2 3 4 5 6 7 8 9 10 Total
Math Score 5 8 10 12 12 14 15 16 18 20 130
(𝒙)
Stat Score 2 7 8 9 10 12 14 10 16 12 100
(𝒚)
𝒙𝒚 10 56 80 108 120 168 210 160 288 240 1440

𝒙𝟐 25 64 100 144 144 196 225 256 324 400 1878

𝒚𝟐 4 49 64 81 100 144 196 100 256 144 1138

101440  130100
r
101878  130 101138  100
2 2

r  0.8692

There is a very strong positive linear relationship between math and stat scores.

Property of and for the exclusive use of SLU. Reproduction, storing in a retrieval system, distributing, uploading or posting online, or transmitting in any form or by any
means, electronic, mechanical, photocopying, recording, or otherwise of any part of this document, without the prior written permission of SLU, is strictly prohibited. 116
B. Regression
It is used to examine the relationship between one dependent and one independent
variable and to predict the dependent variable (Y) when the independent variable (X) is
known.
It finds the best line (regression line) that predicts Y from X.
The Regression Line
It is a line that is as close as possible to all the data points at once.
The Regression Equation
It is an equation that represents the relationship between one dependent and
one independent variable.
𝒚= 𝒂 + 𝒃𝒙
The slope is
n xy   x y
b
n x2   x
2
 
The y-intercept

a
 y  b  x 
n  n 
 

Coefficient of Determination (𝑹𝟐 ). It is the square of the correlation coefficient. It is


interpreted as the proportion of the variance in the dependent variable that is predictable
from the independent variable. The fraction of data points falls on the regression line.
𝑹𝟐 =1 (all points lie exactly on a straight line with no points scattered about the line) means
that the dependent variable is perfectly predicted without error using the independent
variable X
𝑹𝟐 =0 means that the dependent variable cannot be predicted using the independent
variable X.
An 𝑹𝟐 between 0 and 1 indicates the extent to which the dependent variable is
predictable.
An 𝑹𝟐 of 0.10 means that 10 percent of the variance in Y is predictable from X;
an 𝑹𝟐 of 0.20 means that 20 percent is predictable; and so on.
SSR
R2   100
SSY
Where: SSR  b1 SPXY

SPXY   xi y i 
 x  y  i i

SSY   y i
2

 y  i
2

SSX   x i
2

 x  i
2

Property of and for the exclusive use of SLU. Reproduction, storing in a retrieval system, distributing, uploading or posting online, or transmitting in any form or by any
means, electronic, mechanical, photocopying, recording, or otherwise of any part of this document, without the prior written permission of SLU, is strictly prohibited. 117
Example 94: The paired data below consist of the costs of advertising (in thousands
of pesos) and the number of products sold (in thousand units).
Solution

Cost # Products  xy x 2
y 2

(x) Sold
(y)
9,000.00 85,000.00 765,000,000.00 81,000,000.00 7,225,000,000.00
2,000.00 52,000.00 104,000,000.00 4,000,000.00 2,704,000,000.00
3,000.00 55,000.00 165,000,000.00 9,000,000.00 3,025,000,000.00
4,000.00 68,000.00 272,000,000.00 16,000,000.00 4,624,000,000.00
2,000.00 67,000.00 134,000,000.00 4,000,000.00 4,489,000,000.00
5,000.00 86,000.00 430,000,000.00 25,000,000.00 7,396,000,000.00
9,000.00 83,000.00 747,000,000.00 81,000,000.00 6,889,000,000.00
10,000.00 73,000.00 730,000,000.00 100,000,000.00 5,329,000,000.00
Total 44,000.00 569,000.00 3,347,000,000.00 320,000,000.00 41,681,000,000.00

1. Plot a scatter diagram

Property of and for the exclusive use of SLU. Reproduction, storing in a retrieval system, distributing, uploading or posting online, or transmitting in any form or by any
means, electronic, mechanical, photocopying, recording, or otherwise of any part of this document, without the prior written permission of SLU, is strictly prohibited. 118
2. Find the equation of the regression line to predict weekly sales from advertising
expenditures.

Thus, the equation is 𝑦 = 55788.25 + 2.7885𝑥

3. Estimate the number of products sold when advertising costs is P4,500.


𝑦=55788.25+2.7885𝑥
𝑦=𝑛𝑢𝑚𝑏𝑒𝑟 𝑜𝑓 𝑝𝑟𝑜𝑑𝑢𝑐𝑡 𝑠𝑜𝑙𝑑=55788.25+2.7885(4,500)
𝑦=𝑛𝑢𝑚𝑏𝑒𝑟 𝑜𝑓 𝑝𝑟𝑜𝑑𝑢𝑐𝑡 𝑠𝑜𝑙𝑑=68,336.50 units

4. Determine the coefficient of determination

Therefore, 50.08 % of the variance in the number of products sold is predictable from
the cost of advertising.

Property of and for the exclusive use of SLU. Reproduction, storing in a retrieval system, distributing, uploading or posting online, or transmitting in any form or by any
means, electronic, mechanical, photocopying, recording, or otherwise of any part of this document, without the prior written permission of SLU, is strictly prohibited. 119
Learning Reinforcement 6

Directions: Write your solutions and answers on a clean sheet of


paper, or you may print this page and answer there. Submit the
image of your HANDWRITTEN SOLUTIONS as a single pdf file in the
submission bin for this activity in the Classroom. You may use
image scanning apps on your phone (CamScanner or Tap
Scanner) to save several images into one pdf file, or place your
images in a document and save them as a pdf file.

A. Classify the following statements as to whether they belong to the area of


descriptive statistics or inferential statistics
___________________ 1. At most 5% of SLU students are smokers.
___________________ 2. Assuming that less than 20% of the Kalinga coffee beans were
destroyed by a typhoon these past months, we should expect an
increase of no more than P30 for a kilogram of coffee by the end
of the year.
___________________ 3. An employee generalized that the average monthly salary of a
regular employee in a certain company is P12,000.
___________________ 4. A study found out that all customers who have received a gift
certificate from a store 75% went back to the store to shop.
___________________ 5. The average grade in statistics of 50 students is 83.60.
B. At what level are the following variables measured?
_______________ 1. The scores of students in a statistics quiz
_______________ 2. The birth order of children in the family
_______________ 3. Weights of a sample of bags of raw materials for the
production of a certain product, measured in grams.
_______________ 4. The natural eye color of a sample of 100 children.
_______________ 5. The final grade of graduate students taking up Statistics.
C. Classify the following variables as quantitative or qualitative variables. If the variable
is quantitative, identify whether it is discrete or continuous.
_______________ _______________ 1. The type of payment used by customers
_______________ _______________ 2. The evaluation rating of instructors
_______________ _______________ 3. The classification of employees in a
company
_______________ _______________ 4. The weekly allowance of students
_______________ _______________ 5. The length of telephone calls made by
students to their parents

Property of and for the exclusive use of SLU. Reproduction, storing in a retrieval system, distributing, uploading or posting online, or transmitting in any form or by any
means, electronic, mechanical, photocopying, recording, or otherwise of any part of this document, without the prior written permission of SLU, is strictly prohibited. 120
D. In each of the following situations, a random sample must be obtained. Determine
whether a cluster, stratified, or systematic random sampling would be appropriate.
Explain in detail how the sampling is to be conducted. Do not discuss expected results
or conclusions.
1. A large convenience store chain wishes to determine its customers’ level of
satisfaction with regard to their service.
2. A nationwide survey on charter change is to be conducted. (Note: there are
seventeen regions in the Philippines.)
3. An educational researcher wants to compare the difference in career goals
between male and female students of Otto Hahn University who has 10,000
students.
4. A social researcher wants to determine whether electronic engineers who
work in the communications field earn more than those who are in
semiconductor industries.
5. A market analyst would like to compare the durability, in terms of mean time
before wear, of two leading brands of car tires.
E. The numbers of incorrect answers on a true or false competency test for a random
sample of 15 students were recorded as follows: 2, 1, 3, 0, 1, 3, 6, 0, 3, 3, 5, 2, 1, 4, and
2. Find a. mean, b. median, c. mode, d. range, e. standard deviation, f. variance
F. A corporation administers an aptitude test to all new sales representatives.
Management is interested in the extent to which this test is able to predict their
eventual success. The accompanying table records average weekly sales (in
thousands of pesos) and aptitude test scores for a random sample of eight
representatives.

1. Plot a scatter diagram


2. Estimate the linear regression of weekly sales on aptitude test scores.
3. Estimate the weekly sales when test scores is 70
4. Determine the coefficient of determination

Well done! It’s time to answer Quiz 3!

Property of and for the exclusive use of SLU. Reproduction, storing in a retrieval system, distributing, uploading or posting online, or transmitting in any form or by any
means, electronic, mechanical, photocopying, recording, or otherwise of any part of this document, without the prior written permission of SLU, is strictly prohibited. 121
MODULE 7: MATHEMATICS OF GRAPHS

Our target learning outcomes are: a) Characterize and construct different kinds of
graphs; b) Differentiate Eulerian graphs and Hamiltonian graphs; c) Use mathematical
concepts and tools in other areas such as networks, circuits and systems.

A. BASIC CONCEPTS

Graph Theory is a field of mathematics that started hundreds of years ago to help
solve problems such as finding the shortest path between two points passing through several
other points. Now, elements of graph theory are used extensively in many fields like gaming
theory, biology and social networking.
A graph is a collection of vertices (also called points or nodes) and edges (or lines).
The graph 𝐺 is denoted by 𝐺 = (𝑉(𝐺), 𝐸 (𝐺 )) where the set 𝑉(𝐺 ) is called the vertex set of 𝐺
and the set 𝐸(𝐺) is called the edge set of 𝐺.

Example 95:
G: 𝐺 = (𝑉(𝐺), 𝐸 (𝐺 ))
where 𝑉(𝐺) = {𝑎, 𝑏, 𝑐, 𝑑, 𝑒 }
𝐸(𝐺) = { 𝑎𝑏, 𝑐𝑑, 𝑏𝑒, 𝑎𝑒}

The number of vertices in a graph G is called its order (denoted by p) and the number
of edges on a graph G is called its size (denoted by q).

Example 96:
G:

Order of G: p = 5
Size of G: q = 6

Property of and for the exclusive use of SLU. Reproduction, storing in a retrieval system, distributing, uploading or posting online, or transmitting in any form or by any
means, electronic, mechanical, photocopying, recording, or otherwise of any part of this document, without the prior written permission of SLU, is strictly prohibited. 122
The ends of an edge are said to be incident with the edge, and vice versa. Two
vertices which are incident with a common edge are adjacent as are two edges which are
incident with a common vertex.

Example 97:
G:

v1 is incident with edge v1v4


v3 and v2 are adjacent

An edge with identical ends is called a loop. An edge with distinct ends is called a
link. A trivial graph is a graph with just one vertex and no edges. All other graphs are called
nontrivial.

Example 98:

G1: G2:

●a

G1 is a nontrivial graph
while G2 is a trivial graph.
w1w2 is a link while
w3w3 is a loop

A simple graph is a graph with no loops and no two of its links join the same pair of
vertices.
Example 99:

G3:
G3 is a simple
graph.

Property of and for the exclusive use of SLU. Reproduction, storing in a retrieval system, distributing, uploading or posting online, or transmitting in any form or by any
means, electronic, mechanical, photocopying, recording, or otherwise of any part of this document, without the prior written permission of SLU, is strictly prohibited. 123
A complete graph is a simple graph in which each pair of distinct vertices is joined
by an edge. It is denoted by Kn. An empty graph is a graph with no edges.

Example 100 Example 6:

G4: G5 : ● ●
● ●

G4 is a complete graph with 6 vertices denoted as K6 while G5 is an empty graph.

The degree of a vertex v is the number of edges of G incident with v, each loop
counted as two edges. This is denoted by 𝑑(v).

Example 101:

a b
𝑑(a)=3
𝑑(f)=5
c d 𝑑(c)=4

g
f e

THEOREM 1
The sum of the degrees of all the vertices in a graph is equal to twice the number of edges
of the graph.
∑ 𝑑(𝑣) = 2𝑞
𝑣∈𝑉(𝐺)

A graph G is k-regular if 𝑑(v)=k for all v ∈ 𝑉(𝐺 ).

Property of and for the exclusive use of SLU. Reproduction, storing in a retrieval system, distributing, uploading or posting online, or transmitting in any form or by any
means, electronic, mechanical, photocopying, recording, or otherwise of any part of this document, without the prior written permission of SLU, is strictly prohibited. 124
Example 102:

2-regular graph

A walk in a graph G is an alternating sequence W: v0-e1-v1-e2-v2-…-vn-1-en-vn (n ≥ 0)


of vertices and edges, beginning and ending with vertices. The length of a walk is the
number of edges it encounters. A walk of length 0 is called a trivial walk.

Example 103:

WALK: v1-1-v2-3-v3-3-v2-5-v5

There are special kinds of walks – a trail and a path. A trail is a walk in which no edge
is repeated. A path is a trail in which no vertex is repeated.

Example 104:

In the graph, v1-1-v2-5-v5-10-v6-9-v5 is a trail and v1-1-v2-5-v5-10-v6 is a path.

Property of and for the exclusive use of SLU. Reproduction, storing in a retrieval system, distributing, uploading or posting online, or transmitting in any form or by any
means, electronic, mechanical, photocopying, recording, or otherwise of any part of this document, without the prior written permission of SLU, is strictly prohibited. 125
Two vertices u and v of G are said to be connected if there is a (u,v)-path in G. The
graph G is connected if every pair of vertices is connected. Otherwise, the graph is
disconnected.

Example 105:
Connected graph Disconnected graph

A circuit is a trail that begins and ends in the same vertex. That is, a circuit is a nontrivial
closed trail (no repeated edges). A cycle is a path that begins and ends in the same vertex.
That is, a cycle is a closed path (no repeated edges and vertices, except the first and last).
A cycle of length n is referred to as an n-cycle. A circuit and a cycle are identified using a
sequence of vertices v0-v1-…-vn.

Example 106: A

B E

C D
A-C-B-A is a cycle of length 3 in the graph shown above. A-B-C-D-E-A is another
cycle, this time of length 5. An example of a circuit is A-B-C-D-B-E-A.

B. EULER GRAPHS
During the 18th century in the Prussian city of Königsberg, a river ran through the four
land masses of the city and seven bridges crossed the forks of the river. The river and the
bridges are shown below. As an amusement, townsfolk would see if they could find a route
that would take them across every bridge once and return them to where they started.
Leonard Euler, one of the most prolific mathematicians ever, was able to solve this
problem in 1735, by introducing edges representing the bridges and vertices representing
the land masses. This laid the foundation for graph theory as a field in mathematics.

Property of and for the exclusive use of SLU. Reproduction, storing in a retrieval system, distributing, uploading or posting online, or transmitting in any form or by any
means, electronic, mechanical, photocopying, recording, or otherwise of any part of this document, without the prior written permission of SLU, is strictly prohibited. 126
7 BRIDGES OF KÖNIGSBERG

EULER TOURS
A trail that traverses every edge of G is called Euler trail of G. A tour of G is a closed
walk that traverses each edge of G at least once. An Euler tour is a tour which traverses
each edge exactly once (a closed Euler trail). A graph is Eulerian or an Euler graph if it
contains an Euler tour.

THEOREM 2
A nonempty connected graph is Eulerian if and only if it has no vertices of odd degree.

COROLLARY
A connected graph has an Euler trail if and only if it has zero or two vertices of odd degree.

Example 107:

G1 G2

The trail d-a-b-c-d-b of G1 is an Euler trail. G1 has a tour d-a-b-c-d-b-d but it is not
an Euler tour. G2 has an Euler tour B-A-C-E-D-C-B hence is an Euler graph.

Property of and for the exclusive use of SLU. Reproduction, storing in a retrieval system, distributing, uploading or posting online, or transmitting in any form or by any
means, electronic, mechanical, photocopying, recording, or otherwise of any part of this document, without the prior written permission of SLU, is strictly prohibited. 127
C. HAMILTONIAN GRAPHS
A path that contains every vertex of G is called a Hamilton path of G. Note that in
Hamilton path, the edges of the graph may or may not all be covered since edges and
vertices must not repeat. A Hamilton cycle of G is a cycle that contains every vertex of G.
A graph is Hamiltonian if it contains a Hamilton cycle.

Example 108:

The path a-b-c-d of G2 is a Hamilton path.


The cycle a-b-c-d-e-a of G1 is a Hamilton cycle thus G1 is a Hamiltonian graph.
But G2 and G3 are not Hamiltonian graphs.

Note that (1) any Hamilton cycle can be converted to a Hamilton path by removing
one of its edges; (2) every graph that contains a Hamiltonian cycle also contains a
Hamiltonian path but the converse is not true; (3) there may exist more than one Hamilton
path and more than one Hamilton cycle in a graph.

Property of and for the exclusive use of SLU. Reproduction, storing in a retrieval system, distributing, uploading or posting online, or transmitting in any form or by any
means, electronic, mechanical, photocopying, recording, or otherwise of any part of this document, without the prior written permission of SLU, is strictly prohibited. 128
Learning Reinforcement 7

Directions: Write your solutions and answers on a clean sheet of


paper, or you may print this page and answer there. Submit the
image of your HANDWRITTEN SOLUTIONS as a single pdf file in the
submission bin for this activity in the Classroom. You may use
image scanning apps on your phone (CamScanner or Tap
Scanner) to save several images into one pdf file, or place your
images in a document and save them as a pdf file.

1. Graph G shown below:


a b

c y dd
x c e

f g

Identify the following:


a. V(G)
b. vertices incident to x
c. edges incident to a
d. vertices adjacent to d
e. edges adjacent to y
f. p
g. E(G)
h. shortest path from b to g

2. Is the graph an Euler graph?Why or why not?If yes, find an Euler tour.

Property of and for the exclusive use of SLU. Reproduction, storing in a retrieval system, distributing, uploading or posting online, or transmitting in any form or by any
means, electronic, mechanical, photocopying, recording, or otherwise of any part of this document, without the prior written permission of SLU, is strictly prohibited. 129
3. Given the graphs shown below, determine which graph is Hamiltonian and for such
graph, find a Hamilton cycle:

Property of and for the exclusive use of SLU. Reproduction, storing in a retrieval system, distributing, uploading or posting online, or transmitting in any form or by any
means, electronic, mechanical, photocopying, recording, or otherwise of any part of this document, without the prior written permission of SLU, is strictly prohibited. 130
MODULE 8: LINEAR PROGRAMMING

Our target learning outcomes are: a) Identify problems that linear programming can
handle; b) Write linear programming models and solve using the graphical method

For almost all the human activities there is a desire to deliver the most with the least.
For example, in the business point of view, maximum profit is desired from least investment;
maximum number of crop yield is desired with minimum investment on fertilizers; maximizing
the strength, longevity, efficiency, utilization with minimum initial investment and operational
cost of various industrial equipment and machineries. (Kulkarni et al., 2017). This is Operations
Research.
Operations Research is an important tool in making decisions and in analyzing
physical systems. In mathematical terms, it is finding the best solution from among the set of
all feasible solutions. Translating the problem definition into mathematical relationships is the
most important part of the OR study. This phase involves asking the following questions:
 What are the decision alternatives?
 Under what restrictions is the decision made?
 What is an appropriate objective criterion for evaluating alternatives?
Operations Research is a process of mathematical modelling with the objective of
finding the optimal solution for a certain problem. A generalized model in operations
research takes the following form:

Maximize or Minimize
Objective Function
subject to
Constraints

The objective function is the value which we want to be optimized. The constraints
are the limitations or conditions of the problem. A solution to the model is feasible if it satisfies
all the constraints. A solution is optimal if it is both feasible and yields the best (maximum or
minimum) value of the objective function.

A. GRAPHING LINEAR INEQUALITIES

The concept of inequalities is widely used in the study of operations research. As such,
it is important to remember how to draw the graph of a line and how to identify the region
corresponding to the inequality. The following is a simple guide on how to draw the graph of
a line:

Property of and for the exclusive use of SLU. Reproduction, storing in a retrieval system, distributing, uploading or posting online, or transmitting in any form or by any
means, electronic, mechanical, photocopying, recording, or otherwise of any part of this document, without the prior written permission of SLU, is strictly prohibited. 131
1. Identify the 𝑥 and 𝑦 intercepts of the line.
2. Draw a line connecting the intercepts.

The 𝑥 intercept of a line is found by setting the value of 𝑦 to be 0. For the 𝑦 intercept,
set the value of 𝑥 to 0. If both intercepts lie at the point 𝑥 = 0 and 𝑦 = 0, simply find another
point on the line by substituting any value of 𝑥 and solving for 𝑦.
The solution of a linear inequality consists of all the points (𝑥, 𝑦) that satisfy the
inequality. For inequalities with ≤, the corresponding region is, in most cases, the area below
the line, while for inequalities with ≥, the corresponding region is, in most cases, the area
above the line. This can be verified by choosing a point not on the line and checking if this
point satisfies the inequality or not. If it does, the shaded region will be on the side of the line
where the point belongs. Otherwise, shade the other side of the line.

Example 109: Let us graph the inequalities: 2𝑥 + 3𝑦 ≤ 12, 5𝑥 + 2𝑦 ≤ 10 and 3𝑥 ≥ 𝑦 to


illustrate the process of finding the solution of a linear inequality.
Solution:
For 2𝑥 + 3𝑦 ≤ 12, start with the linear equation 2𝑥 + 3𝑦 = 12 then solve for the x and y
intercepts:
When x = 0 in 2𝑥 + 3𝑦 = 12 then y = 4, so the 𝑦 intercept is (0, 4).
When y = 0 in 2𝑥 + 3𝑦 = 12 then x = 6, so the 𝑥 intercept is (6, 0).
Draw a line through these two points and shade the area below the line since the inequality
is ≤. To verify this, choose the point (0, 0) and substitute it to the left side of the inequality:
2𝑥 + 3𝑦 = 2(0) + 3(0) = 0
Since 0 ≤ 12, the point (0, 0) satisfies the inequality so we shade the region containing the
point (0, 0).

Solution of 2𝑥 + 3𝑦 = 12
A similar approach is done for 5𝑥 + 2𝑦 ≤ 10 and 3𝑥 ≥ 𝑦. Verify that the solution for the
inequalities is the shaded region in the figures below. The point (0, 0) can be used as a test
point for 5𝑥 + 2𝑦 ≤ 10 but not for 3𝑥 ≥ 𝑦 since it lies on the line 3𝑥 = 𝑦. Using (1, 1) as a test
point will show the correct region to be shaded.

Property of and for the exclusive use of SLU. Reproduction, storing in a retrieval system, distributing, uploading or posting online, or transmitting in any form or by any
means, electronic, mechanical, photocopying, recording, or otherwise of any part of this document, without the prior written permission of SLU, is strictly prohibited. 132
Solution of 5𝑥 + 2𝑦 ≤ 10 Solution of 3𝑥 ≥ 𝑦

B. LINEAR PROGRAMMING
One of the techniques in operations research is linear programming. A linear
programming (LP) problem is an optimization problem for which the objective function is a
linear function of the decision variables, and the values of the decision variables must satisfy
a set of constraints which are either linear equations or linear inequalities.

Formulating the LP Model


Applications of linear programming require knowledge of how to properly formulate
a model that reflects the parameters as well as the objective of a certain situation. While
there may be different techniques in formulating an OR model, the following can be used
as a guide to formulate an LP model:
Step 1. Define the decision variables.
What quantities need to be determined in the problem and what is the time frame
for these quantities? Use the variables 𝑥 and 𝑦 for a two-variable problem, and 𝑥1 , 𝑥2 , … , 𝑥𝑛 if
more than two quantities need to be determined.
Step 2. Formulate the objective function.
Determine whether the goal is to maximize or minimize, and what quantity is being
optimized. Assign this to the variable 𝑧 and define it completely. Usually, we maximize the
profit or revenue and minimize costs. Then write the equation for 𝑧, known as the objective
function. For a two-variable problem, this will be in the form 𝑧 = 𝑎𝑥 + 𝑏𝑦.
Step 3. Identify the constraints.
What are the limitations or restrictions on the values of the decision variables? These
are usually scarce resources or manpower, dietary requirements, or other similar conditions.
When writing constraints, remember the following:
 Constraints can be =, ≤, or ≥.
 Express the left-hand side as a sum of constant multiples of the decision variables. The
coefficients of the decision variables in the constraints are called technological
coefficients.
 Any constant must be on the right-hand side of the constraint.

Property of and for the exclusive use of SLU. Reproduction, storing in a retrieval system, distributing, uploading or posting online, or transmitting in any form or by any
means, electronic, mechanical, photocopying, recording, or otherwise of any part of this document, without the prior written permission of SLU, is strictly prohibited. 133
 Include nonnegativity constraints if applicable. Most decision variables are not
allowed to have negative values, but can be zero or positive.
Step 4. Write the LP model.
Summarize the results of steps 2 and 3 using the general form of an OR/LP model.

Example 110: Boysen produces enamel and acrylic paint from two particular raw materials
M1 and M2. The following data provides the data regarding the production restrictions.

Tons of Raw Materials per ton Maximum daily


of: availability (in tons)
Acrylic Paint Enamel Paint
Raw Material, M1 6 4 24
Raw Material, M2 1 2 6
Profit per ton (in millions of Php) 5 4

If the maximum daily demand for enamel paint is at 2 tons, determine the production
output that would maximize the profit.

Solution:

We formulate the LP model here first and show the solution later.

1. Define the decision variables


Let 𝑥 = number of tons of acrylic paint produced in a day
𝑦 = number of tons of enamel paint produced in a day

2. Formulate the objective function


The aim is to maximize the profit. The daily profit is based on the number of tons of
each type of paint produced in a day. Knowing that the profit for each ton of acrylic paint
produced in a day is at Php 5 million while the profit for each ton of enamel paint produced
in a day is at Php 4 million, the objective function is:
Maximize Profit, 𝑧: 𝑧 = 5𝑥 + 4𝑦
where 𝑧 is the total profit per day.

3. Identify the constraints


6𝑥 + 4𝑦 ≤ 24 Maximum availability for material M1 is at 24 tons, each ton of acrylic
paint uses 6 tons of material M1 while each ton of enamel point uses 4
tons of material M1.
𝑥 + 2𝑦 ≤ 6 Maximum availability for material M2 is at 6 tons, each ton of acrylic
paint uses 1 ton of material M2 while each ton of enamel paint uses 2
tons of material M2.

Property of and for the exclusive use of SLU. Reproduction, storing in a retrieval system, distributing, uploading or posting online, or transmitting in any form or by any
means, electronic, mechanical, photocopying, recording, or otherwise of any part of this document, without the prior written permission of SLU, is strictly prohibited. 134
𝑦≤2 Daily production for enamel paint should not exceed the daily demand
of 2 tons
𝑥 ≥ 0, 𝑦 ≥ 0 Nonnegativity constraints, since number of tons cannot be negative

4. Write the LP model


Maximize 𝑧 = 5𝑥 + 4𝑦
subject to
6𝑥 + 4𝑦 ≤ 24
𝑥 + 2𝑦 ≤ 6
𝑦≤2
𝑥 ≥ 0, 𝑦 ≥ 0

Graphical Solution of a Two-Variable LP Model


Once the LP model has been formulated, the next step would be to solve the LP
model. While more involved algorithms and/or computer applications can be used to solve
larger models, a two-variable LP model can be solved graphically. The graphical solution
part of linear programming can be summarized into two steps:

Step 1. Determination of the feasible region.


The feasible region of an LP model is the set of all points (𝑥, 𝑦) that satisfies all the
constraints in the LP model. Graphically, this is the intersection of all the solutions of the
inequalities representing the constraints, graphed simultaneously. Any other point outside
the feasible region is infeasible.

Step 2. Determination of the optimal solution from among all the points in the feasible region.
For a maximization problem, an optimal solution to an LP model is a point (𝑥, 𝑦) in the
feasible region with the largest objective function value. Similarly, for a minimization problem,
an optimal solution is a point in the feasible region with the smallest objective function value.
It can be shown that if an LP model has an optimal solution, it lies at a corner point of the
feasible region. Thus, we need to identify the coordinates of the corner points of the feasible
region and solve for their corresponding objective function value to determine which yields
the largest (or smallest) value of 𝑧 for a maximization (or minimization) problem.
Once the optimal solution is found, interpret the result. Write the final answer in complete
statement.

Property of and for the exclusive use of SLU. Reproduction, storing in a retrieval system, distributing, uploading or posting online, or transmitting in any form or by any
means, electronic, mechanical, photocopying, recording, or otherwise of any part of this document, without the prior written permission of SLU, is strictly prohibited. 135
Example 111: Solve for the optimal solution of the following OR Model:
Maximize 𝑧 =𝑥+𝑦
subject to 2𝑥 + 3𝑦 ≤ 12
5𝑥 + 2𝑦 ≤ 10
𝑥 ≥ 0, 𝑦 ≥ 0
Solution:
1. Determine the feasible region

Start by graphing the inequalities in the constraints. The red line is for 2𝑥 + 3𝑦 ≤
12, and the blue line is for 5𝑥 + 2𝑦 ≤ 10.
The feasible region is the intersection of the shaded portions for the inequalities
2𝑥 + 3𝑦 ≤ 12 and 5𝑥 + 2𝑦 ≤ 10 that lies in the first quadrant. We only considered
quadrant 1 because of the nonnegativity constraints 𝑥 ≥ 0 and 𝑦 ≥ 0.

Property of and for the exclusive use of SLU. Reproduction, storing in a retrieval system, distributing, uploading or posting online, or transmitting in any form or by any
means, electronic, mechanical, photocopying, recording, or otherwise of any part of this document, without the prior written permission of SLU, is strictly prohibited. 136
2. Determine the optimal solution
Start by identifying the corner points of the feasible region. Points of intersection of
two lines can be determined by algebraically solving a system of linear equations
corresponding to the intersecting lines.

The corner points of the feasible region are (0, 0), (0, 4), (6/11, 40/11) and (2, 0). The
point (6/11, 40/11) is obtained by solving the system of equations 2𝑥 + 3𝑦 = 12 and 5𝑥 + 2𝑦 =
10 simultaneously.

Next, solve for the objective function value at the corner points and identify the
optimal solution by choosing the maximum value of 𝑧 (since it is a maximization problem).

Corner Points Objective Function Value 𝑧 = 𝑥 + 𝑦


(0,0) 0+0 = 0

(0,4) 0+4 = 4

(2,0) 2+0 = 2
6 40 6 40 46
( , ) + = ≈ 4.1818
11 11 11 11 11

From the table above, we have shown that the optimal solution is found at the point
6 40
( , ), where the objective function is at its maximum with the value of 4.1818. That is, the
11 11
6 40 46
optimal solution is 𝑥 = ,𝑦 = , and 𝑧 = = 4.1818.
11 11 11

Property of and for the exclusive use of SLU. Reproduction, storing in a retrieval system, distributing, uploading or posting online, or transmitting in any form or by any
means, electronic, mechanical, photocopying, recording, or otherwise of any part of this document, without the prior written permission of SLU, is strictly prohibited. 137
Example 112: Solve for the LP model formulated for the problem,
Maximize 𝑧 = 5𝑥 + 4𝑦
subject to
6𝑥 + 4𝑦 ≤ 24
𝑥 + 2𝑦 ≤ 6
𝑦≤2
𝑥 ≥ 0, 𝑦 ≥ 0
Solution:

1. Determine the feasible region

The red line is for 6𝑥 + 4𝑦 ≤ 24, while the green line is for 𝑥 + 2𝑦 ≤ 6, and the blue line
is for 𝑦 ≤ 2. Taking only the intersection of these regions in the first quadrant, we have the
feasible region in the next figure.

2. Determine the optimal solution


The corner points of the feasible region are (0, 0), (0, 2), (2, 2), (3, 1.5) and (4, 0). The
point (2, 2) is obtained by solving the system of equations 𝑥 + 2𝑦 = 6 and 𝑦 = 2 simultaneously.
The point (3, 1.5) is obtained by solving the system of equations 6𝑥 + 4𝑦 = 24 and 𝑥 + 2𝑦 = 6
simultaneously.

Property of and for the exclusive use of SLU. Reproduction, storing in a retrieval system, distributing, uploading or posting online, or transmitting in any form or by any
means, electronic, mechanical, photocopying, recording, or otherwise of any part of this document, without the prior written permission of SLU, is strictly prohibited. 138
The table below summarizes the objective function values at the corner points.

Corner Points Objective Function Value 𝑧 = 5𝑥 + 4𝑦


(0,0) 5(0) + 4(0) = 0

(0,2) 5(0) + 4(2) = 8

(4,0) 5(4) + 4(0) = 20

(2,2) 5(2) + 4(2) = 18

(3,1.5) 5(3) + 4(1.5) = 21

The objective function is maximized at the point (3,1.5) with a value of 21. This means
that a maximum profit of Php 21 million can be achieved if 3 tons of acrylic paint and 1.5
tons of enamel paint are produced in a single day.

Property of and for the exclusive use of SLU. Reproduction, storing in a retrieval system, distributing, uploading or posting online, or transmitting in any form or by any
means, electronic, mechanical, photocopying, recording, or otherwise of any part of this document, without the prior written permission of SLU, is strictly prohibited. 139
Example 113: Bottled water and medical supplies are to be shipped to survivors of an
earthquake by plane. Each container of bottled water will serve 10 people and each
medical kit will aid 6 people. Each plane can carry no more than 80,000 pounds. The
bottled water weighs 20 pounds per container and each medical kit weighs 10 pounds.
Each plane can carry a total volume of supplies that does not exceed 6000 cubic feet.
Each water bottle is 1 cubic foot and each medical kit also has a volume of 1 cubic foot.
Determine how many bottles of water and how many medical kits should be sent
on each plane to maximize the number of earthquake victims who can be assisted.
Solution:

1. Define the decision variables


Let 𝑥 = number of containers containing bottled water to be shipped
𝑦 = number of containers containing medical kits to be shipped

2. Formulate the objective function


The aim is to maximize the number of earthquake survivors (we represent it with 𝑧) that
can be assisted with the bottled water and the medical kit. Since each container of bottled
water will serve 10 people while each medical kit will aid 6 people, the objective function is:

Maximize Number of People Assisted, 𝑧: 𝑧 = 10𝑥 + 6𝑦

3. Identify the constraints


20𝑥 + 10𝑦 ≤ 80000 The bottled water weighs 20 pounds per container while each
medical kit weighs 10 pounds. A plane can carry no more than
8000 pounds.
𝑥 + 𝑦 ≤ 6000 Each water bottle is 1 cubic foot and each medical kit also has
a volume of 1 cubic foot. A plane can carry a total volume of
supplies that does not exceed 6000 cubic feet.
𝑥 ≥ 0, 𝑦 ≥ 0 Nonnegativity constraints

Property of and for the exclusive use of SLU. Reproduction, storing in a retrieval system, distributing, uploading or posting online, or transmitting in any form or by any
means, electronic, mechanical, photocopying, recording, or otherwise of any part of this document, without the prior written permission of SLU, is strictly prohibited. 140
4. Determine the feasible region

The red line is for 20𝑥 + 10𝑦 = 8000 and is shaded below because it is ≤, while the
blue line is for 𝑥 + 𝑦 = 6000 and is shaded below because it is ≤. Taking the intersection that
lies in quadrant 1 because of the nonnegativity constraints 𝑥 ≥ 0 and 𝑦 ≥ 0, we get the
feasible region below.

Feasible
region

Property of and for the exclusive use of SLU. Reproduction, storing in a retrieval system, distributing, uploading or posting online, or transmitting in any form or by any
means, electronic, mechanical, photocopying, recording, or otherwise of any part of this document, without the prior written permission of SLU, is strictly prohibited. 141
5. Determine the optimal solution
The corner points of the feasible region are (0, 0), (0, 6000), (2000, 4000), and (4000,
0). The point (2000, 4000) is obtained by solving the system of equations 20𝑥 + 10𝑦 = 8000
and 𝑥 + 𝑦 = 6000 simultaneously by the method of elimination and substitution. Solving for
the objective function value at each corner point, we get:

Corner Points Objective Function Value 𝑧 = 10𝑥 + 6𝑦


(0,0) 10(0) + 6(0) = 0
(0,6000) 10(0) + 6(6000) = 36000
(2000,4000) 10(2000) + 6(4000) = 44000
(4000,0) 10(4000) + 6(0) = 40000

Thus, the maximum value of 𝑧 is 44000 and this occurs when 𝑥 = 2000 and 𝑦 = 4000. In
practical terms, this means that the maximum number of earthquake survivors who can be
helped with each plane shipment is 44000. This can be accomplished by sending 2000
containers of water bottles and containers of 4000 medical kits.

Property of and for the exclusive use of SLU. Reproduction, storing in a retrieval system, distributing, uploading or posting online, or transmitting in any form or by any
means, electronic, mechanical, photocopying, recording, or otherwise of any part of this document, without the prior written permission of SLU, is strictly prohibited. 142
Learning Reinforcement 8

Directions: Write your solutions and answers on a clean sheet of


paper, or you may print this page and answer there. Submit the
image of your HANDWRITTEN SOLUTIONS as a single pdf file in the
submission bin for this activity in the Classroom. You may use
image scanning apps on your phone (CamScanner or Tap
Scanner) to save several images into one pdf file, or place your
images in a document and save them as a pdf file.

Problem. A garment factory manufactures men’s shirts and women’s blouses. The
production process includes cutting, sewing, and packaging. The factory employs 25
workers in the cutting department, 35 workers in the sewing department, and 5 workers in
the packaging department. Each employee works an 8-hour shift per day for 5 days a week.
The following table gives the time requirements and profit per unit for the two garments:

Minutes per unit

Garment Cutting Sewing Packaging Profit/unit

Shirts 20 70 12 Php 80.00

Blouses 60 60 4 Php 120.00

Determine the optimal weekly production schedule for the garment factory.

Well done! It’s time to answer Quiz 4 and prepare for the FINAL
EXAMINATION!
!

Congratulations! You did it!


You have finished the GMATH
course!

Property of and for the exclusive use of SLU. Reproduction, storing in a retrieval system, distributing, uploading or posting online, or transmitting in any form or by any
means, electronic, mechanical, photocopying, recording, or otherwise of any part of this document, without the prior written permission of SLU, is strictly prohibited. 143
If people do not believe that
mathematics is simple, it is
because they do not realize how
complicated life is. – John Van
Neumann

REFERENCES

A. Main Reference/s
Lipschutz, S. (1998).Schaum’s outline: theory and problems of set theory and related topics.
United States of America: McGraw-Hill Companies, Inc.
Aufmann, R., et al.(2013). Mathematical excursions. United States of America: Cengage
Learning.

B. Books
Anderson, D., et al. (2014). Statistics for business and economics.Stamford, CT :Cengage
Learning.
Barnett, R. et al.(2011). Calculus for business, economics, life sciences, and social sciences.
Boston: Pearson.
Beasley, J. (1990). The mathematics of games(series: recreations in mathematics).Oxford :
Oxford University Press.
Bello,I., et. al.(2014). Topics in contemporary mathematics.Australia :
Brooks/Cole/Cengage Learning.
Blister, R. (2010). College Algbra. Upper Saddle River, New Jersey: Pearson/Prentice Hall.
Dugopolski, M. (1991). Elementary algebra. Reading, Massachusetts : Addison-Wesley.
Fajardo, M.(2000). Simplified mathematics of investment. Quezon City, Philippines : 5138
Merchandising.

Property of and for the exclusive use of SLU. Reproduction, storing in a retrieval system, distributing, uploading or posting online, or transmitting in any form or by any
means, electronic, mechanical, photocopying, recording, or otherwise of any part of this document, without the prior written permission of SLU, is strictly prohibited. 144
Hemphill, G. (1973). A mathematical grammar of English. The Hague : Mouton.
Johnson, R., and Johnson, C.(1975). Algebra : the language of mathematics. Menlo Park,
California : Addison-Wesley Publishing.
Kinney, J. (2015). Probability :an introduction with statistical applications. Hoboken, New
Jersey : John Wiley & Sons.
Kotler, P., and Keller, K. (2009). Marketing management. Upper Saddle River, New Jersey :
Pearson/Prentice Hall.
Lee, C. (2006). Language for learning mathematics assessment for learning in practice.
Berkshire, England : Open University Press
Markowitz, H., andBlay, K. (2014). Risk-return analysis : the theory and practice of rational
investing. New York : McGraw-Hill.
Miller, J. (2014). College Algebra. USA: McGraw-Hill Education LCC
Polya, G. (1957). How To Solve It. 2nd ed., Princeton University Press
Posamentier, A., et.al. (1998). Problem Solving Strategies for Efficient and Elegant Solutions,
A Resource for a Mathematics Teacher. Thousand, Oaks, California: Corwin Press,
INC.
Roussas, G. (2015). An introduction to probability and statistical inference.London :
Academic Press.
Sirug, W. (2010). Mathematics of investment. Manila :Mindshapers.
Weber, J. (1982). Mathematical analysis : business and economic applications. New York :
Harper and Row.
West, D. (2001). Introduction to graph theory. Upper Saddle River, New Jersey : Prentice
Hall.
D. Electronic References
CHED Memorandum Order No. 20, Series of 2013. General Education Curriculum.
https://ched.gov.ph/memorandum-orders/
Diaz, E. (n.d.). Lecture on Mathematics of Voting and Apportionment. Retrieved July 23,
2018, from https://marinmathcircledotorg.files.wordpress.com/2015/12/mmcadv-
20120111-votinglecture-ernestodiaz.pdf
Edward Lewis Dodd,. Fundamentals in the Mathematics of Investment. University of Texas.
Retrieved September 19, 2014 from: http://www.jstor.org/stable/2974341
William L. Hart. Mathematics of Investment. Retrieved September 19, 2014 from
https://archive.org/stream/mathematicsofinv004275mbp/mathematicsofinv004275
mbp_djvu.txt
Dumesnil, J. (2020, June 9).Problem – Solving (Video file) retrieved from
https://www.youtube.com/watch?v=UHHjbeUIVGQ
Maglio,C. ( 2020, June 9). Mathematics: The Language of Numbers. retrieved from
https://jfynet.org/944/mathematics-the-language-of-numbers/
n.a, (2020, June 9). Union of Sets using Venn Diagram. retrieved from https://www.math-
only-math.com/union-of-sets-using-venn-diagram.html
n.a. (2020, June 9). Python Set Difference. retrieved from. retrieved from
https://www.askpython.com/python/set/python-set-difference
n.a (2020, June 9). Python Set Difference. retrieved from
http://amsi.org.au/teacher_modules/pdfs/Sets_and_venn_diagrams.pdf
n.a (2020, June 9). The Map of Mathematics (video file). retrieved from
https://www.youtube.com/watch?v=OmJ-4B-mS-Y

Property of and for the exclusive use of SLU. Reproduction, storing in a retrieval system, distributing, uploading or posting online, or transmitting in any form or by any
means, electronic, mechanical, photocopying, recording, or otherwise of any part of this document, without the prior written permission of SLU, is strictly prohibited. 145
Stewart, I. (2020, June 9). NATURE'S NUMBERS: The Unreal Reality of Mathematics. Retrieved
from https://rctolentino.files.wordpress.com/2018/06/ian-stewart-numerele-
naturii.pdf
n.a (2020, June 9). Mathematics in Nature (image). retrieved from
https://www.google.com/search?q=mathematics+in+NATURE&tbm=isch&ved=2ah
UKEwiLrsDQu-XpAhW9lUsFHcLRALAQ2-
cCegQIABAA&oq=mathematics+in+NATURE&gs_lcp=CgNpbWcQAzIECAAQEzIECA
AQEzIECAAQEzIECAAQEzIECAAQEzIECAAQEzIECAAQEzIECAAQEzIECAAQEzIECAA
QEzoGCAAQHhATUNWfFlikrBZgurMWaABwAHgAgAHEAYgBrgiSAQMwLjaYAQCgAQ
GqAQtnd3Mtd2l6LWltZw&sclient=img&ei=Sn_XXovhJL2rrtoPwqODgAs&bih=626&biw
=1406#imgrc=Vx6GDm5E27ll7M&imgdii=KpoMPVnlmKPm4M
n.a (2020, June 9). Class prayer. retrieved from
https://www.prayerscapes.com/prayers/children/prayer_before_class_starts.html
n.a (2020, June 9). Class prayer. retrieved from https://www.living-
prayers.com/children/prayer_for_a_student.html#student
n.a (2020, June 9). Nature of Mathematics(video File). retrieved from
https://www.youtube.com/watch?v=E3JiQIcoZV8
Tavernetti, W. (2020, June 9). The Beauty and Power of Mathematics(video File). retrieved
from https://www.youtube.com/watch?v=VIbjHIGMjQM
Woo, E. (2020, June 9). Mathematics is the sense you never knew you had(video File).
retrieved from https://www.youtube.com/watch?v=PXwStduNw14
n.a (2020, June 9). MATH ASSIGNMENT ANALYSIS GUIDE. retrieved from
https://1k9gl1yevnfp2lpq1dhrqe17-wpengine.netdna-ssl.com/wp-
content/uploads/2014/09/Math-Assignment-Analysis-Guide.FINAL-4-18.pdf
n.a (2020, June 9). Linear Programming Tutorial (video File). retrieved
fromhttps://www.youtube.com/watch?v=-32jcGMpD2Q
n.a (2020, June 9). Nature by Numbers(video File). retrieved
fromhttps://www.youtube.com/watch?v=kkGeOWYOFoA
n.a (2020, June 9). Real Life Math (video File). retrieved
fromhttps://www.youtube.com/watch?v=HtqlIVN9bh8
Antonsen, R.(2020, June 9). Math is the hidden secret to understanding the world (video
File). retrieved from https://www.youtube.com/watch?v=ZQElzjCsl9o
n.a (2020, June 9). Golden Ratio = Mind Blown! (video file). retrieved from
https://www.youtube.com/watch?v=c8ccsE_IumM
Benjamin, A. (2020, June 9). The magic of Fibonacci numbers (video File). retrieved from
https://www.youtube.com/watch?v=SjSHVDfXHQ4
n.a (2020, June 9). Mathematics, The Language of the Universe [Documentary 2017]
(video file). retrieved from youtube.com/watch?v=S5LuCwZ0bpg
Georges, C. (2020, June 9). My Prayer Time - Instrumental Prayer & Worship Music.retrieved
from.https://www.youtube.com/watch?v=dM_ZNXXucvY&list=PLwUwIvnZLqwaGlwk
g4eKRz84scfvZY88z&index=3

Property of and for the exclusive use of SLU. Reproduction, storing in a retrieval system, distributing, uploading or posting online, or transmitting in any form or by any
means, electronic, mechanical, photocopying, recording, or otherwise of any part of this document, without the prior written permission of SLU, is strictly prohibited. 146
Property of and for the exclusive use of SLU. Reproduction, storing in a retrieval system, distributing, uploading or posting online, or transmitting in any form or by any
means, electronic, mechanical, photocopying, recording, or otherwise of any part of this document, without the prior written permission of SLU, is strictly prohibited. 147

You might also like